Primary Care -- AAFP
Study online at quizlet.com/_h0ow5
1.
=======================================================
Cardiovascular Board Review Questions 01
=======================================================
You see a 23-year-old gravida 1 para 0 for her prenatal
checkup at 38 weeks gestation. She complains of severe
headaches and epigastric pain. She has had an uneventful
pregnancy to date and had a normal prenatal examination 2
weeks ago. Her blood pressure is 140/100 mm Hg. A urinalysis
shows 2+ protein; she has gained 5 lb in the last week, and has
2+ pitting edema of her legs. The most appropriate
management at this point would be: (check one)
D. Admitting the patient to the hospital, treating with parenteral
magnesium sulfate, and planning prompt delivery either vaginally
or by cesarean section. This patient manifests a rapid onset of
preeclampsia at term. The symptoms of epigastric pain and
headache categorize her preeclampsia as severe. These symptoms
indicate that the process is well advanced and that convulsions are
imminent. Treatment should focus on rapid control of symptoms
and delivery of the infant.
A. Strict bed rest at home and reexamination within 48 hours
B. Admitting the patient to the hospital for bed rest and
frequent monitoring of blood pressure, weight, and
proteinuria
C. Admitting the patient to the hospital for bed rest and
monitoring, and beginning hydralazine (Apresoline) to
maintain blood pressure below 140/90 mm Hg
D. Admitting the patient to the hospital, treating with
parenteral magnesium sulfate, and planning prompt delivery
either vaginally or by cesarean section
2.
=======================================================
Cardiovascular Board Review Questions 02
=======================================================
An asymptomatic 3-year-old male presents for a routine
check-up. On examination you notice a systolic heart
murmur. It is heard best in the lower precordium and has a
low, short tone similar to a plucked string or kazoo. It does
not radiate to the axillae or the back and seems to decrease
with inspiration. The remainder of the examination is
normal. Which one of the following is the most likely
diagnosis? (check one)
A. Eisenmenger's syndrome
B. Mitral stenosis
C. Peripheral pulmonic stenosis
D. Still's murmur
E. Venous hum
D. Still's murmur. There are several benign murmurs of childhood
that have no association with physiologic or anatomic abnormalities.
Of these, Still's murmur best fits the murmur described. The cause
of Still's murmur is unknown, but it may be due to vibrations in the
chordae tendinae, semilunar valves, or ventricular wall. A venous
hum consists of a continuous low-pitched murmur caused by the
collapse of the jugular veins and their subsequent fluttering, and it
worsens with inspiration or diastole. The murmur of physiologic
peripheral pulmonic stenosis (PPPS) is caused by physiologic
changes in the newborns pulmonary vessels. PPPS is a systolic
murmur heard loudest in the axillae bilaterally that usually
disappears by 9 months of age. Mitral stenosis causes a diastolic
murmur, and Eisenmenger's syndrome involves multiple
abnormalities of the heart that cause significant signs and
symptoms, including shortness of breath, cyanosis, and
organomegaly, which should become apparent from a routine
history and examination.
3.
=======================================================
Cardiovascular Board Review Questions 03
=======================================================
A 35-year-old African-American female has just returned
home from a vacation in Hawaii. She presents to your office
with a swollen left lower extremity. She has no previous
history of similar problems. Homan's sign is positive, and
ultrasonography reveals a noncompressible vein in the left
popliteal fossa extending distally. Which one of the following
is true in this situation? (check one)
A. Monotherapy with an initial 10-mg loading dose of
warfarin (Coumadin) would be appropriate
B. Enoxaparin (Lovenox) should be administered at a dosage
of 1 mg/kg subcutaneously twice a day
C. The incidence of thrombocytopenia is the same with lowmolecular-weight heparin as with unfractionated heparin
D. The dosage of warfarin should be adjusted to maintain the
INR at 2.5-3.5
E. Anticoagulant therapy should be started as soon as
possible and maintained for 1 year to prevent deep vein
thrombosis (DVT) recurrence
4.
=======================================================
Cardiovascular Board Review Questions 04
=======================================================
A 73-year-old male with COPD presents to the emergency
department with increasing dyspnea. Examination reveals no
sign of jugular venous distention. A chest examination reveals
decreased breath sounds and scattered rhonchi, and the
heart sounds are very distant but no gallop or murmur is
noted. There is +1 edema of the lower extremities. Chest
radiographs reveal cardiomegaly but no pleural effusion. The
patient's B-type natriuretic peptide level is 850 pg/mL (N <100)
and his serum creatinine level is 0.8 mg/dL (N 0.6-1.5). Which
one of the following would be the most appropriate initial
management? (check one)
A. Intravenous heparin
B. Tiotropium (Spiriva)
C. Levalbuterol (Xopenex) via nebulizer
D. Prednisone, 20 mg twice daily for 1 week
E. Furosemide (Lasix), 40 mg intravenously
5.
=======================================================
Cardiovascular Board Review Questions 05
=======================================================
A 35-year-old white male with known long QT syndrome has a
brief episode of syncope requiring cardiopulmonary
resuscitation. Which one of the following is most likely
responsible for this episode? (check one)
A. Sinus tachycardia
B. Atrial flutter with third degree block
C. Asystole
D. Torsades de pointes
B. Enoxaparin (Lovenox) should be administered at a dosage of 1
mg/kg subcutaneously twice a day. The use of low-molecular-weight
heparin allows patients with acute deep vein thrombosis (DVT) to be
managed as outpatients. The dosage is 1 mg/kg subcutaneously
twice daily. Patients chosen for outpatient care should have good
cardiopulmonary reserve, normal renal function, and no risk for
excessive bleeding. Oral anticoagulation with warfarin can be
initiated on the first day of treatment after heparin loading is
completed. Monotherapy with warfarin is inappropriate. The
incidence of thrombocytopenia with low-molecular-weight heparin
is lower than with conventional heparin. The INR should be
maintained at 2.0-3.0 in this patient. The 2.5-3.5 range is used for
patients with mechanical heart valves. The therapeutic INR should
be maintained for 3-6 months in a patient with a first DVT related to
travel.
E. Furosemide (Lasix), 40 mg intravenously. B-type natriuretic
peptide (BNP) is secreted in the ventricles and is sensitive to
changes in left ventricular function. Concentrations correlate with
end-diastolic pressure, which in turn correlates with dyspnea and
congestive heart failure. BNP levels can be useful when trying to
determine whether dyspnea is due to cardiac, pulmonary, or
deconditioning etiologies. A value of less than 100 pg/mL excludes
congestive heart failure as the cause for dyspnea. If it is greater than
400 pg/mL, the likelihood of congestive heart failure is 95%. Patients
with values of 100-400 pg/mL need further investigation. There are
some pulmonary problems that may elevate BNP, such as lung
cancer, cor pulmonale, and pulmonary embolus. However, these
patients do not have the same extent of elevation that those with
acute left ventricular dysfunction will have. If these problems can be
ruled out, then individuals with levels between 100-400 pg/mL most
likely have congestive heart failure. Initial therapy should be a loop
diuretic. It should be noted that BNP is partially excreted by the
kidneys, so levels are inversely proportional to creatinine clearance.
D. Torsades de pointes. Patients with long QT syndrome that have
sudden arrhythmia death syndrome usually have either torsades
de pointes or ventricular fibrillation. Sinus tachycardia would not
explain the syncope, and atrial flutter and asystole are not usual in
long QT syndrome.
6.
=======================================================
Endocrine Board Review Questions 01
=======================================================
A 49-year-old female who takes multiple medications has a
chemistry profile as part of her routine monitoring. She is
found to have an elevated calcium level. All other values on
the profile are normal, and the patient is not currently
symptomatic. Follow-up testing reveals a serum calcium level
of 11.2 mg/dL (N 8.4-10.2) and an intact parathyroid hormone
level of 80 pg/mL (N 10-65). Which one of the following should
be discontinued for 3 months before repeat laboratory
evaluation and treatment? (check one)
A. Lithium. Lithium therapy can elevate calcium levels by elevating
parathyroid hormone secretion from the parathyroid gland. This
duplicates the laboratory findings seen with mild primary
hyperparathyroidism. If possible, lithium should be discontinued
for 3 months before reevaluation (SOR C). This is most important for
avoiding unnecessary parathyroid surgery. Vitamin D and calcium
supplementation could contribute to hypercalcemia in rare
instances, but they would not cause elevation of parathyroid
hormone. Raloxifene has actually been shown to mildly reduce
elevated calcium levels, and furosemide is used with saline infusions
to lower significantly elevated calcium levels.
A. Lithium
B. Furosemide (Lasix)
C. Raloxifene (Evista)
D. Calcium carbonate
E. Vitamin D
7.
=======================================================
Endocrine Board Review Questions 02
=======================================================
In a patient with a solitary thyroid nodule, which one of the
following is associated with a higher incidence of
malignancy? (check one)
A. Hoarseness. When evaluating a patient with a solitary thyroid
nodule, red flags indicating possible thyroid cancer include male
gender; age <20 years or >65 years; rapid growth of the nodule;
symptoms of local invasion such as dysphagia, neck pain, and
hoarseness; a history of head or neck radiation; a family history of
thyroid cancer; a hard, fixed nodule >4 cm; and cervical
lymphadenopathy.
A. Hoarseness
B. Hyperthyroidism
C. Female gender
D. A nodule size of 2 cm
E. A freely movable nodule
8.
=======================================================
Gastrointestinal Board Review Questions 01
=======================================================
A 36-hour-old male is noted to have jaundice extending to the
abdomen. He is breastfeeding well, 10 times a day, and is
voiding and passing meconium-stained stool. He was born by
normal spontaneous vaginal delivery at 38 weeks gestation
after an uncomplicated pregnancy. The mother's blood type
is A positive with a negative antibody screen. The infants total
serum bilirubin is 13.0 mg/dL. Which one of the following
would be the most appropriate management of this infants
jaundice? (check one)
A. Continue breastfeeding and supplement with water or
dextrose in water to prevent dehydration
B. Continue breastfeeding, evaluate for risk factors, and
initiate phototherapy if at risk
C. Discontinue breastfeeding and supplement with formula
until the jaundice resolves
D. Discontinue breastfeeding and supplement with formula
until total serum bilirubin levels begin to decrease
B. Continue breastfeeding, evaluate for risk factors, and initiate
phototherapy if at risk. In 2004 the American Academy of Pediatrics
published updated clinical practice guidelines on the management
of hyperbilirubinemia in the newborn infant at 35 or more weeks
gestation. These guidelines focus on frequent clinical assessment of
jaundice, and treatment based on the total serum bilirubin level, the
infants age in hours, and risk factors. Phototherapy should not be
started based solely on the total serum bilirubin level. The
guidelines encourage breastfeeding 8-12 times daily in the first few
days of life to prevent dehydration. There is no evidence to support
supplementation with water or dextrose in water in a
nondehydrated breastfeeding infant. This infant is not dehydrated
and is getting an adequate number of feedings, and there is no
reason to discontinue breastfeeding at this time.
9.
=======================================================
Gastrointestinal Board Review Questions 02
=======================================================
For 2 weeks, a 62-year-old male with biopsy-documented cirrhosis
and ascites has had diffuse abdominal discomfort, fever, and night
sweats. His current medications are furosemide (Lasix) and
spironolactone (Aldactone). On examination, his temperature is
38.0 C (100.4 F), blood pressure 100/60 mm Hg, heart rate 92
beats/min and regular. The heart and lung examination is normal.
The abdomen is soft with vague tenderness in all quadrants. There is
no rebound or guarding. The presence of ascites is easily verified.
Bowel sounds are quiet. The rectal examination is normal, and the
stool is negative for occult blood. You perform diagnostic
paracentesis and send a sample of fluid for analysis. Which one of the
following findings would best establish the suspected diagnosis of
spontaneous bacterial peritonitis? (check one)
C. Neutrophil count >300/mL. Diagnostic paracentesis is
recommended for patients with ascites of recent onset, as
well as for those with chronic ascites who present with new
clinical findings such as fever or abdominal pain. A
neutrophil count >250/mL is diagnostic for peritonitis. Once
peritonitis is diagnosed, antibiotic therapy should be
started immediately without waiting for culture results.
Bloody ascites with abnormal cytology may be seen with
hepatoma, but is not typical of peritonitis. The ascitic fluid
pH does not become abnormal until well after the
neutrophil count has risen, so it is a less reliable finding for
treatment purposes. A protein level >1 g/dL is actually
evidence against spontaneous bacterial peritonitis.
A. pH <7.2
B. Bloody appearance
C. Neutrophil count >300/mL
D. Positive cytology
E. Total protein >1 g/dL
10.
=======================================================
Gastrointestinal Board Review Questions 03
=======================================================
A 32-year-old meat cutter comes to your office with persistent
symptoms of nausea, vomiting, and diarrhea which began about 36
hours ago on the last day of a 5-day Caribbean cruise. His wife was
sick during the first 2 days of the cruise with similar symptoms. On
the ship, they both ate the "usual foods" in addition to oysters.
Findings on examination are negative, and a stool specimen is
negative for white cells. Which one of the following is the most likely
cause of his illness? (check one)
A. Escherichia coli
B. Rotavirus
C. Norwalk virus
D. Hepatitis A
E. Giardia species
C. Norwalk virus. Recent reports of epidemics of
gastroenteritis on cruise ships are consistent with Norwalk
virus infections due to waterborne or foodborne spread.
In the United States, these viruses are responsible for
about 90% of all epidemics of nonbacterial gastroenteritis.
The Norwalk-like viruses are common causes of
waterborne epidemics of gastroenteritis, and have been
shown to be responsible for outbreaks in nursing homes,
on cruise ships, at summer camps, and in schools.
Symptomatic treatment is usually appropriate.
11.
=======================================================
Integumentary Board Review Questions 01
=======================================================
A 45-year-old white male consults you because of a painless,
circular, 1-cm white spot inside his mouth, which he noticed
3 days ago. You are treating him with propranolol (Inderal)
for hypertension, and you know him to be a heavy alcohol
user. After a careful physical examination, your tentative
diagnosis is leukoplakia of the buccal mucosa. You elect to
observe the lesion for 2 weeks. On the patients return, the
lesion is still present and unchanged in appearance. The best
course of management at this time is to (check one)
E. perform a biopsy of the lesion. Leukoplakia is a white keratotic
lesion seen on mucous membranes. Irritation from various
mechanical and chemical stimuli, including alcohol, favors
development of the lesion. Leukoplakia can occur in any area of the
mouth and usually exhibits benign hyperkeratosis on biopsy. On
long-term follow-up, 2%-6% of these lesions will have undergone
malignant transformation into squamous cell carcinoma. Oral
nystatin would not be appropriate treatment, as this lesion is not
typical of oral candidiasis. Candidal lesions are usually multiple and
spread quickly when left untreated. A fluorescent antinuclear
antibody test is also not indicated, as the oral lesions of lupus
erythematosus are typically irregular, erosive, and necrotic. An
idiosyncratic reaction to propranolol is unlikely in this patient.
A. reassure the patient and continue to observe
B. discontinue propranolol
C. treat with oral nystatin
D. order a fluorescent antinuclear antibody test
E. perform a biopsy of the lesion
12.
=======================================================
Integumentary Board Review Questions 02
=======================================================
Your hospital administrator asks you to develop a
community screening program for melanoma. Which one of
the following is true concerning screening for this disease?
(check one)
A. Screening for melanoma is not indicated since the disease
is rare
B. Screening for melanoma is not indicated since screening
takes too much time
C. No definite clinical evidence has shown that screening for
melanoma reduces mortality
D. Because of sunbathing, female patients are the most
important population to screen
13.
=======================================================
Musculoskeletal Board Review Questions 01
=======================================================
Osteoporotic bone loss can be caused or accelerated by
prolonged use of which one of the following medications?
(check one)
A. Hydrochlorothiazide
B. Phenytoin
C. Raloxifene (Evista)
D. Diazepam (Valium)
E. Fluoxetine (Prozac)
C. No definite clinical evidence has shown that screening for
melanoma reduces mortality. There have been no randomized,
controlled trials or other definitive data to indicate that screening
for melanoma reduces mortality. There are, however, factors which
indicate that screening would be beneficial, including the
increasing prevalence of the disease and the fact that screening is
time-effective and safe. If screening is performed, populations at
greatest risk should be considered. Men, especially those over age
50, have the highest incidence of melanoma.
B. Phenytoin. Secondary osteoporosis can result from a variety of
endocrine, nutritional, or genetic disorders, as well as from
prolonged use of certain medications. Anticonvulsants such as
phenytoin increase the hepatic metabolism of vitamin D, thereby
reducing intestinal calcium absorption. Other medications that
adversely affect bone mineral density include glucocorticoids,
cyclosporine, phenobarbital, and heparin. Thiazide diuretics
reduce urinary calcium loss and are believed to preserve bone
density with long-term use. Benzodiazepines and SSRIs have not
been associated with increases in bone loss or in hip fractures.
Raloxifene, a selective estrogen receptor modulator, is indicated for
the prevention and treatment of osteoporosis in postmenopausal
women.
14.
=======================================================
Musculoskeletal Board Review Questions 02
=======================================================
An overweight 13-year-old male presents with a 3-week
history of right lower thigh pain. He first noticed the pain
when jumping while playing basketball, but now it is present
even when he is just walking. On examination he can bear his
full weight without an obvious limp. There is no localized
tenderness, and the patella tracks normally without
subluxation. Internal rotation of the hip is limited on the
right side compared to the left. Based on the examination
alone, which one of the following is the most likely
diagnosis? (check one)
A. Avascular necrosis of the femoral head (Legg-Calv-Perthes
disease)
B. Osteosarcoma
C. Meralgia paresthetica
D. Pauciarticular juvenile rheumatoid arthritis
E. Slipped capital femoral epiphysis
15.
=======================================================
Musculoskeletal Board Review Questions 03
=======================================================
A 21-year-old white female presents to the emergency
department with a history consistent with a lateral ankle
sprain that occurred 2 hours ago while she was playing
softball. She complains of pain over the distal anterior
talofibular ligament, but is able to bear weight. There is mild
swelling, mild black and blue discoloration, and moderate
tenderness to palpation over the insertion of the anterior
talofibular ligament, but the malleoli are nontender to
palpation. Which one of the following statements is true
regarding the management of this case? (check one)
A. Anteroposterior, lateral, and 30 degrees internal oblique
(mortise view) radiographs should be done to rule out
fracture
B. Stress radiographs will be needed to rule out a major
partial or complete ligamentous tear
C. The patient should use crutches and avoid weight bearing
for 10-14 days
D. Early range-of-motion exercises should be initiated to
maintain flexibility
E. For best results, functional rehabilitation should begin
within the first 24 hours after injury
E. Slipped capital femoral epiphysis. This is a classic presentation for
slipped capital femoral epiphysis (SCFE) in an adolescent male who
has probably had a recent growth spurt. Pain with activity is the
most common presenting symptom, as opposed to the nighttime
pain that is typical of malignancy. Obese males are affected more
often. The pain is typically in the anterior thigh, but in a high
percentage of patients the pain may be referred to the knee, lower
leg, or foot. Limited internal rotation of the hip, especially with the
hip in 90; flexion, is a reliable and specific finding for SCFE and
should be looked for in all adolescents with hip, thigh, or knee pain.
Meralgia paresthetica is pain in the thigh related to entrapment of
the lateral femoral cutaneous nerve, often attributed to excessively
tight clothing. Legg-Calv-Perthes disease (avascular or aseptic
necrosis of the femoral head) is more likely to occur between the
ages of 4 and 8 years. Juvenile rheumatoid arthritis typically is
associated with other constitutional symptoms including stiffness,
fever, and pain in at least one other joint, with the pain not
necessarily associated with activity.
D. Early range-of-motion exercises should be initiated to maintain
flexibility. This patient has an uncomplicated lateral ankle sprain
and requires minimal intervention. The Ottawa ankle rules were
developed to determine when radiographs are needed for ankle
sprains. In summary, ankle radiographs should be done if the
patient has pain at the medial or lateral malleolus and either bone
tenderness at the back edge or tip of the lateral or medial
malleolus, or an inability to bear weight immediately after the injury
or in the emergency department, or both. If the patient complains
of midfoot pain and/or bone tenderness at the base of the fifth
metatarsal or navicular, or an inability to bear weight, radiographs
should be ordered. Sprains can be differentiated from major
partial or complete ligamentous tears by anteroposterior, lateral,
and 30 degrees internal oblique (mortise view) radiographs. If the
joint cleft between either malleolus and the talus is >4 mm, a major
ligamentous tear is probable. Stress radiographs in forced
inversion are sometimes helpful to demonstrate stability, but ankle
instability can be present with a normal stress radiograph. Grade I
and II ankle sprains are best treated with RICE (rest, ice,
compression, elevation) and an air splint for ambulation. NSAIDs
are used for control of pain and inflammation. Heat should not be
applied. Early range-of-motion exercises should be initiated to
maintain flexibility. Weight bearing is appropriate as tolerated and
functional rehabilitation should be started when pain permits.
Exercises on a balance board will help develop coordination.
16.
=======================================================
Neurologic Board Review Questions 01
=======================================================
A 16-year-old white female is brought to your office because
she has been "passing out." She tells you that on several
occasions while playing in the high-school band at the end of
the half-time show she has "blacked out." She describes
feeling lightheaded with spots before her eyes and tunnel
vision just prior to falling. Friends in the band have told her
that she appears to be pale and sweaty when these episodes
occur. No seizure activity has ever been observed. In each
instance she regains consciousness almost immediately;
there is no postictal state. She has been seen in the
emergency department for this on two occasions with
normal vital signs, physical findings, and neurologic findings.
A CBC, a metabolic profile, and an EKG are also normal.
Which one of the following tests is most likely to yield the
correct diagnosis? (check one)
A. A sleep-deprived EEG
B. 24-hour Holter monitoring
C. A pulmonary/cardiac stress test
D. An echocardiogram
E. Tilt table testing
17.
=======================================================
Neurologic Board Review Questions 02
=======================================================
You evaluate an 80-year-old white male who is a heavily
medicated chronic schizophrenic. You note constant,
involuntary chewing motions and repetitive movements of
his legs. Which one of the following is the most likely
diagnosis? (check one)
A. Neuroleptic malignant syndrome
B. Acute dystonia
C. Huntington's disease
D. Tardive dyskinesia
E. Oculogyric crisis
E. Tilt table testing. Reflex syncope is a strong diagnostic
consideration for episodes of syncope associated with a
characteristic precipitating factor. The major categories of syncope
include carotid sinus hypersensitivity, and neurally mediated and
situational syncopes. The most common and benign forms of
syncope are neurally mediated or vasovagal types with sudden
hypotension, frequently accompanied by bradycardia. Other
terms for this include neurocardiogenic, vasomotor, neurovascular,
or vasodepressive syncope. Most patients are young and otherwise
healthy. The mechanism of the syncope seems to be a period of
high sympathetic tone (often induced by pain or fear), followed by
sudden sympathetic withdrawal, which then triggers a paradoxical
vasodilatation and hypotension. Attacks occur with upright
posture, often accompanied by a feeling of warmth or cold
sweating, lightheadedness, yawning, or dimming of vision. If the
patient does not lie down quickly he or she will fall, with the
horizontal position allowing a rapid restoration of central profusion.
Recovery is rapid, with no focal neurologic sense of confusion or
headache. The event can be duplicated with tilt testing,
demonstrating hypotension and bradycardia.
D. Tardive dyskinesia. The patient has classic signs of tardive
dyskinesia. Repetitive movement of the mouth and legs is caused
by antipsychotic agents such as phenothiazines and haloperidol.
Neuroleptic malignant syndrome consists of fever, autonomic
dysfunction, and movement disorder. Acute dystonia involves
twisting of the neck, trunk, and limbs into uncomfortable positions.
Huntington's disease causes choreic movements, which are flowing,
not repetitive. Oculogyric crisis involves the eyes.
18.
=======================================================
Patient-Based Systems Board Review Questions
=======================================================
The American College of Obstetricians and Gynecologists and
the American Academy of Pediatrics support the advance
provision of drugs and instructions for emergency
contraception to sexually active women, so that they have
ready access to them if they are needed. The evidence shows
that advance provision of emergency contraception (check
one)
A. decreases pregnancy rates on a population level
B. decreases the time from unprotected sex to use of
emergency contraception
C. decreases contraception use by the patient prior to sexual
activity
D. increases rates of sexually transmitted infection
E. increases rates of unprotected intercourse
19.
=======================================================
Population-Based Care Board Review Questions
=======================================================
Which one of the following is recommended for routine
prenatal care? (check one)
B. decreases the time from unprotected sex to use of emergency
contraception. A Cochrane review including randomized,
controlled trials (RCTs) compared standard access to emergency
contraception (EC) with advance provision. The review found eight
trials, five of which were conducted in the U.S. Two of the RCTs
were sufficiently powered to show a difference in pregnancy rates.
No study showed that giving advance EC reduced pregnancy rates
on a population level. However, women who were provided with
advance EC took the pills an average of approximately 15 hours
sooner than women without advance access. Five studies that
reported on contraception use did not show a difference in type or
frequency of regular contraception use among women who were
provided advance EC. Women randomized to the advance EC
groups were 2.5 times more likely to use EC once, and 4 times more
likely to use it 2 or more times, compared to those without advance
access. Three studies reported rates of sexually transmitted
infection and none found differences between the advance and
standard access EC groups. Six studies reported rates of
unprotected sexual intercourse and found no difference. The
Cochrane review concludes that advance access to EC appears to
be safe, but does not reduce pregnancy on a population level.
However, advance provision might be beneficial because it
increases the speed and frequency of EC use.
D. HIV screening. HIV screening is recommended as part of routine
prenatal care, even in low-risk pregnancies. Counseling about
cystic fibrosis carrier testing is recommended, but not routine
testing. Hepatitis C and parvovirus antibodies are not part of
routine prenatal screening. Routine screening for bacterial
vaginosis with a vaginal smear for clue cells is not recommended.
A. Hepatitis C antibody testing
B. Parvovirus antibody testing
C. Cystic fibrosis carrier testing
D. HIV screening
E. Examination of a vaginal smear for clue cells
20.
=======================================================
Psychogenic Board Review Questions 01
=======================================================
Which one of the following sleep problems in children is
most likely to occur during the second half of the night?
(check one)
A. Confusional arousals
B. Sleepwalking
C. Sleep terrors
D. Nightmares
D. Nightmares. Nightmares occur in the second half of the night,
when rapid eye movement (REM) sleep is most prominent.
Parasomnias, including sleepwalking, confusional arousal, and
sleep terrors, are disorders of arousal from non-REM (NREM) sleep.
These are more common in children than adults because children
spend more time in deep NREM sleep. Such disorders usually
occur within 1-2 hours after sleep onset, and coincide with the
transition from the first period of slow-wave sleep.
21.
=======================================================
Psychogenic Board Review Questions 02
=======================================================
A 34-year-old white male presents with a history and findings
that satisfy DSM-IV criteria for bipolar disorder. Which one of
the following treatment options is the most effective for
long-term management of the majority of patients with this
disorder? (check one)
A. Electroconvulsive therapy (ECT)
B. Tricyclic antidepressants
C. SSRIs
D. Monoamine oxidase (MAO) inhibitors
E. Lithium
22.
=======================================================
Psychogenic Board Review Questions 03
=======================================================
The mother of a 3-year-old male is concerned that he doesn't
like being held, doesn't interact much with other children,
and rarely smiles. Of the following, which feature would be
most helpful in distinguishing Asperger's syndrome from
autism in this patient? (check one)
A. Normal language development
B. Delayed gross motor development
C. Repetitive fine motor mannerisms
D. Preoccupation with parts of objects
E. Focused patterns of intense interest
23.
=======================================================
Psychogenic Board Review Questions 04
=======================================================
An 85-year-old white male with terminal pancreatic cancer is
expected to survive for another 2 weeks. His pain has been
satisfactorily controlled with sustained-release morphine. He
has now developed a disturbed self-image, hopelessness, and
anhedonia, and has told family members that he has thought
about suicide. Psychomotor retardation is also noted. His
family is supportive. His daughter feels he is depressed, while
his son feels this is more of a grieving process. Which one of
the following would be most appropriate for managing this
problem? (check one)
A. Reassurance
B. Alprazolam (Xanax)
C. Trazodone (Desyrel)
D. Olanzapine (Zyprexa)
E. Methylphenidate (Ritalin)
E. Lithium. Electroconvulsive therapy (ECT) is as effective as
medication for the acute treatment of the severe depression
and/or mania of bipolar disorder. However, ECT should be
reserved for patients with severe mood syndromes who may be
unable to wait for mood-stabilizing drugs to take effect. Neuroleptic
(antipsychotic) drugs are effective in acute mania, but are not
recommended for long-term use because of side effects. Bipolar
depression generally responds to tricyclic antidepressants, SSRIs,
and MAO inhibitors, but when used as long-term therapy these
drugs may induce episodes of mania. Anticonvulsants, such as
carbamazepine, valproic acid, and benzodiazepines, have been
useful adjuncts combined with lithium in patients with
breakthrough episodes of mania and/or depression. Lithium is the
classic mood stabilizer. It has been shown to have antimanic
efficacy, prophylactic efficacy in bipolar disorder, and some efficacy
in prophylaxis against bipolar depression. Lithium remains the
drug of choice for long-term treatment of the majority of patients
with bipolar illness.
A. Normal language development. The DSM-IV categorizes
Asperger's syndrome and autism as pervasive developmental
disorders. In both conditions, children have significant difficulties
with social interactions, although the impairment is more severe
and sustained in autism. Both Asperger's and autism may be
associated with symptoms of repetitive motor mannerisms,
restricted patterns of interest (which are abnormal in focus or
intensity), or preoccupation with parts of objects. However, unlike
children with Asperger's syndrome, autistic children have serious
problems with communication skills, either in the development of
speech itself or in the ability to carry on a conversation. Normal,
age-appropriate language skills in a 3-year-old would rule out a
diagnosis of autism. It is an important distinction to make, as the
prognosis for independent functioning in children with Asperger's
syndrome is significantly better than in children with autism.
E. Methylphenidate (Ritalin). Distinguishing between preparatory
grief and depression in a dying patient is not always simple. Initially
one should evaluate for unresolved physical symptoms and treat
any that are present. If the patient remains in distress, mood
should be evaluated. If it waxes and wanes with time and if selfesteem is normal, this is likely preparatory grief. The patient may
have fleeting thoughts of suicide and likely will express worry
about separation from loved ones. This usually responds to
counseling. In patients with anhedonia, persistent dysphoria,
disturbed self-image, hopelessness, poor sense of self-worth,
rumination about death and suicide, or an active desire for early
death, depression is the problem. For patients who are expected to
live only a few days, psychostimulants such as methylphenidate
should be used. For those who are expected to survive longer,
SSRIs are a good choice.
24.
=======================================================
Random Board Review Questions 01
=======================================================
A 30-year-old previously healthy male comes to your office
with a 1-year history of frequent abdominal pain, nonbloody
diarrhea, and a 20-lb weight loss. He has no history of travel
outside the United States, antibiotic use, or consumption of
well water. His review of systems is notable for a chronic,
intensely pruritic rash that is vesicular in nature. His review
of systems is otherwise negative and he is on no medications.
The most likely cause of his symptoms is: (check one)
A. lactose intolerance
B. irritable bowel syndrome
C. collagenous colitis
D. celiac sprue
E. Crohn's disease
25.
=======================================================
Random Board Review Questions 02
=======================================================
You see a 22-year-old female who sustained a right knee
injury in a recent college soccer game.She is a defender and
executed a sudden cutting maneuver. With her right foot
planted and her ankle locked, she attempted to shift the
position of her body to stop an oncoming ball and felt her
knee pop. She has had a moderate amount of pain and
swelling, which began within 2 hours of the injury, but she is
most concerned about the loss of knee hyperextension.
Which one of the following tests is most likely to be
abnormal in this patient? (check one)
A. Anterior drawer
B. Lachman
C. McMurray
D. Pivot shift
D. celiac sprue. Celiac sprue is an autoimmune disorder
characterized by inflammation of the small bowel wall, blunting of
the villi, and resultant malabsorption. Symptoms commonly include
diarrhea, fatigue, weight loss, abdominal pain, and borborygmus;
treatment consists of elimination of gluten proteins from the diet.
Extraintestinal manifestations are less common but may include
elevated transaminases, osteopenia, and iron deficiency anemia.
Serum IgA tissue transglutaminase (TTG) antibodies are highly
sensitive and specific for celiac sprue, and a small bowel biopsy
showing villous atrophy is the gold standard for diagnosis. This
patient's rash is consistent with dermatitis herpetiformis, which is
pathognomonic for celiac sprue and responds well to a strict
gluten-free diet.
Lactose intolerance, irritable bowel syndrome, collagenous colitis,
and Crohn's disease are in the differential diagnosis for celiac
sprue. However, significant weight loss is not characteristic of
irritable bowel syndrome or lactose intolerance. The diarrhea
associated with Crohn's disease is typically bloody. Collagenous
colitis does cause symptoms similar to those experienced by this
patient, but it is not associated with dermatitis herpetiformis.
B. Lachman. Anterior cruciate ligament (ACL) tears occur more
commonly in women than in men. The intensity of play is also a
factor, with a much greater risk of ACL injuries occurring during
games than during practices. The most accurate maneuver for
detecting an ACL tear is the Lachman test (sensitivity 60%-100%,
mean 84%), followed by the anterior drawer test (sensitivity 9%93%, mean 62%) and the pivot shift test (sensitivity 27%-95%, mean
62%) (SOR C). McMurray's test is used to detect meniscal tears.
26.
=======================================================
Random Board Review Questions 03
=======================================================
As the medical review officer for a local business, you are
required to interpret urine drug tests. Assuming the sample
was properly collected and handled, which one of the
following test results is consistent with the history provided
and should be reported as a negative test? (check one)
A. Diazepam (Valium) identified in an employee taking
oxazepam prescribed by a physician
B. Morphine identified in an employee undergoing a
prescribed methadone pain management program
C. Morphine identified in an employee taking a prescribed
cough medicine containing codeine
D. Tetrahydrocannabinol above the threshold value in an
employee who reports secondary exposure to marijuana
E. Tetrahydrocannabinol identified in an employee taking
prescribed tramadol (Ultram)
27.
=======================================================
Random Board Review Questions 04
=======================================================
A 30-year-old female asks you whether she should have a
colonoscopy, as her father was diagnosed with colon cancer
at the age of 58. There are no other family members with a
history of colon polyps or cancer.
You recommend that she have her first screening
colonoscopy: (check one)
A. now and every 5 years if normal
B. now and every 10 years if normal
C. at age 40 and then every 5 years if normal
D. at age 40 and then every 10 years if normal
E. at age 50 and then every 5 years if normal
C. Morphine identified in an employee taking a prescribed cough
medicine containing codeine. Results of urine drug test panels
obtained in the workplace are reported by a Medical Review
Officer (MRO) as positive, negative, dilute, refusal to test, or test
canceled; the drug/metabolite for which the test is positive or the
reason for refusal (e.g., the presence of an adulterant) or
cancellation is also included in the final report. The MRO
interpretation is based on consideration of many factors, including
the confirmed patient medical history, specimen collection process,
acceptability of the specimen submitted, and qualified laboratory
measurement of drugs or metabolites in excess of the accepted
thresholds. These thresholds are set to preclude the possibility that
secondary contact with smoke, ingestion of poppy seeds, or similar
exposures will result in an undeserved positive urine drug screen
report. Other findings, such as the presence of behavioral or
physical evidence of unauthorized use of opiates, may also factor
into the final report.
When a properly collected, acceptable specimen is found to contain
drugs or metabolites that would be expected based on a review of
confirmed prescribed use of medications, the test is reported as
negative. Morphine is a metabolite of codeine that may be found in
the urine of someone taking a codeine-containing medication;
morphine is not a metabolite of methadone. Oxazepam is a
metabolite of diazepam but the reverse is not true.
Tetrahydrocannabinol would not be found in the urine as a result
of tramadol use.
C. at age 40 and then every 5 years if normal. Patients should be
risk-stratified according to their family history. Patients who have
one first degree relative diagnosed with colorectal cancer or
adenomatous polyps before age 60, or at least two second degree
relatives with colorectal cancer, are in the highest risk group. They
should start colon cancer screening at age 40, or 10 years before
the earliest age at which an affected relative was diagnosed
(whichever comes first) and be rescreened every 5 years.
Colonoscopy is the preferred screening method for this highestrisk group, as high-risk patients are more likely to have right-sided
colon lesions that would not be detected with sigmoidoscopy.
28.
=======================================================
Random Board Review Questions 05
=======================================================
Which one of the following is most consistent with obsessivecompulsive disorder in adults? (check one)
A. Impulses related to excessive worry about real-life
problems
B. A belief by the patient that obsessions are not produced by
his or her own mind, but are "inserted" thoughts
C. Recognition by the patient that the obsessions or
compulsions are excessive or unreasonable
D. Compulsions that bring relief to the patient rather than
causing distress
E. Full remission with treatment
29.
=======================================================
Random Board Review Questions 06
=======================================================
Which one of the following patients should be advised to
take aspirin, 81 mg daily, for the primary prevention of
stroke? (check one)
A. A 42-year-old male with a history of hypertension
B. A 72-year-old female with no chronic medical conditions
C. An 80-year-old male with a history of depression
D. An 87-year-old female with a history of peptic ulcer disease
C. Recognition by the patient that the obsessions or compulsions
are excessive or unreasonable. The DSM-IV criteria for obsessivecompulsive disorder (OCD) indicate that the patient at some point
recognizes that the obsessions or compulsions are excessive or
unreasonable. The impulses of OCD are not related to excessive
worry about one's problems, and the patient recognizes that they
are the product of his or her own mind. In addition, the patient
experiences marked distress because of the impulses. Full
remission is rare, but treatment can provide significant relief.
B. A 72-year-old female with no chronic medical conditions. The U.S.
Preventive Services Task Force (USPSTF) has summarized the
evidence for the use of aspirin in the primary prevention of
cardiovascular disease as follows:
The USPSTF recommends the use of aspirin for men 45-79 years of
age when the potential benefit from a reduction in myocardial
infarctions outweighs the potential harm from an increase in
gastrointestinal hemorrhage (Grade A recommendation)
The USPSTF recommends the use of aspirin for women 55-79 years
of age when the potential benefit of a reduction in ischemic strokes
outweighs the potential harm of an increase in gastrointestinal
hemorrhage (Grade A recommendation)
The USPSTF concludes that the current evidence is insufficient to
assess the balance of benefits and harms of aspirin for
cardiovascular disease prevention in men and women 80 years of
age or older (Grade I statement)
The USPSTF recommends against the use of aspirin for stroke
prevention in women younger than 55 and for myocardial
infarction prevention in men younger than 45 (Grade D
recommendation)
In summary, consistent evidence from randomized clinical trials
indicates that aspirin use reduces the risk for cardiovascular
disease events in adults without a history of cardiovascular disease.
It reduces the risk for myocardial infarction in men, and ischemic
stroke in women. Consistent evidence shows that aspirin use
increases the risk for gastrointestinal bleeding, and limited
evidence shows that aspirin use increases the risk for hemorrhagic
strokes. The overall benefit in the reduction of cardiovascular
disease events with aspirin use depends on baseline risk and the
risk for gastrointestinal bleeding.
30.
=======================================================
Random Board Review Questions 07
=======================================================
Breastfeeding a full-term, healthy infant is contraindicated
when which one of the following maternal conditions is
present? (check one)
A. Chronic hepatitis B infection
B. Seropositive cytomegalovirus carrier state
C. Current tobacco smoking
D. Herpes simplex viral lesions on the breasts
E. Undifferentiated fever
D. Herpes simplex viral lesions on the breasts. Breastfeeding
provides such optimal nutrition for an infant that the benefits still
far outweigh the risks even when the mother smokes tobacco, tests
positive for hepatitis B or C virus, or develops a simple
undifferentiated fever. Maternal seropositivity to cytomegalovirus
(CMV) is not considered a contraindication except when it has a
recent onset or in mothers of low birthweight infants. When
present, the CMV load can be substantially reduced by freezing
and pasteurization of the milk. All patients who smoke should be
strongly encouraged to discontinue use of tobacco, particularly in
the presence of infants, but smoking is not a contraindication to
breastfeeding.
Mothers with active herpes simplex lesions on a breast should not
feed their infant from the infected breast, but may do so from the
other breast if it is not infected. Breastfeeding is also
contraindicated in the presence of active maternal tuberculosis,
and following administration or use of radioactive isotopes,
chemotherapeutic agents, "recreational" drugs, or certain
prescription drugs.
31.
=======================================================
Random Board Review Questions 08
=======================================================
You are asked to perform a preoperative evaluation on a 55year-old white female with type 2 diabetes mellitus prior to
elective femoral-anterior tibial artery bypass surgery. She is
unable to climb a flight of stairs or do heavy work around the
house. She denies exertional chest pain, and is otherwise
healthy.
Based on current guidelines, which one of the following
diagnostic studies would be appropriate prior to surgery
because the results could alter the management of this
patient? (check one)
A. Pulmonary function studies
B. Coronary angiography
C. Carotid angiography
D. A dipyridamole-thallium scan
E. A hemoglobin A1c level
D. A dipyridamole-thallium scan. Family physicians are often asked
to perform a preoperative evaluation prior to noncardiac surgery.
This requires an assessment of the perioperative cardiovascular
risk of the procedure involved, the functional status of the patient,
and clinical factors that can increase the risk, such as diabetes
mellitus, stroke, renal insufficiency, compensated or prior heart
failure, mild angina, or previous myocardial infarction.
This patient is not undergoing emergency surgery, nor does she
have an active cardiac condition; however, she is undergoing a
high-risk procedure (>5% risk of perioperative myocardial
infarction) with vascular surgery. As she cannot climb a flight of
stairs or do heavy housework, her functional status is <4 METs, and
she should be considered for further evaluation. The patient's
diabetes is an additional clinical risk factor.
With vascular surgery being planned, appropriate
recommendations include proceeding with the surgery with heart
rate control, or performing noninvasive testing if it will change the
management of the patient. Coronary angiography is indicated if
the noninvasive testing is abnormal. Pulmonary function studies
are most useful in patients with underlying lung disease or those
undergoing pulmonary resection. Hemoglobin A1c is a measure of
long-term diabetic control and is not particularly useful
perioperatively. Carotid angiography is not indicated in
asymptomatic patients being considered for lower-extremity
vascular procedures.
32.
=======================================================
Random Board Review Questions 09
=======================================================
A 21-year-old African-American female has been confused and
delirious for 2 days. She has no significant past medical history, and
she is taking no medications. She recently returned from a
missionary trip to Southeast Asia. During your initial examination in
the emergency department, she has several convulsions and rapidly
becomes comatose. Her temperature is 37.9C (100.3F) and her
blood pressure is 80/50 mm Hg. A neurologic examination shows no
signs of meningeal irritation and a cranial nerve evaluation is
normal. There is a mild, bilateral, symmetric increase in deep tendon
reflexes. All other physical examination findings are normal.
Laboratory Findings
Hemoglobin........................... 7.0 g/dL (N 12.0-16.0)
Hematocrit............................ 20% (N 36-46)
WBCs.. . . . . . . . . . . . . . . . . . . . . . . . . . . . . . . 6500/mm3 (N 4300-10,800)
Platelets. ............................. 450,000/mm3 (N 150,000-350,000)
Serum bilirubin
Total............................... 5.0 mg/dL (N 0.3-1.1)
Direct.............................. 1.0 mg/dL (N 0.1-0.4)
B. malaria. Clinical clues to the diagnosis of malaria in this
case include an appropriately targeted recent travel
history, a prodrome of delirium or erratic behavior,
unarousable coma following a generalized convulsion,
fever, and a lack of focal neurologic signs in the presence of
a diffuse, symmetric encephalopathy. The peripheral blood
smear shows normochromic, normocytic anemia with
Plasmodium falciparum trophozoites and schizonts
involving erythrocytes, diagnostic of cerebral malaria.
Treatment of this true medical emergency is intravenous
quinidine gluconate.
Vitamin B 12 deficiency is a predominantly peripheral
neuropathy seen in older adults. Ehrlichiosis causes
thrombocytopenia but not hemolytic anemia. Sickle cell
disease presents with painful vaso-occlusive crises in
multiple organs. Coma is rare.
The urine is dark red and positive for hemoglobin. CT of the brain
shows neither bleeding nor infarction.
The most likely diagnosis is: (check one)
A. vitamin B12 deficiency
B. malaria
C. ehrlichiosis
D. sickle cell anemia
33.
=======================================================
Random Board Review Questions 10
=======================================================
A painful thrombosed external hemorrhoid diagnosed within the
first 24 hours after occurrence is ideally treated by: (check one)
A. appropriate antibiotics
B. office banding
C. office cryotherapy
D. thrombectomy under local anesthesia
E. total hemorrhoidectomy
D. thrombectomy under local anesthesia. A thrombosed
external hemorrhoid is manifested by the sudden
development of a painful, tender, perirectal lump. Because
there is somatic innervation, the pain is intense, and
increases with edema. Treatment involves excision of the
acutely thrombosed tissue under local anesthesia, mild
pain medication, and sitz baths. It is inappropriate to use
procedures that would increase the pain, such as banding
or cryotherapy. Total hemorrhoidectomy is inappropriate
and unnecessary.
34.
=======================================================
Random Board Review Questions 11
=======================================================
A 75-year-old male consults you after his family expresses concern
about his loss of interest in his usual activities. They believe he has
become increasingly withdrawn since the death of his wife 8
months earlier. You note he has lost 8 kg (18 lb) since his last office
visit 6 months earlier. He does not drink alcohol. His physical
examination is unremarkable for his age except for a blood
pressure of 105/70 mm Hg. Detailed laboratory studies, including
thyroid function tests, are all within normal limits. He tells you he
would be fine if he could just get some sleep. His Mini-Mental State
Examination is normal, but he is obviously clinically depressed.
B. mirtazapine (Remeron). Trazodone may be useful for
insomnia, but is not recommended as a primary
antidepressant because it causes sedation and orthostatic
hypotension at therapeutic doses. Bupropion would
aggravate this patient's insomnia. Tricyclic antidepressants
may be effective, but are no longer considered first-line
treatments because of side effects and because they can be
cardiotoxic. Mirtazapine has serotonergic and noradrenergic
properties and is associated with increased appetite and
weight gain. It may be particularly useful for patients with
insomnia and weight loss.
The most appropriate medication for his depression would be:
(check one)
A. trazodone (Oleptro)
B. mirtazapine (Remeron)
C. bupropion (Wellbutrin)
D. amitriptyline
E. nortriptyline (Pamelor)
35.
=======================================================
Random Board Review Questions 12
=======================================================
The most appropriate advice for a 50-year-old female who has
passed six calcium oxalate stones over the past 4 years is to: (check
one)
A. restrict her calcium intake
B. restrict her intake of yellow vegetables
C. increase her sodium intake
D. increase her dietary protein intake
E. take potassium citrate with meals
36.
=======================================================
Random Board Review Questions 13
=======================================================
Which one of the following is a recommended treatment for
presumptive methicillin-resistant Staphylococcus aureus (MRSA)
infection? (check one)
A. Azithromycin (Zithromax)
B. Dicloxacillin
C. Levofloxacin (Levaquin)
D. Doxycycline
E. Cephalexin (Keflex)
E. take potassium citrate with meals. Calcium oxalate stones
are the most common of all renal calculi. A low-sodium,
restricted-protein diet with increased fluid intake reduces
stone formation. A low-calcium diet has been shown to be
ineffective. Oxalate restriction also reduces stone formation.
Oxalate-containing foods include spinach, chocolate, tea, and
nuts, but not yellow vegetables. Potassium citrate should be
taken at mealtime to increase urinary pH and urinary citrate
(SOR B).
D. Doxycycline. Community-acquired methicillin-resistant
Staphylococcus aureus (MRSA) is resistant to -lactam and
macrolide antibiotics, and is showing increasing resistance to
fluoroquinolones. FDA-approved treatments include
clindamycin and doxycycline. Other commonly used
treatments include minocycline and
trimethoprim/sulfamethoxazole.
37.
=======================================================
Random Board Review Questions 14
=======================================================
A 3-year-old toilet-trained female is brought to your office by
her mother, who has noted a red rash on the child's perineum
for the last 5 days. The rash is pruritic and has been spreading.
The mother has treated the area for 3 days with nystatin
cream with no obvious improvement. The child has not used
any other recent medications and has no significant past
medical history. Your examination reveals a homogeneous,
beefy red rash surrounding the vulva and anus.
E. group A Streptococcus pyogenes. The epidemiology of group A
streptococcal disease of the perineum is similar to that of group A
streptococcal pharyngitis, and the two often coexist. It is theorized
that either auto-inoculation from mouth to hand to perineum
occurs, or that the bacteria is transmitted through the
gastrointestinal tract. In one study, the average age of patients with
this disease varied from 1 to 11 years, with a mean of 5 years. Girls
and boys were almost equally affected. The incidence is estimated
to be about 1 in 200 pediatric visits and peaks in March, April, and
May in North America. The condition usually presents with itching
and a beefy redness around the anus and/or vulva and will not
clear with medications used to treat candidal infections.
The most likely etiologic agent is: (check one)
A. Malassezia furfur
B. Escherichia coli
C. Haemophilus influenzae
D. Staphylococcus aureus
E. group A Streptococcus pyogenes
38.
=======================================================
Random Board Review Questions 15
=======================================================
You are helping a hospice program manage the symptoms of
a 77-year-old male with end-stage colon cancer. He has
required increasingly higher doses of his opioid medication
to control symptoms of pain and dyspnea.
A. Constipation. Constipation is one adverse effect of opioid
treatment that does not diminish with time. Thus, this effect should
be anticipated, and recommendations for prevention and
treatment of constipation should be discussed when initiating
opioids. Nausea and vomiting, mental status changes, sedation,
and pruritus are also common with the initiation of opioid
treatment, but these symptoms usually diminish with time, and can
be managed expectantly.
In this situation, it should be kept in mind that which one of
the following adverse effects of opioids does NOT diminish
over time? (check one)
A. Constipation
B. Nausea
C. Mental status changes
D. Pruritus
E. Sedation
39.
=======================================================
Random Board Review Questions 16
=======================================================
Two doses of varicella vaccine are recommended for: (check
one)
A. adults under 60 years of age who develop shingles
B. all children with normal immune status
C. only immunocompromised individuals
D. only children between 12 months and 13 years of age
B. all children with normal immune status. Two doses of varicella
vaccine are recommended for all children unless they are
immunocompromised, in which case they should not be
immunized against varicella, or with other live-virus vaccines.
Shingles is evidence of prior varicella infection and is a reason not
to vaccinate with varicella vaccine.
40.
=======================================================
Random Board Review Questions 17
=======================================================
A 53-year-old female presents to the emergency department
following a fall. She is found to have an ankle fracture and a
blood pressure of 160/100 mm Hg. She tells the emergency
department physician that she is not aware of any previous
medical problems. A focused cardiovascular examination is
otherwise normal. You are the patient's regular physician,
and the emergency physician calls your office for further
information about the blood pressure elevation. You confirm
that this is a new problem.
E. Perform no further evaluation of the hypertension, but ask the
patient to follow up with you within a month. Uncomplicated
hypertension is frequently detected in the emergency
department. Many times this is a chronic condition, but it also may
result from an acutely painful situation. Hypertensive emergencies,
defined as severe blood pressure elevations to >180/120 mm Hg
complicated by evidence of impending or worsening target organ
dysfunction, warrant emergent treatment. There is no evidence,
however, to suggest that treatment of an isolated blood pressure
elevation in the emergency department is linked to a reduction in
overall risk. In fact, the aggressive reduction of blood pressure with
either intravenous or oral agents is not without potential risk.
Which one of the following would you ask the emergency
physician to do? (check one)
The appropriate management for the patient in this scenario is
simply to discharge her and ask her to follow up with you in the
near future.
A. Administer a dose of intravenous labetalol (Trandate) and
ask the patient to follow up in your office within the week
B. Administer nifedipine (Adalat, Procardia), 10 mg; discharge
the patient once the blood pressure falls to 140/90 mm Hg;
and ask the patient to follow up with you tomorrow
C. Prescribe an appropriate antihypertensive agent and have
the patient follow up with you in a month
D. Order an EKG and chest radiograph, and ask the patient to
see you in a week if the results are normal
E. Perform no further evaluation of the hypertension, but ask
the patient to follow up with you within a month
41.
=======================================================
Random Board Review Questions 18
=======================================================
When added to compression therapy, which one of the
following has been shown to be an effective adjunctive
treatment for venous ulcers? (check one)
A. Warfarin (Coumadin)
B. Enoxaparin (Lovenox)
C. Clopidogrel (Plavix)
D. Pentoxifylline (Trental)
E. Atorvastatin (Lipitor)
D. Pentoxifylline (Trental). Pentoxifylline is effective when used with
compression therapy for venous ulcers, and may be useful as
monotherapy in patients unable to tolerate compression therapy.
Aspirin has also been shown to be effective. Other treatments that
have been studied but have not been found to be effective include
oral zinc and antibiotics (SOR A).
42.
=======================================================
Random Board Review Questions 19
=======================================================
A 55-year-old male sees you for a follow-up visit for
hypercholesterolemia and hypertension. He is in good
health, does not smoke, and drinks alcohol infrequently. His
medications include a multiple vitamin daily; aspirin, 81 mg
daily; lisinopril (Prinivil, Zestril), 10 mg daily; and lovastatin
(Mevacor), 20 mg daily. His vital signs are within normal limits
except for a BMI of 33.4 kg/m2 .
At today's visit his ALT (SGPT) level is 55 IU/L (N 10-45) and his
AST (SGOT) level is 44 IU/L (N 10-37). The remainder of the liver
panel is normal.
E. Metabolic syndrome. Non-alcoholic fatty liver disease (NAFLD) is
the most common cause of abnormal liver tests in the developed
world. Its prevalence increases with age, body mass index, and
triglyceride concentrations, and in patients with diabetes mellitus,
hypertension, or insulin resistance. There is a significant overlap
between metabolic syndrome and diabetes mellitus, and NAFLD is
regarded as the liver manifestation of insulin resistance.
Statin therapy is considered safe in such individuals and can
improve liver enzyme levels and reduce cardiovascular morbidity
in patients with mild to moderately abnormal liver tests that are
potentially attributable to NAFLD.
Which one of the following is the most likely cause of the
elevation in liver enzymes? (check one)
A. A side effect of lovastatin
B. Gallbladder disease
C. Hepatitis A
D. Alcoholic liver disease
E. Metabolic syndrome
43.
=======================================================
Random Board Review Questions 20
=======================================================
An 82-year-old white male suffers from chronic low back
pain. He is on warfarin (Coumadin) for chronic atrial
fibrillation, tamsulosin (Flomax) for benign prostatic
hyperplasia, and famotidine (Pepcid) for gastroesophageal
reflux disease.
A. The lidocaine patch (Lidoderm). Topical lidocaine produces very
low serum levels of active drug, resulting in very few adverse
effects (SOR C). Hydrocodone could produce any opiate-type
effect. Nortriptyline and duloxetine could aggravate this patient's
atrial arrhythmia and cause urinary retention. Celecoxib could
aggravate his reflux problem.
Which one of the following analgesic medications would
have the least potential for adverse side effects? (check one)
A. The lidocaine patch (Lidoderm)
B. Hydrocodone/acetaminophen
C. Nortriptyline (Pamelor)
D. Duloxetine (Cymbalta)
E. Celecoxib (Celebrex)
44.
=======================================================
Random Board Review Questions 21
=======================================================
Which one of the following treatments for type 2 diabetes
mellitus often produces significant weight loss? (check one)
A. Exenatide (Byetta)
B. Glipizide (Glucotrol)
C. Pioglitazone (Actos)
D. Insulin detemir (Levemir)
E. Insulin lispro (Humalog)
A. Exenatide (Byetta). Of the many currently available medications
to treat diabetes mellitus, only metformin and incretin mimetics
such as exenatide have the additional benefit of helping the
overweight or obese patient lose a significant amount of weight.
Most of the other medications, including all the insulin formulations,
unfortunately lead to weight gain or have no effect on weight.
45.
=======================================================
Random Board Review Questions 22
=======================================================
Actinic keratosis is a precursor lesion to: (check one)
A. keratoacanthoma
B. nodular melanoma
C. superficial spreading melanoma
D. basal cell carcinoma
E. cutaneous squamous cell carcinoma
46.
=======================================================
Random Board Review Questions 23
=======================================================
A 56-year-old white male reports lower leg claudication that
occurs when he walks approximately one block, and is relieved
by standing still or sitting. He has a history of diabetes mellitus
and hyperlipidemia. His most recent hemoglobin A 1c level was
5.9% and his LDL-cholesterol level at that time was 95 mg/dL.
Current medications include glyburide (DiaBeta), metformin
(Glucophage), simvastatin (Zocor), and daily aspirin. He stopped
smoking 1 month ago and began a walking program. A physical
examination is normal, except for barely palpable dorsalis pedis
and posterior tibial pulses. Femoral and popliteal pulses are
normal. Noninvasive vascular studies of his legs show an anklebrachial index of 0.7 bilaterally, and decreased flow.
E. cutaneous squamous cell carcinoma. Actinic keratoses are
precursor lesions for cutaneous squamous cell carcinoma. The
conversion rate of actinic keratoses into squamous cell
carcinoma has been estimated to be 1 in 1000 per year. Thicker
lesions, cutaneous horns, and lesions that show ulceration have
a higher malignant potential. Although sun exposure is a risk
factor for both melanoma and basal cell carcinoma, there are no
recognized precursor lesions for either. Actinic keratosis is not a
precursor lesion to keratoacanthoma.
C. Cilostazol (Pletal). The patient described has symptomatic
arterial vascular disease manifested by intermittent
claudication. He has already initiated the two most important
changes: he has stopped smoking and started a walking
program. His LDL-cholesterol is at target levels; further lowering
is not likely to improve his symptoms. In the presence of diffuse
disease, interventional treatments such as angioplasty or
surgery may not be helpful; in addition, these interventions
should be reserved as a last resort. Cilostazol has been shown
to help with intermittent claudication, but additional antiplatelet
agents are not likely to improve his symptoms. Fish oil and
warfarin have not been found to be helpful in the
management of this condition.
Which one of the following would be most appropriate for
addressing this patient's symptoms? (check one)
A. Fish oil
B. Warfarin (Coumadin)
C. Cilostazol (Pletal)
D. Dipyridamole (Persantine)
E. Clopidogrel (Plavix)
47.
=======================================================
Random Board Review Questions 24
=======================================================
Which one of the following organisms is NOT killed by alcoholbased hand disinfectants? (check one)
A. Methicillin-resistant Staphylococcus aureus (MRSA)
B. Methicillin-sensitive Staphylococcus aureus
C. Pseudomonas aeruginosa
D. Klebsiella pneumoniae
E. Clostridium difficile
E. Clostridium difficile. Sporulating organisms such as
Clostridium difficile are not killed by alcohol products.
Staphylococcus aureus, Pseudomonas aeruginosa, and
Klebsiella pneumoniae are killed by alcohol products (SOR A).
48.
=======================================================
Random Board Review Questions 25
=======================================================
A 46-year-old female presents to your office with a 2-week
history of pain in her left shoulder. She does not recall any
injury, and the pain is present when she is resting and at
night. Her only chronic medical problem is type 2 diabetes
mellitus.
On examination, she has limited movement of the shoulder
and almost complete loss of external rotation. Radiographs
of the shoulder are normal, as is her erythrocyte
sedimentation rate.
Which one of the following is the most likely diagnosis?
(check one)
A. Frozen shoulder. Frozen shoulder is an idiopathic condition that
most commonly affects patients between the ages of 40 and 60.
Diabetes mellitus is the most common risk factor for frozen
shoulder. Symptoms include shoulder stiffness, loss of active and
passive shoulder rotation, and severe pain, including night pain.
Laboratory tests and plain films are normal; the diagnosis is clinical
(SOR C).
Frozen shoulder is differentiated from chronic posterior shoulder
dislocation and osteoarthritis on the basis of radiologic findings.
Both shoulder dislocation and osteoarthritis have characteristic
plain film findings. A patient with a rotator cuff tear will have
normal passive range of motion. Impingement syndrome does not
affect passive range of motion, but there will be pain with elevation
of the shoulder.
A. Frozen shoulder
B. Torn rotator cuff
C. Impingement syndrome
D. Chronic posterior shoulder dislocation
E. Osteoarthritis
49.
=======================================================
Random Board Review Questions 26
=======================================================
Which one of the following is an appropriate rationale for
antibiotic treatment of Bordetella pertussis infections?
(check one)
A. It delays progression from the catarrhal stage to the
paroxysmal stage
B. It reduces the severity of symptoms
C. It reduces the duration of illness
D. It reduces the risk of transmission to others
E. It reduces the need for hospitalization
D. It reduces the risk of transmission to others. Antibiotic treatment
for pertussis is effective for eradicating bacterial infection but not
for reducing the duration or severity of the disease. The
eradication of infection is important for disease control because it
reduces infectivity. Antibiotic treatment is thought to be most
effective if started early in the course of the illness, characterized as
the catarrhal phase. The paroxysmal stage follows the catarrhal
phase. The CDC recommends macrolides for primary treatment of
pertussis. The preferred antimicrobial regimen is azithromycin for
3-5 days or clarithromycin for 7 days. These regimens are as
effective as longer therapy with erythromycin and have fewer side
effects. Children under 1 month of age should be treated with
azithromycin. There is an association between erythromycin and
hypertrophic pyloric stenosis in young infants.
Trimethoprim/sulfamethoxazole can be used in patients who are
unable to take macrolides or where macrolide resistance may be an
issue, but should not be used in children under the age of 2
months. Fluoroquinolones have been shown to reduce pertussis in
vitro but have not been shown to be
clinically effective (SOR A).
50.
=======================================================
Random Board Review Questions 27
=======================================================
The preferred method for diagnosing psychogenic
nonepileptic seizures is: (check one)
A. inducing seizures by suggestion
B. postictal prolactin levels
C. EEG monitoring
D. video-electroencephalography (vEEG) monitoring
E. brain MRI
51.
=======================================================
Random Board Review Questions 28
=======================================================
A 72-year-old male has had persistent interscapular pain with
movement since rebuilding his deck 1 week ago. He rates the
pain as 6 on a 10-point scale. A chest radiograph shows a
thoracic vertebral compression fracture.
Which one of the following would be most appropriate at
this point?
(check one)
A. Complete bed rest for 2 weeks
B. Markedly decreased activity until the pain lessens, and
follow-up in 1 week
C. Referral for vertebroplasty as soon as possible
D. NSAIDs and referral for physical therapy
D. video-electroencephalography (vEEG) monitoring. Inpatient
video-electroencephalography (vEEG) monitoring is the preferred
test for the diagnosis of psychogenic nonepileptic seizures (PNES),
and is considered the gold standard (SOR B). Video-EEG monitoring
combines extended EEG monitoring with time-locked video
acquisition that allows for analysis of clinical and electrographic
features during a captured event. Many other types of evidence
have been used, including the presence or absence of self-injury
and incontinence, the ability to induce seizures by suggestion,
psychologic tests, and ambulatory EEG. While useful in some cases,
these alternatives have been found to be insufficient for the
diagnosis of PNES.
Elevated postictal prolactin levels (at least two times the upper limit
of normal) have been used to differentiate generalized and
complex partial seizures from PNES, but are not reliable (SOR B).
While prolactin levels are often elevated after an epileptic seizure,
they do not always rise, and the timing of measurement is crucial,
making this a less sensitive test than was previously believed. Other
serum markers have also been used to help distinguish PNES from
epileptic seizures, including creatine phosphokinase, cortisol, WBC
counts, lactate dehydrogenase, pCO2 , and neuron-specific
enolase. These also are not reliable, as threshold levels for
abnormality, sensitivity, and specificity have not been determined.
MRI is not reliable because abnormal brain MRIs have been
documented in as many as one-third of patients with PNES. In
addition, patients with epileptic seizures often have normal brain
MRIs.
B. Markedly decreased activity until the pain lessens, and follow-up
in 1 week. This patient has suffered a thoracic vertebral
compression fracture. Most can be managed conservatively with
decreased activity until the pain is tolerable, possibly followed by
some bracing. Vertebroplasty is an option when the pain is not
improved in 2 weeks. Complete bed rest is unnecessary and could
lead to complications. Physical therapy is not indicated, and NSAIDs
should be used with caution.
52.
=======================================================
Random Board Review Questions 29
=======================================================
A 70-year-old male presents to your office for a follow-up visit
for hypertension. He was started on lisinopril (Prinivil,
Zestril), 20 mg daily, 1 month ago. Laboratory tests from his
last visit, including a CBC and a complete metabolic panel,
were normal except for a serum creatinine level of 1.5 mg/dL
(N 0.6-1.5). A follow-up renal panel obtained yesterday shows
a creatinine level of 3.2 mg/dL and a BUN of 34 mg/dL (N 8-25).
Which one of the following is the most likely cause of this
patient's increased creatinine level? (check one)
A. Bilateral renal artery stenosis. Classic clinical clues that suggest a
diagnosis of renal-artery stenosis include the onset of stage 2
hypertension (blood pressure >160/100 mm Hg) after 50 years of
age or in the absence of a family history of hypertension;
hypertension associated with renal insufficiency, especially if renal
function worsens after the administration of an agent that blocks
the renin-angiotensin-aldosterone system; hypertension with
repeated hospital admissions for heart failure; and drug-resistant
hypertension (defined as blood pressure above the goal despite
treatment with three drugs of different classes at optimal doses).
The other conditions mentioned do not cause a significant rise in
serum creatinine after treatment with an ACE inhibitor.
A. Bilateral renal artery stenosis
B. Coarctation of the aorta
C. Essential hypertension
D. Hyperaldosteronism
E. Pheochromocytoma
53.
=======================================================
Random Board Review Questions 30
=======================================================
A 53-year-old male presents to your office with a several-day
history of hiccups. They are not severe, but have been
interrupting his sleep, and he is becoming exasperated.
What should be the primary focus of treatment in this
individual?
(check one)
A. Drug treatment to prevent recurrent episodes
B. Decreasing the intensity of the muscle contractions in the
diaphragm
C. Finding the underlying pathology causing the hiccups
D. Improving the patient's quality of sleep
E. Suppressing the current hiccup symptoms
54.
=======================================================
Random Board Review Questions 31
=======================================================
A patient who takes fluoxetine (Prozac), 40 mg twice daily,
develops shivering, tremors, and diarrhea after taking an
over-the-counter cough and cold medication. On
examination he has dilated pupils and a heart rate of 110
beats/min. His temperature is normal.
Which one of the following medications in combination with
fluoxetine could contribute to this patient's symptoms?
(check one)
A. Dextromethorphan
B. Pseudoephedrine
C. Phenylephrine
D. Guaifenesin
E. Diphenhydramine (Benadryl)
C. Finding the underlying pathology causing the hiccups. Hiccups
are caused by a respiratory reflex that originates from the phrenic
and vagus nerves, as well as the thoracic sympathetic chain.
Hiccups that last a matter of hours are usually benign and selflimited, and may be caused by gastric distention. Treatments
usually focus on interrupting the reflex loop of the hiccup, and can
include mechanical means (e.g., stimulating the pharynx with a
tongue depressor) or medical treatment, although only
chlorpromazine is FDA-approved for this indication.
If the hiccups have lasted more than a couple of days, and
especially if they are waking the patient up at night, there may be
an underlying pathology causing the hiccups. In one study, 66% of
patients who experienced hiccups for longer than 2 days had an
underlying physical cause. Identifying and treating the underlying
disorder should be the focus of management for intractable
hiccups.
A. Dextromethorphan. Dextromethorphan is commonly found in
cough and cold remedies, and is associated with serotonin
syndrome. SSRIs such as fluoxetine are also associated with
serotonin syndrome, and there are many other medications that
increase the risk for serotonin syndrome when combined with
SSRIs. The other medications listed here are not associated with
serotonin syndrome, however.
55.
=======================================================
Random Board Review Questions 32
=======================================================
While playing tennis, a 55-year-old male tripped and fell,
landing on his outstretched hand with his elbow in slight
flexion at impact. Pronation and supination of the forearm
are painful on examination, as are attempts to flex the elbow.
There is tenderness of the radial head without significant
swelling. A radiograph of the elbow shows no fracture, but a
positive fat pad sign is noted.
Appropriate management would include: (check one)
D. a posterior splint and a repeat radiograph in 1-2 weeks.
Nondisplaced radial head fractures can be treated by the primary
care physician and do not require referral. Conservative therapy
includes placing the elbow in a posterior splint for 5-7 days,
followed by early mobilization and a sling for comfort. Sometimes
the joint effusion may be aspirated for pain relief and to increase
mobility. One study compared immediate mobilization with
mobilization beginning in 5 days and found no differences at 1 and
3 months, but early mobilization was associated with better
function and less pain 1 week after the injury. Radiographs should
be repeated in 1-2 weeks to make sure that alignment is
appropriate.
A. a long arm cast for 2 weeks, followed by use of a brace
B. mobilization of the elbow beginning 3 weeks after the
injury
C. a posterior splint for 6 weeks
D. a posterior splint and a repeat radiograph in 1-2 weeks
56.
=======================================================
Random Board Review Questions 33
=======================================================
A 48-year-old female with type 2 diabetes has been
hospitalized for 4 days with persistent fever. Her diabetes has
been controlled with diet and glyburide (Micronase,
DiaBeta). You saw her 2 weeks ago in the office with urinary
frequency, urgency, and dysuria. At that time a urinalysis
showed 25 WBCs/hpf, and a urine culture subsequently grew
Escherichia coli sensitive to all antibiotics. She was placed on
trimethoprim/sulfamethoxazole (Bactrim, Septra)
empirically, and this was continued after the culture results
were reported.
She improved over the next week, but then developed flank
pain, fever to 39.5C (103.1F), and nausea and vomiting. She
was hospitalized and intravenous cefazolin (Kefzol) and
gentamicin were started while blood and urine cultures were
performed. This urine culture also grew E. coli sensitive to the
current antibiotics. Her temperature has continued to spike
to 39.5C since admission, without any change in her
symptoms.
Which one of the following would be most appropriate at
this time? (check one)
E. Order CT of the abdomen. Perinephric abscess is an elusive
diagnostic problem that is defined as a collection of pus in the tissue
surrounding the kidney, generally in the space enclosed by
Gerota's fascia. Mortality rates as high as 50% have been reported,
usually from failure to diagnose the problem in a timely fashion.
The difficulty in making the diagnosis can be attributed to the
variable constellation of symptoms and the sometimes indolent
course of this disease. The diagnosis should be considered when a
patient has fever and persistence of flank pain.
A. Add vancomycin (Vancocin) to the regimen
B. Order a radionuclide renal scan
C. Order intravenous pyelography
D. Order a urine culture for tuberculosis
E. Order CT of the abdomen
Drainage, either percutaneously or surgically, along with
appropriate antibiotic coverage reduces both morbidity and
mortality from this condition.
Most perinephric infections occur as an extension of an ascending
urinary tract infection, commonly in association with renal calculi or
urinary tract obstruction. Patients with anatomic urinary tract
abnormalities or diabetes mellitus have an increased risk. Clinical
features may be quite variable, and the most useful predictive
factor in distinguishing uncomplicated pyelonephritis from
perinephric abscess is persistence of fever for more than 4 days
after initiation of antibiotic therapy. The radiologic study of choice is
CT. This can detect perirenal fluid, enlargement of the psoas muscle
(both are highly suggestive of the diagnosis), and perirenal gas
(which is diagnostic). The sensitivity and specificity of CT is
significantly greater than that of either ultrasonography or
intravenous pyelography.
57.
=======================================================
Random Board Review Questions 34
=======================================================
Which one of the following should be used first for
ventricular fibrillation when an initial defibrillation attempt
fails? (check one)
A. Amiodarone (Cordarone)
B. Lidocaine (Xylocaine)
C. Adenosine (Adenocard)
D. Vasopressin (Pitressin)
E. Magnesium
58.
=======================================================
Random Board Review Questions 35
=======================================================
A 14-year-old female with a history of asthma is having
daytime symptoms about once a week and symptoms that
awaken her at night about once a month. Her asthma does
not interfere with normal activity, and her FEV1 is >80% of
predicted.
Which one of the following is the most appropriate
treatment plan for this patient?
(check one)
D. Vasopressin (Pitressin). For persistent ventricular fibrillation (VF),
in addition to electrical defibrillation and CPR, patients should be
given a vasopressor, which can be either epinephrine or
vasopressin. Vasopressin may be substituted for the first or second
dose of epinephrine.
Amiodarone should be considered for treatment of VF
unresponsive to shock delivery, CPR, and a vasopressor. Lidocaine
is an alternative antiarrhythmic agent, but should be used only
when amiodarone is not available. Magnesium may terminate or
prevent torsades de pointes in patients who have a prolonged QT
interval during normal sinus rhythm. Adenosine is used for the
treatment of narrow complex, regular tachycardias and is not used
in the treatment of ventricular fibrillation.
A. A short-acting inhaled -agonist as needed. Based on this
patient's reported frequency of asthma symptoms, she should be
classified as having intermittent asthma. The preferred first step in
managing intermittent asthma is an inhaled short-acting -agonist
as needed. Daily medication is reserved for patients with persistent
asthma (symptoms >2 days per week for mild, daily for moderate,
and throughout the day for severe) and is initiated in a stepwise
approach, starting with a daily low-dose inhaled corticosteroid or
leukotriene receptor antagonist and then progressing to a
medium-dose inhaled corticosteroid or low-dose inhaled
corticosteroid plus a long-acting inhaled -agonist.
A. A short-acting inhaled -agonist as needed
B. Low-dose inhaled corticosteroids daily
C. A leukotriene receptor antagonist daily
D. Medium-dose inhaled corticosteroids daily
E. Low-dose inhaled corticosteroids plus a long-acting
inhaled -agonist daily
59.
=======================================================
Random Board Review Questions 36
=======================================================
A 91-year-old white male presents with a 6-month history of a
painless ulcer on the dorsum of the proximal interphalangeal
joint of the second toe. Examination reveals a hallux valgus
and a rigid hammer toe of the second digit. His foot has mild
to moderate atrophic skin changes, and the dorsal and
posterior tibial pulses are absent.
Appropriate treatment includes which one of the following?
(check one)
A. Surgical correction of the hammer toe
B. Custom-made shoes to protect the hammer toe
C. Bunionectomy
D. A metatarsal pad
B. Custom-made shoes to protect the hammer toe. The treatment of
foot problems in the elderly is difficult because of systemic and local
infirmities, the most limiting being the poor vascular status of the
foot. Conservative, supportive, and palliative therapy replace
definitive reconstructive surgical therapy. Surgical correction of a
hammer toe and bunionectomy could be disastrous in an elderly
patient with a small ulcer and peripheral vascular disease. The best
approach with this patient is to prescribe custom-made shoes and
a protective shield with a central aperture of foam rubber placed
over the hammer toe. Metatarsal pads are not useful in the
treatment of hallux valgus and a rigid hammer toe.
60.
=======================================================
Random Board Review Questions 37
=======================================================
A patient presents with a pigmented skin lesion that could be a
melanoma. Its largest dimension is 0.5 cm.
B. Excision with a 1-mm margin. The diagnosis of melanoma
should be made by simple excision with clear margins. A shave
biopsy should be avoided because determining the thickness
of the lesion is critical for staging. Wide excision with or
without node dissection is indicated for confirmed melanoma,
depending on the findings from the initial excisional biopsy.
What should be the first step in management? (check one)
A. A shave biopsy
B. Excision with a 1-mm margin
C. Wide excision with a 1-cm margin
D. Wide excision with a 1-cm margin
E. Excision with sentinel node dissection
61.
=======================================================
Random Board Review Questions 38
=======================================================
A 60-year-old female receiving home hospice care was taking oral
morphine, 15 mg every 2 hours, to control pain. When this was no
longer effective, she was transferred to an inpatient facility for
pain control. She required 105 mg of morphine in a 24-hour
period, so she was started on intravenous morphine, 2 mg/hr with
a bolus of 2 mg, and was well controlled for 5 days. However, her
pain has worsened over the past 2 days.
E. Tolerance to morphine. This patient has become tolerant to
morphine. The intravenous dose should be a third of the oral
dose, so the starting intravenous dose was adequate.
Addiction is compulsive narcotic use. Pseudoaddiction is
inadequate narcotic dosing that mimics addiction because of
unrelieved pain. Physical dependence is seen with abrupt
narcotic withdrawal.
Which one of the following is the most likely cause of this
patient's increased pain? (check one)
A. An inadequate initial morphine dose
B. Addiction to morphine
C. Pseudoaddiction to morphine
D. Physical dependence on morphine
E. Tolerance to morphine
62.
=======================================================
Random Board Review Questions 39
=======================================================
A 40-year-old white male presents with a 5-year history of periodic
episodes of severe right-sided headaches. During the most recent
episode the headaches occurred most days during January and
February and lasted about 1 hour.
The most likely diagnosis is which one of the following? (check
one)
A. Migraine headache
B. Cluster headache
C. Temporal arteritis
D. Trigeminal neuralgia
B. Cluster headache. Cluster headache is predominantly a
male disorder. The mean age of onset is 27-30 years. Attacks
often occur in cycles and are unilateral. Migraine headaches
are more common in women, start at an earlier age (second or
third decade), and last longer (4-24 hours). Temporal arteritis
occurs in patients above age 50. Trigeminal neuralgia usually
occurs in paroxysms lasting 20-30 seconds.
63.
=======================================================
Random Board Review Questions 40
=======================================================
Which one of the following is true regarding hospice? (check
one)
A. Hospice benefits end if the patient lives beyond the
estimated 6-month life expectancy
B. A do-not-resuscitate (DNR) order is required for a patient
receiving Medicare hospice benefits
C. Patients in hospice cannot receive chemotherapy, blood
transfusions, or radiation treatments
D. Patients must be referred to hospice by their physician
E. Any terminal patient with a life expectancy <6 months is
eligible
64.
=======================================================
Random Board Review Questions 41
=======================================================
A 62-year-old African-American female undergoes a workup
for pruritus. Laboratory findings include a hematocrit of
55.0% (N 36.0-46.0) and a hemoglobin level of 18.5 g/dL (N
12.0-16.0).
Which one of the following additional findings would help
establish the diagnosis of polycythemia vera? (check one)
A. A platelet count >400,000/mm3
B. An O2 saturation <90%
C. A WBC count <4500/mm (N 4300-10,800)3
D. An elevated uric acid level
65.
=======================================================
Random Board Review Questions 42
=======================================================
Which one of the following is NOT considered a first-line
treatment for head lice? (check one)
A. Lindane 1%
B. Malathion 0.5% (Ovide)
C. Permethrin 1% (Nix)
D. Pyrethrins 0.33%/pipernyl butoxide 4% (RID)
66.
=======================================================
Random Board Review Questions 43
=======================================================
Which one of the following insulin regimens most closely
mimics the normal pattern of pancreatic insulin release in a
nondiabetic person? (check one)
A. 70/30 NPH/regular insulin (Humulin 70/30) twice daily
B. NPH insulin twice daily plus an insulin sliding-scale
protocol using regular insulin
C. Insulin glargine (Lantus) daily plus an insulin sliding-scale
protocol using regular insulin
D. Insulin detemir (Levamir) daily plus rapid-acting insulin
with meals
E. Rapid-acting insulin before each meal
E. Any terminal patient with a life expectancy <6 months is eligible.
Any patient with a life expectancy of less than 6 months who
chooses a palliative care approach is an appropriate candidate for
hospice. There is no penalty if patients do not die within 6 months,
as long as the disease is allowed to run its natural course. Medicare
does not require a DNR order to enroll in hospice, but it does
require that patients seek only palliative, not curative, treatment.
Patients may receive chemotherapy, blood transfusions, or
radiation if the goal of the treatment is to provide symptom relief.
Patients can be referred to hospice by anyone, including nurses,
social workers, family members, or friends.
A. A platelet count >400,000/mm3. Polycythemia vera should be
suspected in African-Americans or white females whose
hemoglobin level is >16 g/dL or whose hematocrit is >47%. For white
males, the thresholds are 18 g/dL and 52%. It should also be
suspected in patients with portal vein thrombosis and
splenomegaly, with or without thrombocytosis and leukocytosis.
Major criteria include an increased red cell mass, a normal O2
saturation,and the presence ofsplenomegaly. Minor criteria
includeelevated vitamin B 12 levels, elevated leukocyte alkaline
phosphatase, a platelet count >400,000/mm3 and a WBC count
>12,000/mm3 . Patients with polycythemia vera may present with
gout and an elevated uric acid level, but neither is considered a
criterion for the diagnosis.
A. Lindane 1%. Lindane's efficacy has waned over the years and it
is inconsistently ovicidal. Because of its neurotoxicity, lindane
carries a black box warning and is specifically recommended only
as second-line treatment by the FDA. Pyrethroid resistance is
widespread, but permethrin is still considered to be a first-line
treatment because of its favorable safety profile. The efficacy of
malathion is attributed to its triple action with isopropyl alcohol and
terpineol, likely making this a resistance-breaking formulation. The
probability of simultaneously developing resistance to all three
substances is small. Malathion is both ovicidal and pediculicidal.
D. Insulin detemir (Levamir) daily plus rapid-acting insulin with
meals. Basal insulin provides a relatively constant level of insulin for
24 hours, with an onset of action in 1 hour and no peak. NPH gives
approximately 12 hours of coverage with a peak around 6-8 hours.
Regular insulin has an onset of action of about 30 minutes and lasts
about 5-8 hours, with a peak at about 2-4 hours. New rapid-acting
analogue insulins have an onset of action within 5-15 minutes, peak
within 30-75 minutes, and last only about 2-3 hours after
administration. Thus, a 70/30 insulin mix (typically 70% NPH and
30% regular) provides coverage for 12 hours, but the peaks of
insulin release do not closely mimic natural patterns. NPH given
twice daily along with an insulin sliding-scale protocol using regular
insulin is only slightly closer than a 70/30 twice-daily regimen. Rapid
insulin alone does not provide any basal insulin, and the patient
would therefore not have insulin available during the night.
67.
=======================================================
Random Board Review Questions 44
=======================================================
A 34-year-old female with a history of bilateral tubal ligation
consults you because of excessive body and facial hair. She
has a normal body weight, no other signs of virilization, and
regular menses.
Which one of the following is the most appropriate
treatment for her mild hirsutism? (check one)
A. Spironolactone (Aldactone). Antiandrogens such as
spironolactone, along with oral contraceptives, are recommended
for treatment of hirsutism in premenopausal women (SOR C). In
addition to having side effects, prednisone is only minimally helpful
for reducing hirsutism by suppressing adrenal androgens.
Leuprolide, although better than placebo, has many side effects
and is expensive. Metformin can be used to treat patients with
polycystic ovarian syndrome, but this patient does not meet the
criteria for this diagnosis.
A. Spironolactone (Aldactone)
B. Leuprolide (Lupron)
C. Prednisone
D. Metformin (Glucophage)
68.
=======================================================
Random Board Review Questions 45
=======================================================
A 26-month-old child presents with a 2-day history of 6-8
loose stools per day and a low-grade fever. When evaluating
the child to determine whether he is dehydrated, which one
of the following would NOT be useful? (check one)
A. Skin turgor
B. Capillary refill time
C. Respiratory rate and pattern
D. The BUN/creatinine ratio
E. The serum bicarbonate level
69.
=======================================================
Random Board Review Questions 46
=======================================================
For a healthy 1-month-old, daily vitamin D intake should be:
(check one)
A. 50 IU
B. 100 IU
C. 200 IU
D. 400 IU
E. 800 IU
D. The BUN/creatinine ratio. The most useful findings for
identifying dehydration are prolonged capillary refill time,
abnormal skin turgor, and abnormal respiratory pattern (SOR C).
Capillary refill time is not affected by fever and should be less than
2 seconds. Skin recoil is normally instantaneous, but recoil time
increases linearly with the degree of dehydration. The respiratory
pattern should be compared with age-specific normal values, but
will be increased and sometimes labored, depending on the
degree of dehydration.
Unlike in adults, calculation of the BUN/creatinine ratio is not useful
in children. Although the normal BUN level is the same for children
and adults, the normal serum creatinine level changes with age in
children. In combination with other clinical indicators, a low serum
bicarbonate level (<17 mmol/L) is helpful in identifying children
who are dehydrated, and a level <13 mmol/L is associated with an
increased risk of failure of outpatient rehydration efforts.
D. 400 IU. It is now recommended that all infants and children,
including adolescents, have a minimum daily intake of 400 IU of
vitamin D, beginning soon after birth. The current
recommendation replaces the previous recommendation of a
minimum daily intake of 200 IU/day of vitamin D supplementation
beginning in the first 2 months after birth and continuing through
adolescence. These revised guidelines for vitamin D intake for
healthy infants, children, and adolescents are based on evidence
from new clinical trials and the historical precedent of safely giving
400 IU of vitamin D per day in the pediatric and adolescent
population. New evidence supports a potential role for vitamin D in
maintaining innate immunity and
preventing diseases such as diabetes mellitus and cancer.
70.
=======================================================
Random Board Review Questions 47
=======================================================
The mother of a 16-year-old male calls to report that her son
has a severe sore throat and has been running a fever of 102F.
Which one of the following additional findings would be
most specific for peritonsillar abscess? (check one)
C. Difficulty opening his mouth. Trismus is almost universally
present with peritonsillar abscess, while voice changes, otalgia, and
odynophagia may or may not be present. Pharyngotonsillitis and
peritonsillar cellulitis may also be associated with these complaints.
Otalgia is common with peritonsillar abscess, otitis media,
temporomandibular joint disorders, and a variety of other
conditions. Peritonsillar abscess is rarely found in patients who do
not have at least a 3-day history of progressive sore throat.
A. A 1-day duration of illness
B. Ear pain
C. Difficulty opening his mouth
D. Hoarseness
E. Pain with swallowing
71.
=======================================================
Random Board Review Questions 48
=======================================================
A 60-year-old Chinese female asks you about being tested for
osteoporosis. She is postmenopausal and has never used
hormone therapy. She does not consume dairy products
because she has lactose intolerance. She is on no
medications, is otherwise healthy, and has no history of falls
or fractures. Her mother had osteoporosis and vertebral
compression fractures. Her BMI is 20 kg/m2 .
A. A central DXA scan of the lumbar spine and hips. This patient
has several risk factors for osteoporosis: Asian ethnicity, low body
weight, positive family history, postmenopausal status with no
history of hormone replacement, and low calcium intake. The best
diagnostic test for osteoporosis is a central DXA scan of the hip,
femoral neck, and lumbar spine. Quantitative CT is accurate, but
cost and radiation exposure are issues. Peripheral DXA and
calcaneal sonography results do not correlate well with central DXA.
Measurement of biochemical markers is not recommended for the
diagnosis of osteoporosis.
Which one of the following tests would be best to determine
whether this patient has osteoporosis? (check one)
A. A central DXA scan of the lumbar spine and hips
B. A forearm DXA scan
C. Quantitative CT of the lumbar spine
D. Quantitative calcaneal ultrasonography
E. Measurement of biochemical markers of bone turnover in
the urine
72.
=======================================================
Random Board Review Questions 49
=======================================================
Which one of the following drugs inhibits platelet function
for the life of the platelet? (check one)
A. Aspirin
B. Ibuprofen
C. Dipyridamole (Persantine)
D. Ticlopidine (Ticlid)
E. Warfarin (Coumadin)
A. Aspirin. A number of drugs inhibit platelet function, but aspirin
is the only effective drug that interferes with platelet aggregation
for the life of the platelet. It does this by permanently acetylating
the platelet enzyme cyclooxygenase, thus inhibiting prostaglandin
synthesis. This phenomenon is clinically helpful when an
antithrombotic effect is desired, but it may require that necessary
surgical procedures be delayed. The effect of a single aspirin on
bleeding times can persist for up to 5 days. Other NSAIDs (i.e.,
indomethacin, sulfinpyrazone) also inhibit platelet activity, but their
effect on prostaglandin synthesis is reversible. The anti-platelet
effect of dipyridamole is less well understood. Warfarin is a
biochemical antagonist of prothrombin and vitamin K-dependent
coagulation factors, and therefore has no significant effect on
platelet activity.
73.
=======================================================
Random Board Review Questions 50
=======================================================
A 25-year-old female is concerned about recurrent
psychological and physical symptoms that occur during the
luteal phase of her menstrual cycle and resolve by the end of
menstruation. She wants help managing these symptoms,
but does not want to take additional estrogen or
progesterone.
B. Spironolactone during the luteal phase. Randomized, controlled
trials found that luteal-phase spironolactone improved
psychological and physical symptoms of premenstrual syndrome
over 2-6 months compared with placebo. Based on existing
evidence, the effectiveness is unknown for cognitive-behavioral
therapy, bright light therapy, evening primrose oil, and black
cohosh.
Which one of the following management strategies is
supported by the best clinical evidence?
(check one)
A. Cognitive-behavioral therapy
B. Spironolactone during the luteal phase
C. Bright light therapy during the luteal phase
D. Evening primrose oil started 2-4 days prior to the luteal
phase
E. Black cohosh
74.
=======================================================
Random Board Review Questions 51
=======================================================
Which one of the following is most associated with falls in
older adults? (check one)
A. Diphenhydramine (Benadryl)
B. Atorvastatin (Lipitor)
C. Metformin (Glucophage)
D. Memantine (Namenda)
E. Theophylline (Theo-24)
75.
=======================================================
Random Board Review Questions 52
=======================================================
A 20-year-old nonsmoker presents to your office with a
sudden onset of chest pain. You order a chest radiograph,
which shows a small (<15%) pneumothorax. He is in no
respiratory distress and vital signs are normal.
Pulse oximetry shows a saturation of 98% on room air. which
one of the following would be most appropriate initially?
(check one)
A. CT of the affected lung
B. Analgesics and a follow-up visit in 48 hours
C. Chest tube insertion
D. Hospital admission and a repeat chest film in 24 hours
A. Diphenhydramine (Benadryl). Certain classes of medications are
frequently associated with falls in older adults. These classes
include benzodiazepines, antidepressants, antipsychotics,
antiepileptics, anticholinergics, sedative hypnotics, muscle
relaxants, and cardiovascular medications. Diphenhydramine is
one of the anticholinergic medications associated with falls in older
adults. The other drugs listed are not in the higher-risk groups of
medications.
B. Analgesics and a follow-up visit in 48 hours. Practice guidelines
state that a patient without apparent lung disease who develops a
spontaneous "small" pneumothorax (<15% of lung volume) can be
managed as an outpatient with analgesics and follow-up within 72
hours. CT of the lung is needed in complicated cases, including
patients with known lung disease or recurrent pneumothoraces. A
chest tube is required only when the pneumothorax involves >15%
of lung volume.
76.
=======================================================
Random Board Review Questions 53
=======================================================
A 13-year-old male presents with a 3-week history of left
lower thigh and knee pain. There is no history of a specific
injury, and his past medical history is negative. He has had no
fevers, night sweats, or weight loss, and the pain does not
awaken him at night. He tried out for the basketball team but
had to quit because of the pain, which was worse when he
tried to run.
Which one of the following physical examination findings
would be pathognomonic for slipped capital femoral
epiphysis? (check one)
A. Excessive forward passive motion of the tibia with the knee
flexed
B. Lateral displacement of the patella with active knee flexion
C. Limited internal rotation of the flexed hip
D. Reduced hip abduction with the hip flexed
E. Inability to extend the hip past the neutral position
77.
=======================================================
Random Board Review Questions 54
=======================================================
You test a patient's muscles and find that his maximum
performance consists of the ability to move with gravity
neutralized. This qualifies as which grade of muscle strength,
on a scale of 5? (check one)
C. Limited internal rotation of the flexed hip. Slipped capital femoral
epiphysis (SCFE) typically occurs in young adolescents during the
growth spurt. Physical activity, obesity, and male gender are
predisposing factors for the development of this condition, in which
the femoral head is displaced posteriorly through the growth plate.
There is pain with physical activity, most commonly in the upper
thigh anteriorly, but one-third of patients present with referred
lower thigh or knee pain, which can make accurate and timely
diagnosis more difficult. The hallmark of SCFE on examination is
limited internal rotation of the hip. Specific to SCFE is the even
greater limitation of internal rotation when the hip is flexed to 90.
No other pediatric condition has this physical finding, which makes
the maneuver very useful in children with lower extremity pain.
Orthopedic consultation is advised if SCFE is suspected. Hip
extension and abduction are also limited in SCFE, but these
findings are nonspecific. The knee findings in this patient are not
associated with SCFE.
C. 2. Muscle strength is scored on a scale of 0 to 5. The inability to
contract a muscle is scored as 0. Contraction without movement
constitutes grade 1 strength. Movement with the effect of gravity
neutralized is grade 2 strength, while movement against gravity
only is grade 3 strength. Movement against gravity plus some
additional resistance indicates grade 4 strength. Normal, or grade
5, strength is demonstrated by movement against substantial
resistance.
A. 0
B. 1
C. 2
D. 3
E. 4
78.
=======================================================
Random Board Review Questions 55
=======================================================
A 7-year-old Hispanic female has a 3-day history of a fever of
40.0C (104.0F), muscle aches, vomiting, anorexia, and
headache. Over the past 12 hours she has developed a painless
maculopapular rash that includes her palms and soles but
spares her face, lips, and mouth. She has recently returned
from a week at summer camp in Texas. Her pulse rate is 140
beats/min, and her blood pressure is 80/50 mm Hg in the right
arm while lying down.
Which one of the following is the most likely diagnosis?
(check one)
A. Mucocutaneous lymph node syndrome
B. Leptospirosis
C. Rocky Mountain spotted fever
D. Scarlet fever
E. Toxic shock syndrome
C. Rocky Mountain spotted fever. While all of the diagnoses listed
are in the differential, the most likely is Rocky Mountain spotted
fever (RMSF) (SOR C). It occurs throughout the United States, but is
primarily found in the South Atlantic and south central states. It is
most common in the summer and with exposure to tall vegetation
(e.g., while camping, hiking, or gardening), and is transmitted by
ticks. The diagnosis is based on clinical criteria that include fever,
hypotension, rash, myalgia, vomiting, and headache (sometimes
severe). The rash associated with RMSF usually appears 2-4 days
after the onset of fever and begins as small, pink, blanching
macules on the ankles, wrists, or forearms that evolve into
maculopapules. It can occur anywhere on the body, including the
palms and soles, but the face is usually spared. Mucocutaneous
lymph node syndrome is a similar condition in children (usually <2
years old), but symptoms include changes in the lips and oral
cavity, such as strawberry tongue, redness and cracking of the lips,
and erythema of the oropharyngeal mucosa. Leptospirosis is
usually accompanied by severe cutaneous hyperesthesia. The
patient with scarlet fever usually has prominent pharyngitis and a
fine, papular, erythematous rash. Toxic shock syndrome may
present in a similar fashion, but usually in postmenarchal females.
79.
=======================================================
Random Board Review Questions 56
=======================================================
The most common presenting symptom of obstructive sleep
apnea is: (check one)
A. excessive daytime sleepiness. The most common presenting
symptom of obstructive sleep apnea is excessive daytime
sleepiness (SOR A). Other symptoms include snoring, unrefreshing
or restless sleep, witnessed apneas and nocturnal choking,
morning headache, nocturia or enuresis, gastroesophageal reflux,
and reduced libido.
A. excessive daytime sleepiness
B. snoring
C. morning headache
D. gastroesophageal reflux
E. enuresis
80.
=======================================================
Random Board Review Questions 57
=======================================================
A 68-year-old African-American male with a history of
hypertension and heart failure continues to have shortness of
breath and fatigue after walking only one block. He has
normal breath sounds, no murmur, and no edema on
examination. His current medications include furosemide
(Lasix), 20 mg/day, and metoprolol extended-release (ToprolXL), 50 mg/day. He previously took lisinopril (Prinivil, Zestril),
but it was discontinued because of angioedema. A recent
echocardiogram showed an ejection fraction of 35%.
Which one of the following would be most likely to improve
both symptoms and survival in this patient?
(check one)
E. Isosorbide/hydralazine (BiDil). In patients with systolic heart
failure, the usual management includes an ACE inhibitor and a blocker. Since this patient had angioedema with an ACE inhibitor,
an angiotensin receptor blocker may cause this side effect as well.
Adding metolazone is generally not necessary unless the patient
has volume overload that does not respond to increased doses of
furosemide. Digoxin may improve symptoms, but has not been
shown to increase survival. For patients who cannot tolerate an
ACE inhibitor, especially African-Americans, a combination of directacting vasodilators such as isorbide and hydralazine is
preferred.Verapamil has a negative inotropic effect and should not
be used.
A. Valsartan (Diovan)
B. Metolazone (Zaroxolyn)
C. Digoxin
D. Verapamil (Calan, Isoptin)
E. Isosorbide/hydralazine (BiDil)
81.
=======================================================
Random Board Review Questions 58
=======================================================
A 52-year-old male requests "everything you've got" to help
him stop smoking. You review common barriers to quitting
and the benefits of cessation with him, and develop a plan
that includes follow-up. He chooses to start varenicline
(Chantix) to assist with his efforts, and asks about also using
nicotine replacement.
Which one of the following would be accurate advice?
(check one)
A. Combining these medications has not proven to be
beneficial
B. The addition of transdermal nicotine, but not nicotine
gum, has proven benefits
C. The combination is highly efficacious
D. Nicotine replacement doses need to be doubled in a
patient taking varenicline
E. The combination of nicotine and varenicline is potentially
lethal
A. Combining these medications has not proven to be beneficial.
Varenicline works by binding to nicotine receptors in the brain,
providing much lower stimulation than nicotine itself would. This
has the effect of reducing the reinforcement and reward that
smoking provides to the brain. However, this medication also blocks
the benefit a patient would receive from nicotine replacement
products. Studies have shown that using nicotine replacement
products concurrently with varenicline leads to an increase in
nausea, headaches, dizziness, and fatigue.
82.
=======================================================
Random Board Review Questions 59
=======================================================
A 40-year-old male with a 20-pack-year history of smoking is
concerned about lung cancer. He denies any constitutional
symptoms, or breathing or weight changes. You encourage
him to quit smoking and order which one of the following?
(check one)
A. No testing. This patient is at risk for lung cancer, even with no
symptoms. He should be encouraged to stop smoking, especially if
he has concerns that may help motivate him to quit. No study has
demonstrated that screening with any of the tests listed improves
survival, and no major organization endorses lung cancer
screening.
A. No testing
B. A chest radiograph
C. Low-dose CT of the chest
D. Sputum cytology
83.
=======================================================
Random Board Review Questions 60
=======================================================
You are a member of a committee at your local hospital that
has been asked to develop measures to reduce the incidence
of postoperative methicillin-resistant Staphylococcus aureus
(MRSA) infections. Which one of the following would be most
effective for preventing these infections? (check one)
A. Give preoperative antibiotics to all surgical patients to
eradicate bacteria
B. Screen all admitted patients for MRSA and use antibiotics
pre- and postoperatively in positive cases
C. Culture the nares of all hospital employees upon hiring and
on a routine basis thereafter
D. Institute an intensive program of good hand washing for
all employees
D. Institute an intensive program of good hand washing for all
employees. Nosocomial infections are a significant factor in
morbidity and cost in the health care field. Methicillinresistant
Staphylococcus aureus (MRSA) has rapidly increased in frequency,
first being found only at tertiary centers, then local hospitals, and
now in the outpatient setting.
In 2004, an estimated 1.5% of U.S. residents carried MRSA in the
anterior nares of the nose. Of those who are found to be colonized,
either at the time of hospitalization or later by a routine culture,
25% will develop a MRSA infection. However, a recent study
showed that of 93 patients who became infected with the organism,
57% were not colonized at the time of infection. The study also
attempted to screen all patients for MRSA on admission, but found
that even though 337 previously unknown carriers were found (in
addition to those already known to harbor the organism), there
was not a significant decrease in the rate of MRSA infections during
the study.
Although MRSA infections can be serious, they comprise only 8% of
nosocomial infections in the hospital, and concentrating prevention
efforts only on MRSA has little effect on that 8%, and no effect on
the 92% of infections caused by other organisms.
Iatrogenic complications arise from trying to treat MRSA carriers,
including both drug reactions and the development of other
resistant organisms. Costs related to attempts at prophylaxis also go
up.
Culturing all hospital employees has not been proven to be of
value, as employees can pick up the organism after screening, and
also can spontaneously eradicate the organism without treatment.
The best way to prevent complications and postoperative infections
is to aggressively advocate universal and frequent hand washing
and room cleaning, and use good isolation techniques and
methods of preventing infection, such as strict catheter and
intravenous tubing protocols.
84.
=======================================================
Random Board Review Questions 61
=======================================================
A 35-year-old male consults you about a vague chest pain he
developed while sitting at his desk earlier in the day. The pain
is right-sided and was sharp for a brief time when it began,
but it rapidly subsided. There was no hemoptysis and the
pain does not seem pleuritic. His physical examination, EKG,
and oxygen saturation are unremarkable. A chest film shows
a 10% right pneumothorax. Which one of the following is true
in this situation? (check one)
A. He is likely to be an overweight smoker with a chronic
cough
B. Rupture of subpleural bullae would be an unlikely cause of
his problem
C. Outpatient observation with a repeat chest radiograph in
24 hours is indicated
D. A chest tube should be placed expeditiously
E. After treatment his probability of recurrence is less than
15%
85.
=======================================================
Random Board Review Questions 62
=======================================================
A 24-year-old African-American male presents with a history
of several weeks of dyspnea, cough productive of bloody
streaks, and malaise. His examination is normal except for
bilateral facial nerve palsy. A CBC and urinalysis are normal. A
chest radiograph reveals bilateral lymph node
enlargement.This presentation is most consistent with:
(check one)
A. polyarteritis nodosa
B. Goodpasture's syndrome
C. sarcoidosis
D. pulmonary embolus
C. Outpatient observation with a repeat chest radiograph in 24
hours is indicated. The majority of patients presenting with
spontaneous pneumothorax are tall, thin individuals under 40
years of age. Most do not have clinically apparent lung disease, and
the chest pain is sometimes minimal at onset and may resolve
within 24 hours even if untreated. Patients with small
pneumothoraces involving less than 15% of the hemithorax may
have a normal physical examination, although tachycardia is
occasionally noted. The diagnosis is confirmed by chest
radiographs. Studies of recurrence have found that an average of
30% of patients will have a recurrence within 6 months to 2 years.
The treatment of an initial pneumothorax of less than 20% may be
monitored if a patient has few symptoms. Follow-up should include
a chest radiograph to assess stability at 24-48 hours. Indications for
treatment include progression, delayed expansion, or the
development of symptoms. The majority of patients with
spontaneous pneumothoraces, and perhaps almost all of them, will
have subcutaneous bullae on a CT scan.
C. sarcoidosis. Sarcoidosis, a disease of unknown etiology, affects
young to middle-age adults (predominantly 20-29 years ld). In the
U.S. it is more common in African-Americans. It is asymptomatic in
30%-50% of patients, and is often diagnosed on a routine chest film.
About one-third of cases will present with fever, malaise, weight
loss, cough, and dyspnea. The pulmonary system is the main organ
system affected, and findings may include bilateral hilar
lymphadenopathy and discrete, noncaseating epithelial
granulomas. Facial nerve palsy is seen in <5% of patients, and
usually occurs late in the process. Before Lyme disease was
recognized, bilateral facial nerve palsy was almost always due to
sarcoidosis. Hemoptysis does not generally occur until late in the
course of sarcoidosis, and is usually related to Aspergillus infection
or cavitation. Renal involvement rarely results in significant
proteinuria or hematuria. Polyarteritis nodosa may involve the
lungs. Although pneumonic episodes may be associated with
hemoptysis in a small percentage of patients, the chest radiograph
is more likely to reveal granulomatous lesions rather than patchy
infiltrates. Goodpasture's syndrome is characterized by pulmonary
hemorrhage, glomerulonephritis, and antiglomerular basement
membrane antibodies. Hemoptysis, pulmonary alveolar infiltrates,
dyspnea, and iron-deficiency anemia are frequent presenting
features. Within days or weeks, the pulmonary findings are
generally followed by hematuria, proteinuria, and the rapid loss of
renal function. Pulmonary embolus is an acute event, and would
present with dyspnea and possibly hemoptysis,
but not hilar lymphadenopathy.
86.
=======================================================
Random Board Review Questions 63
=======================================================
A 4-week-old white male is brought to your office with a 2week history of increasing dyspnea, cough, and poor feeding.
The child appears nontoxic and is afebrile. On examination
you note conjunctivitis, and a chest examination reveals
tachypnea and rales. A chest film shows hyperinflation and
diffuse interstitial infiltrates. A WBC count reveals
eosinophilia.
What is the most likely etiologic agent?
(check one)
A. Staphylococcus species
B. Chlamydia trachomatis
C. Respiratory syncytial virus
D. Parainfluenza virus
87.
=======================================================
Random Board Review Questions 64
=======================================================
A 15-year-old white female who has had regular periods since
age 12 comes to your office because of secondary amenorrhea
and a milky discharge from her breasts. A pregnancy test is
negative.
The best test for initial evaluation of the pituitary in this
patient is:
(check one)
A. plasma antidiuretic hormone
B. plasma ACTH
C. serum prolactin
D. serum FSH and LH
E. fasting growth hormone
B. Chlamydia trachomatis. Chlamydial pneumonia is usually seen in
infants 3-16 weeks of age, and they frequently have been sick for
several weeks. The infant appears nontoxic and is afebrile, but is
tachypneic with a prominent cough. Physical examination reveals
diffuse rales with few wheezes. Conjunctivitis is present in about
50% of cases. The chest film shows hyperinflation and diffuse
interstitial or patchy infiltrates.
Staphylococcal pneumonia has a sudden onset. The infant appears
very ill and has a fever. At the time of onset there may be an
expiratory wheeze simulating bronchiolitis. Signs of abdominal
distress, tachypnea, dyspnea, and localized or diffuse
bronchopneumonia or lobar disease may be present. The WBC
count shows a prominent leukocytosis.
Respiratory syncytial infections start with rhinorrhea and
pharyngitis, followed in 1-3 days by cough and wheezing.
Auscultation reveals diffuse rhonchi, fine rales, and wheezes. The
chest film is often normal. If the illness progresses, cough and
wheezing increase, air hunger and intercostal retractions develop,
and evidence of hyperexpansion of the chest is seen. In some
infants, the course of the illness may be similar to that of
pneumonia. Rash or conjunctivitis may occur occasionally, and
fever is an inconsistent sign. The WBC count is normal or elevated,
and the differential may be normal or shifted either to the right or
left. Chlamydial infections may be differentiated from respiratory
syncytial infections by a history of
conjunctivitis and a subacute onset. Coughing is prominent, but
wheezing is not. There may also be
eosinophilia. Fever is usually absent.
Parainfluenza virus infection presents with typical cold symptoms.
Eight percent of infections affect the upper respiratory tract. In
children hospitalized for severe respiratory illness, parainfluenza
viruses account for about 50% of the cases of laryngotracheitis and
about 15% each of the cases of bronchitis, bronchiolitis, and
pneumonia.
C. serum prolactin. Anterior pituitary hormone overproduction is
suspected on clinical grounds and confirmed by appropriate
laboratory evaluation. The most common secretory pituitary
adenomas are prolactinomas. They cause galactorrhea and
hypogonadism, including amenorrhea, infertility, and impotence.
Growth hormone-secreting tumors, which are the next most
common secretory pituitary tumors, cause acromegaly or gigantism.
Next in frequency are corticotropic (ACTH-secreting) adenomas,
which cause cortisol excess (Cushing's disease). Glycoprotein
hormone-secreting pituitary adenomas (secreting TSH, LH, or FSH)
are the least common. TSH-secreting adenomas are a rare cause of
hyperthyroidism. Paradoxically, most patients with gonadotropinsecreting adenomas have hypogonadism.
88.
=======================================================
Random Board Review Questions 65
=======================================================
A patient in the first trimester of pregnancy has just learned
that her husband has acute hepatitis B. She feels well, and her
screening test for hepatitis B surface antigen (HBsAg) was
negative last month. She has not been immunized against
hepatitis B.
Which one of the following would be the most appropriate
management of this patient?
(check one)
A. No further workup or immunization at this time, a repeat
HBsAg test near term, and treatment of the newborn if the
test is positive
B. Use of condoms for the remainder of the pregnancy, and
administration of immunization after delivery
C. Testing for hepatitis B immunity (anti-HBs), and
immunization if needed
D. Administration of hepatitis B immune globulin (HBIG) now
and hepatitis B vaccine after the first trimester
E. Administration of both HBIG and hepatitis B vaccine now
89.
=======================================================
Random Board Review Questions 66
=======================================================
A 14-year-old female sees you for follow-up after
hypercalcemia is found on a chemistry profile obtained
during a 5-day episode of vomiting and diarrhea. She is now
asymptomatic, but her serum calcium level at this visit is 11.0
mg/dL (N 8.5-10.5). Her aunt underwent unsuccessful
parathyroid surgery for hypercalcemia a few years ago.
Which one of the following laboratory findings would suggest
a diagnosis other than primary hyperparathyroidism?
(check one)
A. Low 24-hour urine calcium
B. Decreased serum phosphate
C. High-normal to increased serum chloride
D. Elevated alkaline phosphatase
E. Elevated parathyroid hormone
E. Administration of both HBIG and hepatitis B vaccine now.
Hepatitis B immune globulin (HBIG) should be administered as
soon as possible to patients with known exposure to hepatitis B.
Hepatitis B vaccine is a killed-virus vaccine and can be used safely
in pregnancy, with no need to wait until after organogenesis. This
patient has been exposed to sexual transmission for at least 6
weeks, given that the incubation period is at least that long, so it is
too late to use condoms to prevent infection. The patient is unlikely
to be previously immune to hepatitis B, given that she has no
history of hepatitis B infection, immunization, or carriage. Because
the patient's HBsAg is negative, she is not the source of her
husband's infection. Full treatment for this patient has an efficacy
of only 75% so follow-up testing is still needed.
A. Low 24-hour urine calcium. Low urine 24-hour calcium levels or a
low urine calcium to urine creatinine ratio is not characteristic of
hyperparathyroidism. This finding should suggest familial
hypocalciuric hypercalcemia (SOR C). Awareness of this condition is
important to avoid unnecessary surgery. The parathyroid
hormone level may be mildly elevated. Parathyroid hormone is
elevated in hyperparathyroidism. Serum chloride tends to be high
normal or mildly elevated. Alkaline phosphatase may be elevated
in more severe cases, while serum phosphate levels tend to be low.
90.
=======================================================
Random Board Review Questions 67
=======================================================
A 27-year-old male with a diagnosis of depression prefers to
avoid pharmacologic treatment. You agree to engage in a trial
of therapy in your office. During the treatment process, you
help the patient realize that some of his perceptions and
interpretations of reality may be false and lead to negative
thoughts. Next, you help him discover alternative thoughts
that reflect reality more closely, and to learn to discard his
previous distorted thinking. By learning to substitute healthy
thoughts for negative thoughts, he finds his mood, behavior,
and physical reaction to different situations are improved.
Which one of the following best categorizes this type of
therapy?
(check one)
C. Cognitive therapy. This patient is engaged in cognitive therapy,
which is a treatment process that helps patients correct false selfbeliefs that can lead to negative moods and behaviors. Cognitive
therapy has been shown to effectively treat patients with unipolar
major depression, and is particularly useful in patients who do not
respond to medication or who prefer nonpharmacologic therapy.
Psychoanalysis is a process of free association where repressed
memories are recovered. Biofeedback involves instrumentation
that gives feedback about a patient's physiologic response to
various situations in order to bring the autonomic nervous system
under voluntary control. Group psychotherapy is a form of
treatment in which people who are emotionally ill meet in a group
guided by a trained therapist and help one another effect
personality change. Hypnosis involves helping a patient enter a
state of heightened focal concentration and receptivity that is
typified by a feeling of involuntariness or an altered state of
consciousness.
A. Psychoanalysis
B. Biofeedback
C. Cognitive therapy
D. Group psychotherapy
E. Hypnosis therapy
91.
=======================================================
Random Board Review Questions 68
=======================================================
A 44-year-old male sees you for evaluation of an episode of
pink-tinged urine last week. He denies any flank or abdominal
pain, as well as frequency, urgency, and dysuria. He has no
prior history of renal or other urologic disease, and no other
significant medical problems. He has a 24-pack-year smoking
history. A urinalysis today reveals 8-10 RBCs/hpf. You refer him
to a urologist for cystoscopy.
Which one of the following would be the most appropriate
additional evaluation?
(check one)
A. KUB radiography
B. Transabdominal ultrasonography
C. Voiding cystourethrography
D. CT urography
E. Magnetic resonance urography
D. CT urography. CT urography or intravenous pyelography is
recommended by the American College of Radiology as the most
appropriate imaging procedure for hematuria in all patients, with
the exception of those with generalized renal parenchymal
disease, young women with hemorrhagic cystitis, children, and
pregnant females.
92.
=======================================================
Random Board Review Questions 69
=======================================================
A 35-year-old white male who has had diabetes mellitus for 20
years begins having episodes of hypoglycemia. He was previously
stable and well controlled and has not recently changed his diet
or insulin regimen.
Which one of the following is the most likely cause of the
hypoglycemia?
(check one)
B. Renal disease. The most common cause of hypoglycemia in a
previously stable, well-controlled diabetic patient who has not
changed his or her diet or insulin dosage is diabetic renal
disease. A reduction in physical activity or the appearance of
insulin antibodies (unlikely after 20 years of therapy) would
increase insulin requirements and produce hyperglycemia.
Spontaneous improvement -cell function after 20 years would
be very rare.
A. Spontaneous improvement of -cell function
B. Renal disease
C. Reduced physical activity
D. Insulin antibodies
93.
=======================================================
Random Board Review Questions 70
=======================================================
Which one of the following is recommended for the treatment of
patients with obsessive compulsive disorder? (check one)
A. Cognitive-behavioral therapy
B. Psychoanalytic therapy
C. Family therapy
D. Psychodynamic psychotherapy
E. Motivational interviewing
94.
=======================================================
Random Board Review Questions 71
=======================================================
A 31-year-old African-American female presents with the chief
complaint of bilateral galactorrhea of 3 months' duration. She
also has not menstruated for 1 year despite changing birth
control pills several times. A review of systems is otherwise
noncontributory. Except for a milky discharge with stimulation
of the breasts, her examination is within normal limits. Serum
prolactin on two occasions is >200 g/L (N 0-20). Which one of
the following would be most appropriate at this point? (check
one)
A. Stop her oral contraceptive and repeat the serum prolactin
level in 1 month
B. Order a brain MRI with enhancement and emphasis on the
pituitary fossa
C. Order bilateral mammography
D. Start the patient on risperidone (Risperdal)
A. Cognitive-behavioral therapy. Cognitive-behavioral therapy
is the recommended treatment for obsessive-compulsive
disorder (OCD).Psychoanalytic therapy has not been shown to
help treat OCD. Family therapy can help reduce family tensions
that result from the disease. Psychodynamic psychotherapy
and motivational interviewing may help patients overcome their
resistance to treatment.
B. Order a brain MRI with enhancement and emphasis on the
pituitary fossa. Galactorrhea associated with a prolactin level
>200 g/L usually indicates a prolactinoma, and requires MRI of
the pituitary with gadolinium enhancement. Many drugs can
cause galactorrhea, including oral contraceptives and
risperidone, but they would not elevate serum prolactin to this
level. Dopamine agonists such as bromocriptine or cabergoline
are the preferred treatment for most patients with
hyperprolactinemia.
95.
=======================================================
Random Board Review Questions 72
=======================================================
Sympathomimetic decongestants such as pseudoephedrine
and phenylephrine can be problematic in elderly patients
because they can: (check one)
C. worsen existing urinary obstruction. Sympathomimetic agents
can elevate blood pressure and intraocular pressure, may worsen
existing urinary obstruction, and adversely interact with -blockers,
methyldopa, tricyclic antidepressants, and oral hypoglycemic
agents and MAOIs. They also speed up the heart rate. Firstgeneration nonprescription antihistamines can enhance the
anticholinergic and sedative effects of other medications.
A. decrease blood pressure
B. cause bradycardia
C. worsen existing urinary obstruction
D. enhance the anticholinergic effects of other medications
E. enhance the sedative effects of other medications
96.
=======================================================
Random Board Review Questions 73
=======================================================
In the elderly, the risk of heat wave-related death is highest in
those who (check one)
A. have COPD
B. have diabetes and are insulin-dependent
C. have a functioning fan, but not air conditioning
D. are homebound
97.
=======================================================
Random Board Review Questions 74
=======================================================
A 5-year-old female presents with a lesion on her forearm. It
began as a red macule, turned into a small vesicle that easily
ruptured, then dried into a 1-cm honey-colored, crusted
lesion seen now. Which one of the following would be the
most appropriate therapy? (check one)
A. Oral penicillin V
B. Oral erythromycin
C. Topical disinfectant (e.g., hydrogen peroxide)
D. Topical bacitracin
E. Topical mupirocin (Bactroban)
D. are homebound. Factors associated with a higher risk of heatrelated deaths include being confined to bed, not leaving home
daily, and being unable to care for oneself. Living alone during a
heat wave is associated with an increased risk of death, but this
increase is not statistically significant. Among medical conditions, the
highest risk is associated with preexisting psychiatric illnesses,
followed by cardiovascular disease, use of psychotropic
medications, and pulmonary disease. A lower risk of heat-related
deaths has been noted in those who have working air conditioning,
visit air-conditioned sites, or participate in social activities. Those
who take extra showers or baths and who use fans have a lower
risk, but this difference is not statistically significant.
E. Topical mupirocin (Bactroban). Topical mupirocin is as effective as
cephalexin or amoxicillin/clavulanate in the treatment of impetigo,
which is most often caused by Staphlococcal species. Oral penicillin
V, oral erythromycin, and topical bacitracin are less effective than
mupirocin. Topical treatment is well suited to this localized lesion.
Topical disinfectants such as hydrogen peroxide are no more
effective than placebo.
98.
=======================================================
Random Board Review Questions 75
=======================================================
A 59-year-old male reports nausea, vomiting, and progressive
fatigue for the past few months. At his last visit, 6 months
ago, his blood pressure was poorly controlled and
hydrochlorothiazide was added to his -blocker therapy. At
this visit he appears moderately dehydrated on examination.
Laboratory testing reveals a serum calcium level of 12.5
mg/dL (N 8.0-10.0), a BUN level of 36 mg/dL (N 6-20), and a
creatinine level of 2.2 mg/dL (N 0.6-1.1). A CBC, albumin level,
and electrolyte levels are normal. His intact parathyroid
hormone level is reported a few days later, and is 60 pg/mL (N
10-65). What is the most likely cause of his hypercalcemia?
(check one)
A. Renal failure
B. Hyperparathyroidism
C. Milk alkali syndrome
D. Sarcoidosis
99.
=======================================================
Random Board Review Questions 76
=======================================================
Which one of the following is the most likely cause of hearing
loss in newborns? (check one)
A. Intraventricular hemorrhage
B. Anomalies of the external ear canal
C. Congenital cholesteatoma
D. Genetic disorders
E. Infectious diseases
B. Hyperparathyroidism. Many patients have mild
hyperparathyroidism that becomes evident only with an added
calcium load. Thiazide diuretics reduce calcium excretion and can
cause overt symptoms in a patient whose hyperparathyroidism
would otherwise have remained asymptomatic. The finding of a
normal parathyroid hormone (PTH) level in a patient with
hypercalcemia is diagnostic for hyperparathyroidism, since PTH
should be suppressed in the presence of elevated calcium.
Symptomatic hypercalcemia causes dehydration because of both
intestinal symptoms and diuresis. Reversible renal insufficiency can
result, and can become permanent if it is long-standing and severe.
Conversely, renal failure usually causes hypocalcemia, but can
cause hypercalcemia resulting from tertiary hyperparathyroidism.
This develops after severe hyperphosphatemia and vitamin D
deficiency eventually produce hypersecretion of PTH. This patient's
renal insufficiency is not severe enough to cause tertiary
hyperparathyroidism. Milk alkali syndrome is hypercalcemia
resulting from a chronic overdose of calcium carbonate, and is
becoming more common as more patients take calcium and vitamin
D supplements. In milk alkali syndrome, and other causes of
hypercalcemia such as sarcoidosis, the PTH level is appropriately
suppressed.
D. Genetic disorders. Genetic disorders (e.g., Waardenburg
syndrome, Usher's syndrome, Alport syndrome, and Turner's
syndrome) are responsible for more than 50% of hearing
impairments in children. Intraventricular hemorrhage is a central
cause of hearing loss, and is rare. Conductive abnormalities such as
external canal anomalies and congenital cholesteatoma, and
sensorineural causes other than genetic disorders (e.g., infectious
diseases) are important but less frequent.
100.
=======================================================
Random Board Review Questions 77
=======================================================
A 3-year-old female is brought to your office for a health
maintenance examination, and her father expresses concern
about her vision. Her visual acuity is 20/20 bilaterally on a
tumbling E visual acuity chart. With both eyes uncovered
during a cover/uncover test, the corneal light reflex in the
right eye is medial to the pupil when focused on a fixed point,
but the light reflex in the left eye is almost centered in the
pupil. When the left eye is covered, the right eye moves
quickly inward to focus on the fixed point, and the corneal
light reflex is centered in the pupil. When the left eye is
uncovered, the right eye returns to its original position.
When you cover the right eye, no left eye movement is noted.
Which one of the following is the most likely diagnosis?
(check one)
A. Strabismus. Strabismus is an ocular misalignment that can be
diagnosed on a cover/uncover test when the corneal light reflex is
deviated from its normal position slightly nasal to mid-pupil. The
misaligned eye then moves to fixate on a held object when the
opposite eye is covered. The eye drifts back to its original position
when the opposite eye is uncovered. Amblyopia is cortical visual
impairment from abnormal eye development-most often as a
result of strabismus. Cataract is a less frequent cause of
amblyopia. Esotropia is a type of strabismus with an inward or
nasal deviation of the eye that would be evidenced by a corneal
light reflex lateral to its normal position. (The outward eye
deviation seen in this patient is exotropia.) Heterophoria, or latent
strabismus, does not cause eye deviation when both eyes are
uncovered.
A. Strabismus
B. Amblyopia
C. Cataract
D. Esotropia
E. Heterophoria
101.
=======================================================
Random Board Review Questions 78
=======================================================
A 26-year-old female presents with a 1-year history of
recurring abdominal pain associated with intermittent
diarrhea, 5-7 days per month. Her pain improves with
defecation. There has been no blood in her stool and no
weight loss. Laboratory findings are normal, including a CBC,
chemistry profile, TSH level, and antibodies for celiac disease.
Which one of the following would be most appropriate at
this point? (check one)
A. Colonoscopy
B. An upper GI series with small-bowel follow-through
C. Abdominal CT with contrast
D. A gluten-free diet
E. Loperamide (Imodium)
E. Loperamide (Imodium). This patient has classic symptoms of
irritable bowel syndrome (IBS) and meets the Rome criteria by
having 3 days per month of abdominal pain for the past 3 months,
a change in the frequency of stool, and improvement with
defecation. According to current clinical guidelines IBS can be
diagnosed by history, physical examination, and routine
laboratory testing, as long as there are no warning signs. Warning
signs include rectal bleeding, anemia, weight loss, fever, a family
history of colon cancer, onset of symptoms after age 50, and a
major change in symptoms. Colonoscopy, CT, and GI contrast
studies are not indicated. A gluten-free diet would not be
indicated since the antibody tests for celiac disease are negative.
Antidiarrheal agents such as loperamide are generally safe and
effective in the management of diarrheal symptoms in IBS.
102.
=======================================================
Random Board Review Questions 79
=======================================================
Which one of the following would be the most effective
monotherapy for alcohol withdrawal syndrome? (check one)
A. Clonidine (Catapres)
B. Phenytoin (Dilantin)
C. Atenolol (Tenormin)
D. Phenobarbital
E. Chlordiazepoxide (Librium)
103.
=======================================================
Random Board Review Questions 80
=======================================================
A 70-year-old female with type 2 diabetes mellitus is admitted
to the hospital with a 4-week history of fever, anorexia, and
weight loss. Two blood cultures are positive for
Streptococcus bovis. In addition to being treated for the
infection, she should be evaluated for which one of the
following? (check one)
E. Chlordiazepoxide (Librium). Alcohol withdrawal syndrome
encompasses a wide range of symptoms involving primarily the
central nervous, cardiovascular, and gastrointestinal systems, and
is mediated by the abrupt removal of alcohol-enhanced GABA
inhibition of excitatory glutamate receptors in the central nervous
system. It generally is divided into three stages, based on severity
and timeline; seizures may occur during any of these stages and
may be the first sign of withdrawal. The ideal pharmacologic agent
should provide not only safe sedation but also protection from
seizures. Long-acting benzodiazepines such as chlordiazepoxide
have been shown to be superior to the other choices in numerous
studies. Clonidine and atenolol have been found to be useful in
symptom reduction but not in seizure prevention. Phenytoin
would seem to offer protection from seizures, but studies have not
consistently shown this to be the case. Phenobarbital, while
effective, has a very narrow therapeutic window, making its use
problematic.
E. Colorectal cancer. For unknown reasons, Streptococcus bovis
bacteremia or endocarditis is associated with a high incidence of
occult colorectal malignancies. It may also occur with upper
gastrointestinal cancers. Radiography or endoscopy is indicated.
A. B-cell lymphoma
B. T-cell lymphoma
C. Multiple myeloma
D. Lung cancer
E. Colorectal cancer
104.
=======================================================
Random Board Review Questions 81
=======================================================
An obviously intoxicated 50-year-old white male is brought
to the emergency department after the car he was driving hit
a telephone pole. He has a fracture of the femur, and is
confused and uncooperative. His pulse rate is 120 beats/min,
his blood pressure is 80/40 mm Hg, and his skin is clammy.
Initial physical examination of his abdomen does not
indicate significant intra-abdominal injury. Which one of the
following would be best for determining whether
laparotomy is needed? (check one)
A. CT of the abdomen
B. MRI of the abdomen
C. Upright and lateral decubitus radiographs of the abdomen
D. Contrast duodenography
E. Peritoneal lavage
E. Peritoneal lavage. Physical examination of the abdomen is often
unreliable for detecting significant intra-abdominal injury,
especially in the head-injured or intoxicated patient. In a
hemodynamically unstable patient with a high-risk mechanism of
injury and altered mental status, peritoneal lavage is the quickest,
most reliable modality to determine whether there is a
concomitant intra-abdominal injury requiring laparotomy. CT of
the abdomen and contrast duodenography may complement
lavage in stable patients with negative or equivocal lavage results,
but in an unstable or uncooperative patient these studies are too
time-consuming or require ill-advised sedation. Ultrasonography
may also complement lavage in selected patients, but its
usefulness is limited in the acute situation. MRI is extremely
accurate for the anatomic definition of structural injury, but
logistics limit its practical application in acute abdominal trauma.
105.
=======================================================
Random Board Review Questions 82
=======================================================
A 75-year-old African-American female is diagnosed with
macular degeneration. She is being treated for type 2 diabetes
mellitus, hypothyroidism, hypertension,
hypercholesterolemia, and gout. Which one of her
conditions is associated with macular degeneration? (check
one)
C. Hypertension. Age-related macular degeneration is the most
common cause of blindness in the older population. It occurs more
frequently in light-skinned individuals than in dark-skinned
individuals. Risk factors include smoking and hypertension.
A. Type 2 diabetes mellitus
B. Hypothyroidism
C. Hypertension
D. Gout
106.
=======================================================
Random Board Review Questions 83
=======================================================
A 4-year-old male has a fever of 1 week's duration. It has been
at or slightly above 38 C (101 F) and has responded poorly to
antipyretics. The patient complains of photophobia, burning
in his eyes, and a sore throat. His mother also notes that his
eyes look red, his lips are red and cracked, and he has a
"strawberry tongue." The child's palms and soles are
erythematous and the periungual regions show
desquamation of the skin. He has minimally painful nodes
located in the anterior cervical region, about 22 cm in size. A
Streptococcus screen is negative. The most appropriate
management at this time would be: (check one)
A. Intramuscular benzathine penicillin G (Bicillin L-A), 600,000
U
B. Intravenous nafcillin (Nallpen)
C. Intravenous immune globulin and aspirin
D. Prednisone, 2-3 mg/kg daily
E. A fine-needle biopsy of the lymph nodes
C. Intravenous immune globulin and aspirin. Kawasaki disease, or
mucocutaneous lymph node syndrome, is a common form of
vasculitis in childhood. It is typically self-limited, with fever and
acute inflammation lasting 12 days on average without therapy.
However, if untreated, this illness can result in heart failure,
coronary artery aneurysm, myocardial infarction, arrhythmias, or
occlusion of peripheral arteries. It is most common in those under
the age of 5 years. To diagnose this disease, fever must be present
for 5 days or more with no other explanation. In addition, at least
four of the following symptoms must be present: 1) nonexudative
conjunctivitis that spares the limbus; 2) changes in the oral
membranes such as diffuse erythema, injected or fissured lips, or
"strawberry tongue"; 3) erythema of palms and soles, and/or
edema of the hands or feet followed by periungual
desquamation; 4) cervical adenopathy in the anterior cervical
triangle with at least one node larger than 1.5 cm in diameter; and,
5) an erythematous polymorphous rash, which may be targetoid
or purpuric in 20% of cases. The disease must be distinguished
from toxic shock syndrome, streptococcal scarlet fever, StevensJohnson syndrome, juvenile rheumatoid arthritis, measles,
adenovirus infection, echovirus infection, and drug reactions.
Treatment significantly diminishes the risk of complications.
Current recommendations are to hospitalize the patient for
treatment with intravenous immune globulin. In addition, aspirin
is used for both its anti-inflammatory and antithrombolitic effects.
While prednisone is used to treat other forms of vasculitis, it is
considered unsafe in Kawasaki disease, as a previous study
showed an extraordinarily high rate of coronary artery aneurysm
with its use.
107.
=======================================================
Random Board Review Questions 84
=======================================================
In 2001, the National Cholesterol Education Program
published updated guidelines for cholesterol testing and
management, as recommended by its Expert Panel on
Detection, Evaluation, and Treatment of High Blood
Cholesterol in Adults. According to these guidelines and the
2004 Adult Treatment Panel (ATP) III Update, the target LDL
cholesterol for patients with type 2 diabetes mellitus is:
(check one)
A. 60 mg/dL
B. 100 mg/dL
C. 130 mg/dL
D. 160 mg/dL
E. 200 mg/dL
108.
=======================================================
Random Board Review Questions 85
=======================================================
A 3-week-old white male presents with a history of several
days of projectile vomiting after feeding, and documented
weight loss despite a good appetite. There is a questionable
history of a paternal uncle having surgery for a similar
problem when he was an infant. Which one of the following
findings is a characteristic sign of this disease? (check one)
A. Hypochloremic alkalosis
B. Pneumonia
C. Generalized abdominal distention
D. Currant jelly stool
E. Direct hyperbilirubinemia
109.
=======================================================
Random Board Review Questions 86
=======================================================
A 72-year-old white female who is otherwise healthy
complains of occasional incontinence. She reports that this
occurs mainly at night when she awakens with an intense
desire to void, and by the time she is able to get to the
bathroom she has "wet herself." The most likely diagnosis is:
(check one)
A. Sphincter incompetence
B. Detrusor instability
C. Detrusor hypotonia
D. Uninhibited neurogenic bladder
B. 100 mg/dL. The 2001 National Cholesterol Education Program
Adult Treatment Panel III guidelines and the 2004 update, as well
as guidelines previously published by the American Diabetes
Association, have established a target LDL cholesterol level of 100
mg/dL for patients with diabetes. This target is also applicable for
individuals with known coronary artery disease (CAD),
symptomatic carotid artery disease, abdominal aortic aneurysm,
peripheral vascular disease, and multiple risk factors that confer a
10-year risk for coronary heart disease that is >20%. An LDL level
of 130 mg/dL is acceptable for other individuals with only two risk
factors for CAD and a 10-year CAD risk <20%, and 160 mg/dL is the
upper limit of acceptability for patients with no more than one risk
factor for CAD and a 10-year CAD risk <20%.
A. Hypochloremic alkalosis. Hypertrophic pyloric stenosis is the
most likely diagnosis in this case. If it is allowed to progress
untreated, there may be signs of malnutrition, constipation,
oliguria, and profound hypochloremic metabolic alkalosis. The
latter is a characteristic sign of pyloric obstruction. As the child
vomits chloride and hydrogen-rich gastric contents,
hypochloremic alkalosis sets in. Pneumonia is not a common
problem with pyloric stenosis, as it can be with congenital
tracheoesophageal fistulae for example. After feeding, there may
be a visible peristaltic wave that progresses across the abdomen.
However, since the point of obstruction is proximal to the small
and large intestines and affected infants lose weight, the
abdomen is usually flat rather than distended, especially in the
malnourished infant. Currant jelly stool is a common clinical
manifestation of intussusception. Mild jaundice with elevated
indirect bilirubin is seen in about 5% of infants with pyloric
stenosis, but is not a characteristic sign.
B. Detrusor instability. Urinary incontinence is very common in the
elderly female. Treatment depends entirely on a careful history to
ascertain the exact circumstances when the patient wets herself.
One of the most common types of incontinence results from
uninhibited contractions of the detrusor muscle. This detrusor
instability causes an intense urge to void, which overcomes the
patient's voluntary attempt to hold the sphincter closed; hence,
the common term urge incontinence. Other common causes of
incontinence include a weak sphincter (sphincter incompetence),
which leads to leakage associated with ordinary activities such as
coughing or lifting (stress incontinence). Another common cause is
overflow of urine from an abnormally distended, hypotonic,
poorly contractile bladder (detrusor hypotonia). This is probably
more common in males with longstanding obstruction due to
prostatic hypertrophy. A rare type of incontinence is caused by
spinal cord damage. This reflex incontinence is due to the patient
being unable to sense the need to void.
110.
=======================================================
Random Board Review Questions 87
=======================================================
A 4-week-old full-term male is brought to your office by his parents. They
report that their child started vomiting just after his 1-week visit. The
parents are concerned because they think the vomiting is worsening,
occurring after every feeding, and "shooting across the room." You note
that the baby is afebrile, but has not gained any weight since birth. Based
on this information, the most likely diagnosis is: (check one)
D. Pyloric stenosis. Pyloric stenosis fits the described
scenario; it is characterized by the early onset of
worsening projectile vomiting and poor weight gain,
and occurs most often in full-term male infants who
are otherwise healthy. Formula intolerance causes
regurgitation, as would inappropriate feeding.
Meningitis, whether viral or bacterial, would be
associated with fever. Viral gastroenteritis is a common
cause of vomiting in older children, and is usually
associated with fever and diarrhea.
A. Formula intolerance
B. Meningitis
C. Viral gastroenteritis
D. Pyloric stenosis
E. Inappropriate feeding
111.
=======================================================
Random Board Review Questions 88
=======================================================
A 68-year-old white female presents to your office and reports that
yesterday she had a 20-minute episode of difficulty speaking and
weakness of the right side of the face and right arm. She has never
experienced any episodes similar to this in the past and reports her
overall health to be excellent. In fact, she tells you that she has not seen a
physician since her hysterectomy for fibroids 20 years ago. Her only
medication is occasional acetaminophen or ibuprofen for knee pain.
Physical examination reveals a blood pressure of 160/90 mm Hg, an
irregularly irregular heartbeat with a rate of 90/min, an otherwise normal
cardiovascular examination, and a completely normal neurologic
examination. Her EKG confirms atrial fibrillation with evidence for left
ventricular hypertrophy but no Q waves or ST elevation. You are able to
obtain an emergent CT scan of the brain without contrast, which is
negative. Which one of the following is the most appropriate immediate
management? (check one)
A. Lowering blood pressure
B. Antiplatelet therapy with clopidogrel (Plavix)
C. Anticoagulation with warfarin (Coumadin)
D. Electrical or chemical cardioversion
E. An MRI scan of the brain with contrast
C. Anticoagulation with warfarin (Coumadin). The
patient described presents with a history most
consistent with a recent, resolved transient ischemic
attack (TIA). This was most likely due to an embolus
related to her atrial fibrillation. Her risk for a
recurrent neurologic event (TIA or stroke) is high.
Long-term anticoagulation with warfarin reduces this
risk. The use of antiplatelet agents such as clopidogrel
to reduce TIAs has not been studied. Lowering blood
pressure and lipid levels can reduce risks over the
long term, but do not require immediate intervention.
Cardioversion for patients with atrial fibrillation of
uncertain or long duration may be appropriate but
should not be attempted before several weeks of
anticoagulation in the stable patient.
112.
=======================================================
Random Board Review Questions 89
=======================================================
A 78-year-old white female presents with a 3-day history of lower thoracic back
pain. She denies any antecedent fall or trauma, and first noted the pain upon
arising. Her description of the pain indicates that it is severe, bilateral, and
without radiation to the arms or legs. Her past medical history is positive for
hypertension and controlled diabetes mellitus. Her medications include
hydrochlorothiazide, enalapril (Vasotec), metformin (Glucophage), and a
general multivitamin. She is a previous smoker but does not drink alcohol. She
underwent menopause at age 50 and took estrogen for "a few months" for hot
flashes. Physical examination reveals her to be in moderate pain with a
somewhat stooped posture and mild tenderness over T12-L1. She has negative
straight-leg raising and normal lower extremity sensation, strength, and
reflexes. Which one of the following is true regarding this patient's likely
condition? (check one)
D. Subcutaneous or intranasal calcitonin
(Calcimar, Miacalcin) may be very helpful for
pain relief. The patient described has a classic
presentation of an osteoporotic vertebral
compression fracture. The diagnosis should be
confirmed with a plain radiograph. Treatment
is basically symptomatic, with a period of bed
rest as short as possible (to avoid complications
of immobility), pain medication, and bracing.
Salmon calcitonin (injectable or intranasal) is
often helpful in providing pain relief. Longterm management of underlying osteoporosis
may help prevent future fractures.
A. An MRI or nuclear medicine bone scan should be performed
B. Prolonged (approximately 2 weeks) bed rest will increase the chance of
complete recovery
C. Investigation for an underlying malignancy is indicated
D. Subcutaneous or intranasal calcitonin (Calcimar, Miacalcin) may be very
helpful for pain relief
113.
=======================================================
Random Board Review Questions 90
=======================================================
The most common stress fracture in children involves which one of the
following bones? (check one)
A. Calcaneus
B. Tibia
C. Fibula
D. Tarsal navicular
E. Metatarsal
B. Tibia. Tibial fractures are the most common
lower extremity stress fractures in both
children and adults, accounting for about half
of all stress fractures.
114.
=======================================================
Random Board Review Questions 91
=======================================================
Three members of the same family present with a high fever
and cough that began abruptly yesterday. All three report
having fevers over 40 C (104 F), painful coughs, moderate
sore throats, and prostration. They have loss of appetite, but
no vomiting or diarrhea. Two other family members have
similar symptoms. On examination the patients appear ill
and flushed. There is no cervical adenopathy, no visible
pharyngeal inflammation, and no significant findings on
examination of the chest. Which one of the following is the
most likely diagnosis? (check one)
A. Mycoplasma pneumonia
B. Influenza-like illness
C. Bacterial bronchitis
D. Upper respiratory infection
E. Legionnaires disease
115.
=======================================================
Random Board Review Questions 92
=======================================================
You have hospitalized a 67-year-old obese white female for
urosepsis. She has completed a course of intravenous
antibiotics. She has hypertension, diabetes mellitus, and
congestive heart failure. In addition, she has renal failure
which has worsened, and she has been on hemodialysis for 1
week. The chart lists her medications as enalapril (Vasotec),
furosemide (Lasix), labetalol (Trandate, Normodyne), insulin,
and heparin for flushing intravenous lines. For the past 2 days
she has had nosebleeds. A CBC is normal except for mild
anemia and marked thrombocytopenia of 28,000/mL. Which
one of the following is most likely the cause of her
thrombocytopenia? (check one)
B. Influenza-like illness. Influenza has a very abrupt onset, and a
fever with a nonproductive cough is almost always present.
Unconfirmed cases are referred to as influenza-like illness (ILI) or
suspected influenza. Patients with confirmed cases tend to say
they have never been so ill. Mycoplasma pneumonia can spread
among family members, but it is milder and has a more indolent
onset and a longer incubation period. Bacterial bronchitis is an
overdiagnosed, supposed complication of upper respiratory
infections, and is not contagious. While the phrase cold and flu is
often used, upper respiratory infections are not so febrile or
prostrating, and coryza is the dominant syndrome sooner or later.
Legionella can have point-source epidemics, but the incubation
period is longer, symptoms vary from mild illness to lifethreatening pneumonia, and diarrhea is prominent in many
cases. Reference: Thibodeau KP, Viera AJ: Atypical pathogens and
challenges in community-acquired pneumonia. Am Fam Physician
2004;69(7):1699-1706.
E. Heparin. A number of medications can cause
thrombocytopenia, but heparin is a more likely cause than
enalapril, furosemide, labetalol, or insulin. Even the small doses of
heparin used to flush intravenous lines can be a source of
thrombocytopenia.
A. Enalapril
B. Furosemide
C. Labetalol
D. Insulin
E. Heparin
116.
=======================================================
Random Board Review Questions 93
=======================================================
When draining a felon, which one of the following incisions is
recommended? (check one)
A. A "fishmouth" bilateral incision
B. A "hockey stick" J-shaped incision including the distal and
lateral aspects of the digit
C. A transverse volar incision
D. A high lateral incision
D. A high lateral incision. When draining a felon, a volar
longitudinal incision or a high lateral incision is recommended.
Incisions that are not recommended are the "fish-mouth" incision,
the "hockey stick" (or "J") incision, and the transverse palmar
incision.
117.
118.
=======================================================
Reproductive (Female) Board Review Questions 01
=======================================================
A 16-year-old female presents with a complaint of pelvic
cramps with her menses over the past 2 years. She describes
her periods as heavy, and says they occur once a month and
last for 7 days, with no spotting in between. She has never
been sexually active and does not expect this to change in the
foreseeable future. An abdominal examination is normal.
Which one of the following would be the most appropriate
next step? (check one)
A. A pelvic examination
B. Ultrasonography
C. A TSH level
D. Naproxen prior to and during menses
D. Naproxen prior to and during menses. This patient is
experiencing primary dysmenorrhea, a common finding in
adolescents, with estimates of prevalence ranging from 20% to
90%. Because symptoms started at a rather young age and she
has pain only during menses, endometriosis or other significant
pelvic pathology is unlikely. An infection is doubtful, considering
that she is not sexually active and that symptoms have been
present for 2 years. In the absence of red flags, a pelvic
examination, laboratory evaluation, and pelvic ultrasonography
are not necessary at this time. However, they can be ordered if
she does not respond to simple treatment. NSAIDs such as
naproxen have a slight effect on platelet function, but because
they inhibit prostaglandin synthesis they actually decrease the
volume of menstrual flow and lessen the discomfort of pelvic
cramping. Acetaminophen would have no effect on
prostaglandins.
=======================================================
Reproductive (Female) Board Review Questions 02
=======================================================
A 17-year-old white female presents with new-onset left-sided
lower abdominal pain. Color flow Doppler ultrasonography,
in addition to pelvic ultrasonography, would be most useful
for evaluating: (check one)
A. Adnexal torsion. Color Doppler flow studies are useful for
evaluating blood flow to the ovary in possible cases of adnexal or
ovarian torsion. Adnexal torsion is a surgical emergency. Pelvic
ultrasonography, preferably with a vaginal probe, can be
beneficial in the workup of ruptured ovarian cyst, pelvic abscess,
and pelvic inflammatory disease without abscess. The Doppler
flow study is not required with these condition.
A. Adnexal torsion
B. Pelvic abscess
C. Pelvic inflammatory disease
D. Ruptured ovarian cyst
119.
=======================================================
Reproductive (Female) Board Review Questions 03
=======================================================
A 30-year-old African-American female presents with a vaginal
discharge. On examination the discharge is homogeneous
with a pH of 5.5, a positive whiff test, and many clue cells.
Which one of the following findings in this patient is most
sensitive for the diagnosis of bacterial vaginosis? (check one)
A. The pH of the discharge. Patients must have 3 of 4 Amsel criteria
to be diagnosed with bacterial vaginosis. These include a pH >4.5
(most sensitive), clue cells >20% (most specific), a homogeneous
discharge, and a positive whiff test (amine odor with addition of
KOH).
A. The pH of the discharge
B. The presence of clue cells
C. The character of the discharge
D. The whiff test
120.
=======================================================
Reproductive (Female) Board Review Questions 04
=======================================================
A 31-year-old married white female complains of vaginal
discharge, odor, and itching. Speculum examination reveals a
homogeneous yellow discharge, vulvar and vaginal erythema,
and a "strawberry" cervix. The most likely diagnosis is: (check
one)
A. Candidal vaginitis
B. Bacterial vaginosis
C. Trichomonal vaginitis
D. Chlamydial infection
E. Herpes simplex type 2
C. Trichomonal vaginitis. Trichomonal vaginitis usually causes a
yellowish discharge which sometimes has a frothy appearance.
Colpitis macularis (strawberry cervix) is often present. Monilial
vaginitis classically causes a cheesy, whitish exudate with
associated vaginal itching and burning. There may be vaginal and
vulvar erythema and edema, but colpitis macularis is not a
feature. Bacterial vaginosis is characterized by a grayish discharge
with few other physical signs or symptoms, if any. Chlamydia may
cause a yellowish cervical discharge and symptoms of pelvic
inflammatory disease or, alternatively, may be totally
asymptomatic. Herpes simplex type 2 causes ulcerations on the
vulva and vaginal mucosa which are exquisitely tender, often with
marked surrounding erythema and edema.
121.
=======================================================
Reproductive (Female) Board Review Questions 05
=======================================================
The definition of post-term pregnancy is a pregnancy that has
reached: (check one)
C. 42 weeks' gestation. Postdate and post-term pregnancy are
terms that are used interchangeably. The postdate pregnancy is
defined as a pregnancy that has reached 42 weeks of
amenorrhea. This is important because perinatal mortality
doubles at 42 weeks gestational age. The diagnosis of postdate
pregnancy depends heavily on accurate dating methods.
A. 40 weeks' gestation
B. 41 weeks' gestation
C. 42 weeks' gestation
D. 43 weeks' gestation
122.
=======================================================
Reproductive (Female) Board Review Questions 06
=======================================================
A 22-year-old gravida 2 para 1 presents to your office with a 1day history of vaginal bleeding and abdominal pain. Her last
menstrual period was 10 weeks ago, and she had a positive
home pregnancy test 6 weeks ago. She denies any passage of
clots. On pelvic examination, you note blood in the vaginal
vault. The internal cervical os is open. Which one of the
following best describes the patient's current condition?
(check one)
A. Inevitable abortion. Inevitable abortion is defined by bleeding,
an open os, and no passage of products of conception (POCs).
Bleeding also occurs with completed abortion, but the os is closed
and there is complete passage of POCs. Threatened abortion also
is characterized by bleeding and a closed os, but there is no
passage of POCs. With incomplete abortion there is bleeding and
an open os, but POCs are visualized in the os or vaginal vault.
There are no symptoms with missed abortion, but there is no
embryo or fetus on ultrasonography.
A. Inevitable abortion
B. Completed abortion
C. Threatened abortion
D. Incomplete abortion
E. Missed abortion
123.
=======================================================
Reproductive (Female) Board Review Questions 07
=======================================================
Black cohosh is: (check one)
A. A form of herbal licorice with gastrointestinal effects
B. A botanical medicine used to alleviate menopausal
symptoms
C. A type of toxic hallucinogenic mushroom
D. A variety of Cannabis sativa
E. A form of dried hashish
124.
=======================================================
Respiratory Board Review Questions 01
=======================================================
Which one of the following is an indication for a second dose
of pneumococcal polysaccharide vaccine (Pneumovax 23) in
children? (check one)
A. Cerebrospinal fluid leak
B. Cyanotic congenital heart disease
C. Type 1 diabetes mellitus
D. Sickle cell disease
E. Chronic bronchopulmonary dysplasia
B. A botanical medicine used to alleviate menopausal symptoms.
Black cohosh is an herbal preparation widely used in the
treatment of menopausal symptoms and menstrual dysfunction.
Studies have demonstrated that this botanic medicine appears to
be effective in alleviating menopausal symptoms. It has not been
proven effective in randomized controlled trials and should not be
used to prevent osteoporosis. Questions as to its stimulating effect
on endometrial tissue are as yet unanswered.
D. Sickle cell disease. Patients with chronic illness, diabetes mellitus,
cerebrospinal fluid leaks, chronic bronchopulmonary dysplasia,
cyanotic congenital heart disease, or cochlear implants should
receive one dose of pneumococcal polysaccharide vaccine after 2
years of age, and at least 2 months after the last dose of
pneumococcal conjugate vaccine (Prevnar 13). Revaccination with
polysaccharide vaccine is not recommended for these patients.
Individuals with sickle cell disease, those with anatomic or
functional asplenia, immunocompromised persons with renal
failure or leukemia, and HIV-infected persons should receive
polysaccharide vaccine on this schedule and should be
revaccinated at least 5 years after the first dose.
125.
=======================================================
Respiratory Board Review Questions 03
=======================================================
A 72-year-old male slipped on a rug in his kitchen and struck his right
side against a counter. He presents several days after the fall with a
complaint of ongoing pain in his flank. He has a history of chronic
atrial fibrillation, which is treated with warfarin (Coumadin). His vital
signs are normal. A physical examination reveals tenderness to
palpation along the posterior-lateral chest wall and decreased breath
sounds in the right base. Radiographs reveal two fractured ribs on the
right side and a moderately large pleural effusion in the right
hemithorax. Laboratory test results include a hemoglobin of 10.5
mg/dL (baseline 11.0-12.0 mg/dL) and a prothrombin time of 33.5
seconds with an INR of 3.5. Which one of the following would be the
most appropriate management at this time? (check one)
A. Evacuation of the pleural space. This patient has been
clinically stable despite losing what appears to be a fair
amount of blood into his pleural space after fracturing
two ribs, a condition referred to as hemothorax. The
treatment of choice in this condition is to remove the
bloody fluid and re-expand the associated lung. This
therapy is felt to decrease any ongoing blood loss by
having the lung pleura put a direct barrier over the site
that is bleeding. It also prevents the development of
empyema or fibrosis, which could occur if the blood
were to remain.
A. Evacuation of the pleural space
B. Prophylactic antibiotics
C. Open fixation of the ribs with control of bleeding
D. Symptomatic treatment and close follow-up
E. Use of a rib binder for 2-3 weeks
126.
=======================================================
Respiratory Board Review Questions 04
=======================================================
In adults, which one of the following is the most likely cause of
chronic, unilateral nasal obstruction? (check one)
A. Nasal septal deviation
B. Foreign body impaction
C. Allergic rhinitis
D. Adenoidal hypertrophy
127.
=======================================================
Special Sensory Board Review Questions
=======================================================
You are the team physician for the local high-school swim team. Over
the past week, seven members of the team have developed both
folliculitis and outer ear infections. You suspect bacterial
contamination of the swimming pool. Which one of the following is
the most likely cause? (check one)
A. Streptococcus pneumoniae
B. Pseudomonas aeruginosa
C. Corynebacterium ulcerans
D. Staphylococcus epidermidis
E. Escherichia coli
A. Nasal septal deviation. The most common cause of
nasal obstruction in all age groups is the common cold,
which is classified as mucosal disease. Anatomic
abnormalities, however, are the most frequent cause of
constant unilateral obstruction. Of these, septal
deviation is the most common. Foreign body impaction is
an important, but infrequent, cause of unilateral
obstruction and purulent rhinorrhea. Mucosal disease is
usually bilateral and intermittent. Adenoidal
hypertrophy is the most common tumor or growth to
cause nasal obstruction, followed by nasal polyps, but
both are less frequent than true anatomic causes of
constant obstruction.
B. Pseudomonas aeruginosa. Athletes, including
swimmers, are susceptible to a number of skin
infections. The pH of the external ear is normally acidic.
Continued water exposure raises the pH, creating
conditions for bacterial overgrowth, most often caused
by either Pseudomonas aeruginosa or Staphylococcus
aureus. Swimming pool folliculitis is most often attributed
to colonization of water with P. aeruginosa.
128.
A 2-week-old
female is brought
to the office for a
well child visit. The
physical
examination is
completely normal
except for a
clunking sensation
and feeling of
movement when
adducting the hip
and applying
posterior pressure.
Which one of the
following would
be the most
appropriate next
step? (check one)
A. Referral for
orthopedic
consultation
B. Reassurance
that the problem
resolves
spontaneously in
90% of cases, and
follow-up in 2
weeks
C. Triple diapering
and follow-up in 2
weeks
D. A radiograph of
the pelvis
A. Referral for orthopedic consultation. Developmental dysplasia of the hip encompasses both subluxation and
dislocation of the newborn hip, as well as anatomic abnormalities. It is more common in firstborns, females,
breech presentations, oligohydramnios, and patients with a family history of developmental dysplasia. Experts
are divided with regard to whether hip subluxation can be merely observed during the newborn period, but if
there is any question of a hip problem on examination by 2 weeks of age, the recommendation is to refer to a
specialist for further testing and treatment. Studies show that these problems disappear by 1 week of age in
60% of cases, and by 2 months of age in 90% of cases. Triple diapering should not be used because it puts the
hip joint in the wrong position and may aggravate the problem. Plain radiographs may be helpful after 4-6
months of age, but prior to that time the ossification centers are too immature to be seen. Because the condition
can be difficult to diagnose, and can result in significant problems, the current recommendation is to treat all
children with developmental dysplasia of the hip. Closed reduction and immobilization in a Pavlik harness, with
ultrasonography of the hip to ensure proper positioning, is the treatment of choice until 6 months of age. The
American Academy of Pediatrics recommends ultrasound screening at 6 weeks for breech girls, breech boys
(optional), and girls with a positive family history of developmental dysplasia of the hip. Other countries have
recommended universal screening, but a review of the literature has not shown that the benefits of early
diagnosis through universal screening outweigh the risks and potential problems of overtreating.
129.
A 2-year-old child stumbles, but his mother
keeps him from falling by pulling up on his
right hand. An hour later the child refuses
to use his right arm and cries when his
mother tries to move it. The most likely
diagnosis is (check one)
C. subluxation of the head of the radius. "Nursemaid's elbow" is one of the most
common injuries in children under 5 years of age. It occurs when the child's hand is
suddenly jerked up, forcing the elbow into extension and causing the radial head to
slip out from the annular ligament.
A. dislocation of the ulna
B. dislocation of the olecranon epiphysis
C. subluxation of the head of the radius
D. subluxation of the head of the ulna
E. anterior dislocation of the humeral head
130.
A 2-year-old female is brought to the
emergency department with a 2-day
history of fever and increasing redness on
the left forearm. She is otherwise healthy.
On examination her temperature is 39.9C
(103.8F), pulse rate 140 beats/min, and
respiratory rate 42/min. She is irritable,
and the left forearm has a 4-cm
erythematous, warm, tender area, with a
fluctuant area centrally. Her WBC count is
21,000/mm3 (N 4300-1 3 0,800), with 14%
immature bands.
In addition to incision and drainage, which
one of the following is the best initial
treatment in this patient?
(check one)
A. Intravenous vancomycin. This patient has systemic symptoms that suggest a severe
underlying infection. Community-acquired methicillin-resistant Staphylococcus
aureus (CA-MRSA) should be considered the cause of this type of infection until
definitive cultures are obtained. CA-MRSA can cause aggressive infections in children,
especially in the skin and soft tissue. Incision and drainage of the abscess is necessary
for treatment. In a severe infection, vancomycin should be started initially until
culture and sensitivities are available (SORB).
A. Intravenous vancomycin
B. Intravenous ampicillin/sulbactam
(Unasyn)
C. Intravenous nafcillin
D. Intravenous clindamycin (Cleocin)
E. No antibiotics
131.
A 2-year-old Hispanic male with a 3-day
history of nasal congestion presents with a
barking cough and hoarseness. He is
afebrile. The examination reveals
tachypnea, inspiratory and expiratory
stridor, noticeable intercostal retractions,
and good color.
Which one of the following is indicated?
(check one)
A. Albuterol syrup and the use of a
humidifier
B. Inhaled albuterol (Proventil, Ventolin)
C. Aerosolized epinephrine and
intramuscular dexamethasone
D. Visualization of the epiglottis, and
ceftriaxone (Rocephin)
C. Aerosolized epinephrine and intramuscular dexamethasone. This child has a
history and physical findings typical of viral laryngotracheobronchitis, or croup. In
rare instances, this illness can be complicated by critical upper airway obstruction. The
symptoms of cough, respiratory stridor, and distress result from edema of the
subglottic portion of the upper airway. Humidification of inspired air is sometimes
beneficial, but the child should not be sent home until improvement is demonstrated.
Because this child has stridor and intercostal retractions, aerosolized epinephrine is
indicated, along with intramuscular dexamethasone, and hospitalization may be
required for observation and continued treatment. Antibiotics do not have a role in
the treatment of viral croup, and attempted visualization of the epiglottis is not
indicated since it will increase the child's anxiety and worsen the symptoms.
132.
A 2-year-old white female is brought to
your office by her parents, who are
concerned about the child's "flat feet." On
evaluation, the child's feet are flat with
weight-bearing, but with toe standing and
with sitting the arch appears. You would:
(check one)
A. Reassure the parents. Flexible flat feet as described are not pathologic unless
painful, which is uncommon. Flexibility of the flat foot is determined by appearance of
an arch when the feet are not bearing weight. No treatment is indicated for painless
flexible flatfoot. Spontaneous correction is usually expected within 1 year of walking.
A. Reassure the parents
B. Recommend orthotics
C. Recommend surgery
D. Recommend casting
E. Recommend foot-stretching exercises
133.
A 2-year-old white male is seen for a well
child visit. His mother is concerned
because he is not yet able to walk. The
routine physical examination, including an
orthopedic evaluation, is unremarkable.
Speech and other developmental
landmarks seem normal for his age.
Which one of the following tests would be
most appropriate? (check one)
A. A TSH level
B. Random urine for aminoaciduria
C. Phenylketonuria screening
D. A serum creatine kinase level
E. Chromosome analysis
D. A serum creatine kinase level. The diagnosis of Duchenne muscular dystrophy, the
most common neuromuscular disorder of childhood, is usually not made until the
affected individual presents with an established gait abnormality at the age of 4-5
years. By then, parents unaware of the X-linked inheritance may have had additional
children who would also be at risk.
The disease can be diagnosed earlier by testing for elevated creatine kinase in boys
who are slow to walk. The mean age for walking in affected boys is 17.2 months,
whereas over 75% of developmentally normal children in the United States walk by
13.5 months. Massive elevation of creatine kinase (CK) from 20 to 100 times normal
occurs in every young infant with the disease. Early detection allows appropriate
genetic counseling regarding future pregnancies.
Hypothyroidism and phenylketonuria could present as delayed walking. However,
these diseases cause significant mental retardation and would be associated with
global developmental delay. Furthermore, these disorders are now diagnosed in the
neonatal period by routine screening. Disorders of amino acid metabolism present in
the newborn period with failure to thrive, poor feeding, and lethargy. Gross
chromosomal abnormalities would usually be incompatible with a normal physical
examination at 18 months of age.
134.
A 3-day-old female developed a rash 1 day ago
that has continued to progress and spread. The
infant was born at term after an uncomplicated
pregnancy and delivery to a healthy mother
following excellent prenatal care. The infant was
discharged 2 days ago in good health. She does
not appear to be irritable or in distress, and she
is afebrile and feeding well. On examination,
abnormal findings are confined to the skin,
including her face, trunk, and proximal
extremities, which have macules, papules, and
pustules that are all 2-3 mm in diameter. Her
palms and soles are spared. A stain of a pustular
smear shows numerous eosinophils.
D. Erythema toxicum neonatorum. This infant has a typical presentation of
erythema toxicum neonatorum. Staphylococcal pyoderma is vesicular and the
stain of the vesicle content shows polymorphonuclear leukocytes and clusters of
gram-positive bacteria. Because the mother is healthy and the infant shows no
evidence of being otherwise ill, systemic infections such as herpes are unlikely.
Acne neonatorum consists of closed comedones on the forehead, nose, and
cheeks. Rocky Mountain spotted fever is a tickborne disease that does not need
to be considered in a child who is not at risk.
Which one of the following is the most likely
diagnosis? (check one)
A. Staphylococcal pyoderma
B. Herpes simplex
C. Acne neonatorum
D. Erythema toxicum neonatorum
E. Rocky Mountain spotted fever
135.
A 3-week-old infant is brought to your office
with a fever. He has a rectal temperature of
38.3C (101.0F), but does not appear toxic. The
remainder of the examination is within normal
limits. Which one of the following would be the
most appropriate management for this patient?
(check one)
A. Admit to the hospital; obtain urine, blood,
and CSF cultures; and start intravenous
antibiotics
B. Admit to the hospital and treat for herpes
simplex virus infection
C. Follow up in the office in 24 hours and admit
to the hospital if not improved
D. Order a CBC and urinalysis with urine culture,
and send the patient home if the results are
normal
A. Admit to the hospital; obtain urine, blood, and CSF cultures; and start
intravenous antibiotics. Any child younger than 29 days old with a fever and any
child who appears toxic, regardless of age, should undergo a complete sepsis
workup and be admitted to the hospital for observation until culture results are
known or the source of the fever is found and treated (SOR A). Observation
only, with close follow-up, is recommended for nontoxic infants 3-36 months of
age with a temperature <39.0C (102.2F) (SOR B). Children 29-90 days old who
appear to be nontoxic and have negative screening laboratory studies,
including a CBC and urinalysis, can be sent home with precautions and with
follow-up in 24 hours (SOR B). Testing for neonatal herpes simplex virus
infection should be considered in patients with risk factors, including maternal
infection at the time of delivery, use of fetal scalp electrodes, vaginal delivery,
cerebrospinal fluid pleocytosis, or herpetic lesions. Testing also should be
considered when a child does not respond to antibiotics (SOR C).
136.
A 3-week-old male
is brought to your
office because of a
sudden onset of
bilious vomiting of
several hours
duration. He is
irritable and
refuses to
breastfeed, but
stools have been
normal. He was
delivered at term
after a normal
pregnancy, and has
had no health
problems to date. A
physical
examination shows
a fussy child with a
distended
abdomen.
Radiography of the
abdomen shows a
double bubble
sign. Which one of
the following is the
most likely
diagnosis? (check
one)
A. Infantile colic
B. Necrotizing
enterocolitis
C. Hypertrophic
pyloric stenosis
D. Intussusception
E. Midgut volvulus
E. Midgut volvulus. Volvulus may present in one of three ways: as a sudden onset of bilious vomiting and
abdominal pain in a neonate; as a history of feeding problems with bilious vomiting that appears to be a bowel
obstruction; or less commonly, as failure to thrive with severe feeding intolerance. The classic finding on
abdominal plain films is the double bubble sign, which shows a paucity of gas (airless abdomen) with two air
bubbles, one in the stomach and one in the duodenum. However, the plain film can be entirely normal. The
upper gastrointestinal contrast study is considered the gold standard for diagnosing volvulus. Infantile colic
usually begins during the second week of life and typically occurs in the evening. It is characterized by
screaming episodes and a distended or tight abdomen. Its etiology has yet to be determined. There are no
abnormalities on physical examination and ancillary studies, and symptoms usually resolve spontaneously
around 12 weeks of age. Necrotizing enterocolitis is typically seen in the distressed neonate in the intensive-care
nursery, but it may occasionally be seen in the healthy neonate within the first 2 weeks of life. The child will
appear ill, with symptoms including irritability, poor feeding, a distended abdomen, and bloody stools.
Abdominal plain films will show pneumatosis intestinalis, caused by gas in the intestinal wall, which is diagnostic
of the condition. Hypertrophic pyloric stenosis is a narrowing of the pyloric canal caused by hypertrophy of the
musculature. It usually presents during the third to fifth weeks of life. Projectile vomiting after feeding, weight
loss, and dehydration are common. The vomitus is always nonbilious, because the obstruction is proximal to the
duodenum. If a small olive-size mass cannot be felt in the right upper or middle quadrant, ultrasonography will
confirm the diagnosis. Intussusception is seen most frequently between the ages of 3 months and 5 years, with
60% of cases occurring in the first year and a peak incidence at 6-11 months of age. The disorder occurs
predominantly in males. The classic triad of intermittent colicky abdominal pain, vomiting, and bloody, mucous
stools is encountered in only 20%-40% of cases. At least two of these findings will be present in approximately
60% of patients. The abdomen may be distended and tender, and there may be an elongated mass in the right
upper or lower quadrants. Rectal examination may reveal either occult blood or frankly bloody, foul-smelling
stool, classically described as currant jelly. An air enema using fluoroscopic guidance is useful for both diagnosis
and treatment.
137.
A 3-year-old female is brought to
your office for evaluation of mild
intoeing. The child's patellae face
forward, and her feet point slightly
inward. Which one of the following
would be most appropriate? (check
one)
A. Reassurance. Intoeing, as described, is usually caused by internal tibial torsion. This
problem is believed to be caused by sleeping in the prone position, and sitting on the feet. In
90% of cases, internal tibial torsion gradually resolves without intervention by the age of 8.
Avoiding prone sleeping enhances resolution of the problem. Night splints, orthotics, and
shoe wedges are ineffective. Surgery (osteotomy) has been associated with a high
complication rate, and is therefore not recommended in mild cases before the age of 8.
A. Reassurance
B. Foot stretching exercises
C. Use of orthotics
D. Use of night splints
E. Surgery
138.
A 3-year-old female is brought to
your office with a 3-hour history of
skin lesions that are prominent,
warm, papular, and serpiginous (see
Figure 3). Which one of the following
is the most likely cause of these
lesions?
(check one)
A. Heredity
B. Physical abuse
C. Infection
D. A topical agent
E. An oral medication
E. An oral medication. Acute urticaria occurs when an allergen activates mast cells in the skin,
and is commonly caused by oral
and parenteral drugs, food, and, less frequently, infections. Topical agents and physical
abuse are unlikely
to present in this manner, and hereditary angioedema is more a systemic illness than a skin
disorder.
139.
A 3-year-old female presents with
urinary frequency, dysuria, and fever
to 39.0 degrees C (102.2 degrees F).
She denies nausea, vomiting, fever,
and flank pain. There is no prior
history of urinary infection and no
family history of urinary tract
abnormalities. Urethral
catheterization reveals bacteriuria
and a urine culture reveals >100,000
colony-forming units of Escherichia
coli. She is started on appropriate
antibiotic therapy. Evaluation to
rule out anatomic abnormalities
should include: (check one)
E. Renal ultrasonography and VCUG for this primary episode of infection. In the first few
months of life, the incidence of urinary tract infection (UTI) in boys is higher than that of girls.
However, after that time period, UTIs are much more common in females, with the peak
incidence in the 2- to 3-year age range. The clinical presentation of UTI in children is similar to
that of adults, including dysuria, hematuria, frequency, incontinence, suprapubic
tenderness, and low-grade fever. Upper tract infection is suggested by high fever, nausea,
vomiting, flank pain, and lethargy. All children who have a culture-documented UTI should
undergo evaluation of the anatomy of the urinary tract. This is due to the fact that children
who are at most risk for renal parenchymal damage are those with an anatomic defect. In
general, studies to evaluate both the upper and lower tract are recommended. Children
under the age of 5 years with a UTI, any child with a UTI and a fever, school-aged girls who
have had two or more UTIs, and any boy with a UTI should have a voiding cystourethrogram
(VCUG) to evaluate for vesiculoureteral reflux and renal ultrasonography to evaluate the
kidneys. Cystoscopy and retrograde pyelography are rarely indicated in the workup.
A. Renal ultrasonography only if she
has recurrent infections
B. Renal ultrasonography and
voiding cystourethrography (VCUG)
only if she has recurrent infections
C. Renal ultrasonography and
cystoscopy only if she has recurrent
infections
D. Renal ultrasonography for this
primary episode of infection
E. Renal ultrasonography and VCUG
for this primary episode of infection
140.
A 3-year-old male is brought to the
emergency department by his
parents, who report seeing him
swallow a handful of adult
ibuprofen tablets 20 minutes ago.
Which one of the following would
be the most appropriate initial
management of this patient? (check
one)
A. Oral ipecac
B. Oral activated charcoal
C. Gastric lavage
D. Whole-bowel irrigation
E. Close observation
B. Oral activated charcoal. A single dose of activated charcoal is the decontamination
treatment of choice for most medication ingestions. It should be used within 1 hour of
ingestion of a potentially toxic amount of medication (SOR C). Gastric lavage, cathartics, or
whole bowel irrigation is best for ingestion of medications that are poorly absorbed by
activated charcoal (iron, lithium) or medications in sustained-release or enteric-coated
formulations. Ipecac has no role in home use or in the health care setting (SOR C).
141.
A 3-year-old male is brought to your office because
of ear pain. On examination you find a round,
plastic bead in the lower third of the ear canal close
to the tympanic membrane. You restrain the child
and are unable to remove the object despite several
attempts, first using water irrigation and then fastacting glue on an applicator. Which one of the
following is the best option for removal? (check
one)
B. Referral for removal under anesthesia. After several unsuccessful
attempts to remove an object deep in the ear canal of an uncooperative
child, it is best to refer the patient to an otolaryngologist for removal under
anesthesia. Additional attempts are very unlikely to succeed, especially with
the techniques listed. A loop curette cannot be safely placed behind a
foreign body that is close to the tympanic membrane. A round, hard object
cannot be grasped with forceps. Acetone can be used to dissolve Styrofoam
foreign bodies, but it would not dissolve a plastic bead.
A. A plastic loop curette through an otoscope
B. Referral for removal under anesthesia
C. Grasping with forceps
D. Applying acetone to dissolve the object
142.
A 3-year-old male is brought to your office by his
parents because they are concerned about three
"spells" he has had in the past month. In each case,
the child started crying when he was prevented by a
parent from doing something he wished to do.
While crying, he suddenly stopped breathing and
his face and lips began to turn blue. After 30-45
seconds he resumed crying, his color returned to
normal, and he showed no evidence of impairment.
A physical examination today is normal and the
child is developmentally appropriate for his age. A
recent hemoglobin level was in the normal range.
Which one of the following should you do now?
(check one)
B. Reassure the parents that this is a benign condition and will resolve as the
child gets older. This child is experiencing simple breath-holding spells, a
relatively common and benign condition that usually begins in children
between the ages of 6 months and 6 years. The cause is uncertain but seems
to be related to overactivity of the autonomic nervous system in association
with emotions such as fear, anger, and frustration. The episodes are selflimited and may be associated with pallor, cyanosis, and loss of conciousness
if prolonged. There may be an association with iron deficiency anemia, but
this child had a recent normal hemoglobin level.
These events are not volitional, so disciplinary methods are neither effective
nor warranted. While children may experience a loss of consciousness and
even exhibit some twitching behavior, the episodes are not seizures so
neither EEG evaluation nor anticonvulsant therapy is indicated. No
additional laboratory studies are indicated. Parents should be reassured
that the episodes are benign and will resolve without treatment.
A. Teach the parents age-appropriate disciplinary
procedures to implement when the child behaves in
this manner
B. Reassure the parents that this is a benign
condition and will resolve as the child gets older
C. Order an EEG
D. Obtain appropriate laboratory studies to confirm
the most likely diagnosis
E. Initiate treatment with valproic acid (Depakene)
143.
A 3-year-old male is carried into the office by his
mother. Yesterday evening he began complaining of
pain around his right hip. Today he has a
temperature of 37.6C (99.7F), cries when bearing
weight on his right leg, and will not allow the leg to
be moved in any direction. A radiograph of the hip
is normal.
Which one of the following would be most
appropriate at this time?
(check one)
A. A CBC and an erythrocyte sedimentation rate
B. A serum antinuclear antibody level
C. Ultrasonography of the hip
D. MRI of the hip
E. In-office aspiration of the hip
A. A CBC and an erythrocyte sedimentation rate. This presentation is typical
of either transient synovitis or septic arthritis of the hip. Because the
conditions have very different treatment regimens and outcomes, it is
important to differentiate the two. It is recommended that after plain films,
the first studies to be performed should be a CBC and an erythrocyte
sedimentation rate (ESR). Studies have shown that septic arthritis should be
considered highly likely in a child who has a fever over 38.7C (101.7F),
refuses to bear weight on the leg, has a WBC count >12,000 cells/mm , and
has an ESR >40 mm/hr. If several or all of these conditions exist, aspiration 3
of the hip guided by ultrasonography or fluoroscopy should be performed
by an experienced practitioner. MRI may be helpful in cases that are
unclear based on standard data, or if other etiologies need to be excluded.
144.
A 3-year-old male presents with a 3-day history of
fever and refusal to eat. Today his parents noted some
sores just inside his lips. No one else in the family is ill,
and he has no significant past medical history. He is
up-to-date on his immunizations and has no known
allergies.
On examination, positive findings include a
temperature of 38.9C (102.0F) rectally, irritability,
and ulcers on the oral buccal mucosa, soft palate,
tongue, and lips. He also has cervical
lymphadenopathy. The remainder of the physical
examination is normal. The child is alert and has no
skin lesions or meningeal signs.
Which one of the following would be the most
appropriate treatment? (check one)
A. Ceftriaxone (Rocephin) intramuscularly
B. Nystatin oral suspension
C. Amoxicillin suspension
D. Acyclovir (Zovirax) suspension
E. Methotrexate (Trexall)
145.
A 3-year-old male was treated for acute otitis media
last month. His mother brings him in for follow-up
because she believes his hearing has not been normal
since then. He attends day care and has had several
upper respiratory infections. On examination the
tympanic membranes are not inflamed, but the
membrane is retracted on the right side. An office
tympanogram shows a normal peak (type A) on the
left side, but a flat tracing (type B) on the right side.
Which one of the following would be the most
appropriate recommendation? (check one)
A. Audiometry
B. Observation with follow-up
C. An antihistamine/decongestant combination
D. Intranasal corticosteroids
E. Systemic corticosteroids
D. Acyclovir (Zovirax) suspension. The history and physical findings in this
patient are consistent with gingivostomatitis due to a primary or initial
infection with herpes simplex virus type 1 (HSV-1). There are no additional
findings to suggest other diagnoses such as aphthous ulcers, Behet's
syndrome, or herpangina (coxsackievirus).
After a primary HSV-1 infection with oral involvement, the virus invades
the neurons and replicates in the trigeminal sensory ganglion, leading to
recurrent herpes labialis and erythema multiforme, among other things.
Although some clinicians might choose to use oral anesthetics for
symptomatic care, it is not a specific therapy.
Antibiotics are not useful for the treatment of herpetic gingivostomatitis
and could confuse the clinical picture should this child develop erythema
multiforme, which occurs with HSV-1 infections. An orally applied
corticosteroid is not specific treatment, but some might try it for
symptomatic relief. An immunosuppressant is sometimes used for the
treatment of Behet's syndrome, but this patient's findings are not
consistent with that diagnosis. Therefore, the only specific treatment listed
is acyclovir suspension, which has been shown to lead to earlier resolution
of fever, oral lesions, and difficulties with eating and drinking. It also
reduces viral shedding from 5 days to 1 day (SOR B).
B. Observation with follow-up. This patient has unilateral serous otitis and
is unlikely to have delayed language from decreased hearing on one side.
The patient should be observed for now. Hearing loss of longer than 3
months may indicate a need for tympanostomy tubes. Surgical treatment
has been shown to be helpful, but should be reserved for patients with
chronic effusion. Audiometry is not needed to make a decision about
surgery at this point. The mother's judgment is likely correct about his
current hearing loss, so a hearing test most likely would not add any useful
information. Numerous studies have shown that all medical treatments for
serous otitis are ineffective, including antihistamine and decongestant
therapy, and corticosteroids by any route.
146.
A 3-year-old white female is brought to the emergency department
with an acute onset of epistaxis. The child, who has a history of good
health, is brought in by her recently-divorced mother, a registered
nurse. The mother appears relatively unconcerned about the child's
illness, but otherwise is friendly and interacts appropriately with the
health care team evaluating the child. The child's vital signs are
normal, but she is bleeding mildly from both nostrils and there are
areas of ecchymosis. Laboratory Findings Hemoglobin 12.3 g/dL (N
11.5-15.0) Hematocrit 36% (N 32-42) WBC count 4500/mm3 (N 600015,000) Platelets 235,000/mm3 (N >50,000) Prothrombin time 40.0 sec
(N 11.0-15.0) Partial thromboplastin time 30 sec (N 24-36) INR 3.9 sec
(N 2.0-3.0) ALT (SGPT) 18 IU/L (N 7-35) AST (SGOT) 16 IU/L (N 15-60)
Bilirubin 0.8 mg/dL (N 0.3-1.2) You hospitalize the child for
observation and further testing. Her bleeding subsides in several
hours, no new skin lesions develop, and her PT/INR decreases to 32
sec/3.0, 23 sec/2.1, and 15 sec/1.4 on subsequent days. You suspect
that the child's condition is due to: (check one)
C. Munchausen syndrome by proxy. The patient exhibits
signs of a moderate bleeding diathesis. Her prothrombin
time (PT) elevation, without evidence of hepatocellular
damage or hepatic dysfunction, is highly suspicious for
warfarin ingestion. The normalization of the PT under
observation in a hospital setting is consistent with this
suspicion. Although accidental poisoning is a possibility,
the mother's affect is highly suspicious for Munchausen
syndrome by proxy. The fact that her mother is a healthcare worker and develops a close and appropriate
relationship with the health-care team is consistent with
this diagnosis. Acetaminophen toxicity of this degree
would likely produce transaminase and bilirubin
elevations, as well as mental status changes.
Antiphospholipid syndrome produces a hypercoagulable
state. Henoch-Schonlein purpura presents with purpura,
joint pain, abdominal pain, and a normal PT. Traumatic
injury would not result in PT elevations.
A. Acetaminophen overdose
B. Antiphospholipid syndrome with lupus anticoagulant
C. Munchausen syndrome by proxy
D. Henoch-Schnlein purpura
E. Traumatic injury (child abuse)
147.
A 4-month-old white male in respiratory distress is brought to the
emergency department. On examination, heart sounds include a
grade 4/6 pansystolic murmur, best heard at the lower left sternal
border. He is acyanotic. A chest radiograph shows an enlarged heart
and increased pulmonary vascular markings, and an EKG shows
combined ventricular hypertrophy.
C. ventricular septal defect. Ventricular septal defect
causes overload of both ventricles, since the blood is
shunted left to right. The murmur is harsh and
holosystolic, generally heard best at the lower left sternal
border. As the volume of the shunting increases, cardiac
enlargement and increased pulmonary vascular markings
can be seen on a chest radiograph.
Of the following, the most likely diagnosis is: (check one)
A. hypoplastic left heart syndrome (aortic valve atresia)
B. transposition of the great vessels
C. ventricular septal defect
D. tetralogy of Fallot
E. patent ductus arteriosus
Hypoplastic left heart syndrome would be manifested by
near-obliteration of the left ventricle on the EKG and chest
radiograph, and the infant would be cyanotic.
Transposition of the great vessels would cause AV
conduction defects and single-sided hypertrophy on the
EKG. The chest radiograph would show a straight
shoulder on the left heart border where the aorta was
directed to the right. Tetralogy of Fallot causes cyanosis
and right ventricular enlargement. The murmur of patent
ductus arteriosus is continuous, best heard below the left
clavicle. The EKG shows left atrial and ventricular
enlargement.
148.
A 4-year-old female has had three urinary
tract infections in the past 6 months. She
complains of difficulty with urination and on
examination is noted to have labial adhesions
that have resulted in near closing of the
introitus. Which one of the following is the
most appropriate management? (check one)
C. Application of estrogen cream to the site. The etiology of prepubertal labial
adhesions is idiopathic. The adhesions may be partial or complete; in some cases
only a small pinhole orifice may be seen that allows urine to exit from the fused
labia. This problem may be asymptomatic, but the patient may also have a pulling
sensation, difficulty with voiding, recurrent urinary tract infections, or vaginitis. If
there is enough labial fusion to interfere with urination, treatment should be
undertaken. The use of topical estrogen cream twice daily at the point of the
midline fusion will usually result in resolution of the problem.
A. No treatment at this time
B. Reporting your suspicion of child abuse to
the appropriate authorities
C. Application of estrogen cream to the site
D. Gentle insertion of progressively larger
dilators over a period of several days
E. Referral to a gynecologist for surgical
correction
149.
A 4-year-old Hispanic female has been
discovered to have a congenital hearing loss.
Her mother is an 18-year-old migrant farm
worker who is currently at 8 weeks' gestation
with her second pregnancy. The mother has
been found to have cervical dysplasia on her
current Papanicolaou (Pap) smear and has
also tested positive for Chlamydia. The most
likely cause of this child's hearing loss is:
(check one)
E. Cytomegalovirus. Cytomegalovirus (CMV) is the most common congenital infection
and occurs in up to 2.2% of newborns. It is the leading cause of congenital hearing
loss. The virus is transmitted by contact with infected blood, urine, or saliva, or by
sexual contact. Risk factors for CMV include low socioeconomic status, birth outside
North America, first pregnancy prior to age 15, a history of cervical dysplasia, and a
history of sexually transmitted diseases. Infection can be primary or a reactivation
of a previous infection. While the greatest risk of infection is during the third
trimester, those occurring in the first trimester are the most dangerous to the fetus.
A. Human parvovirus B19
B. Varicella zoster virus
C. Herpes simplex virus
D. Toxoplasmosis
E. Cytomegalovirus
150.
A 4-year-old is brought to the emergency
department with abdominal pain and is
noted to have 3+ proteinuria on a dipstick.
Three days later the pain has resolved
spontaneously, and a repeat urinalysis in
your office shows 2+ proteinuria with normal
findings on microscopic examination. A
metabolic panel, including creatinine and
total protein, is also normal.
Which one of the following would be most
appropriate at this point? (check one)
A. Renal ultrasonography
B. A spot first morning urine
protein/creatinine ratio
C. An antinuclear antibody and complement
panel
D. Referral to a nephrologist
B. A spot first morning urine protein/creatinine ratio. When proteinuria is noted on
a dipstick and the history, examination, full urinalysis, and serum studies suggest
no obvious underlying problem or renal insufficiency, a urine protein/creatinine
ratio is recommended. This test correlates well with 24-hour urine protein, which is
particularly difficult to collect in a younger patient. Renal ultrasonography is
appropriate once renal insufficiency or nephritis is established. If pathogenic
proteinuria is confirmed, an antinuclear antibody and/or complement panel may
be indicated. A nephrology referral is not necessary until the presence of kidney
disease or proteinuria from a cause other than benign postural proteinuria is
confirmed.
151.
A 4-year-old male is brought to your office for evaluation of fever,
coryza, and cough. On examination, the child appears mildly ill
but in no respiratory distress. His temperature is 37.4C (99.3F)
and other vital signs are within the normal range. An HEENT
examination is significant only for light yellow rhinorrhea and
reddened nasal mucous membranes. Lung auscultation reveals
good air flow with a few coarse upper airway sounds. While
performing the examination you note multiple red welts and
superficial abrasions scattered on the chest and upper back.
When you question the parents, they tell you the marks are
where "the sickness is leaving his body," and were produced by
rubbing the skin with a coin.
This traditional healing custom is practiced principally by people
from which geographic region? (check one)
B. Southeast Asia. Coin rubbing is a traditional healing custom
practiced primarily in east Asian countries such as Cambodia,
Korea, China, and Vietnam. The belief is that one's illness must
be drawn out of the body, and the red marks produced by
rubbing the skin with a coin are evidence of the body's
"release" of the illness. These marks may be confused with
abuse, trauma from some other source, or an unusual
manifestation of the illness itself.
A. Sub-Saharan Africa
B. Southeast Asia
C. The Middle East
D. Caribbean islands
E. Andean South America
152.
A 4-year-old male presents with a 3-day history of sores on his
right leg. The sores began as small red papules but have
progressed in size and now are crusting and weeping. Otherwise
he is in good health and is up to date with immunizations.
On examination he has three lesions on the right anterior lower
leg that are 0.5-1.5 cm in diameter, with red bases and honeycolored crusts. There is no regional lymphangitis or
lymphadenitis.
Which one of the following is the preferred first-line therapy?
(check one)
A. Oral erythromycin (Erythrocin)
B. Oral penicillin V
C. Topical hexachlorophene (pHisoHex)
D. Topical mupirocin (Bactroban)
D. Topical mupirocin (Bactroban). The lesions described are
nonbullous impetigo, due to either Staphylococcus aureus or
Streptococcus pyogenes. Topical antibiotics, such as mupirocin,
but not compounds containing neomycin, are the preferred
first-line therapy for impetigo involving a limited area. Oral
antibiotics are widely used, based on expert opinion and
traditional practice, but are usually reserved for patients with
more extensive impetigo or with systemic symptoms or signs.
Penicillin V and hexachlorophene have both been shown to
be no more effective than placebo. Topical antibiotics have
been shown to be as effective as erythromycin, which has a
common adverse effect of nausea.
153.
A 4-year-old white female is
brought to your office by her
mother, who reports that the
child recently developed a foulsmelling vaginal discharge. After
an appropriate history and
general examination, you
determine that a genital
examination is necessary. Which
one of the following positions is
most likely to allow for
visualization of the child's vagina
and cervix without
instrumentation? (check one)
D. The knee-chest position on an examination table. The knee-chest position has been found to
allow for visualization of the vagina and cervix of a prepubertal child after 2 years of age without
instrumentation. The vagina is filled with air when the child is in the knee-chest position,
facilitating inspection. An assistant holds the child's buttocks apart and the child is asked to relax
her abdominal muscles and take a few deep breaths. With these preliminary steps, the vaginal
orifice opens and the short vaginal canal fills with air. A bright light will help to illuminate the
prepubertal child's vagina and cervix. Inspection of genitalia (where examination of the vaginal
canal and cervix are not indicated) during a general physical examination need not be in the
knee-chest position. In the young child (usually less than 2 years of age), examination is best
done with the child lying supine in the mother's lap. For the older prepubertal child,
examination is best done with the child lying supine with the knees spread apart on the
examination table. The other positions listed are not helpful or recommended when examining
the genital area of a prepubertal child.
A. Supine in the mother's lap
B. The left lateral decubitus
position on an examination table
C. Trendelenburg's position on an
examination table
D. The knee-chest position on an
examination table
E. Supine with the knees spread
apart on an examination table
154.
A 4-year-old white male is
brought to your office because he
has had a low-grade fever and
decreased oral intake over the
past few days. On examination
you note shallow oral ulcerations
confined to the posterior
pharynx. Which one of the
following is the most likely
diagnosis? (check one)
A. Herpangina
B. Herpes
C. Mononucleosis
D. Roseola infantum
E. Rubella
A. Herpangina. Herpangina is a febrile disease caused by coxsackieviruses and echoviruses.
Vesicles and subsequent ulcers develop in the posterior pharyngeal area (SOR C). Herpes
infection causes a gingivostomatitis that involves the anterior mouth. Mononucleosis may be
associated with petechiae of the soft palate, but does not usually cause pharyngeal lesions. The
exanthem in roseola usually coincides with defervescence. Mucosal involvement is not noted.
Rubella may cause an enanthem of pinpoint petechiae involving the soft palate (Forschheimer
spots), but not the pharynx.
155.
A 4-year-old white male is brought to your
office in late August. His mother tells you
that over the past few days he has
developed a rash on his hands and sores in
his mouth. On examination you note a
vesicular exanthem on his hands, with
lesions ranging from 3 to 6 mm in diameter.
The oral lesions are shallow, whitish, 4- to 8mm ulcerations distributed randomly over
the hard palate, buccal mucosa, gingiva,
tongue, lips, and pharynx. Except for a
temperature of 37.4C (99.3F), the
remainder of the examination is normal.
The most likely diagnosis is (check one)
B. hand, foot, and mouth disease. Hand, foot, and mouth disease is a mild infection
occurring in young children, and is caused by coxsackievirus A16, or occasionally by
other strains of coxsackie- or enterovirus. In addition to the oral lesions, vesicular
lesions may occur on the feet and nonvesicular lesions may occur on the buttocks. A
low-grade fever may also develop. Herpangina is also caused by coxsackieviruses,
but it is a more severe illness characterized by severe sore throat and vesiculoulcerative lesions limited to the tonsillar pillars, soft palate, and uvula, and occasionally
the posterior oropharynx. Temperatures can range to as high as 41C (106F). The
etiology of aphthous stomatitis is multifactorial, and it may be due to a number of
conditions. Systemic signs, such as fever, are generally absent. Lesions are randomly
distributed. Herpetic gingivostomatitis also causes randomly distributed oral ulcers,
but it is a more severe illness, regularly accompanied by a higher fever, and is
extremely painful. Streptococcal pharyngitis is rarely accompanied by ulceration
except in agranulocytic patients.
A. herpangina
B. hand, foot, and mouth disease
C. aphthous stomatitis
D. herpetic gingivostomatitis
E. streptococcal pharyngitis
156.
A 5-month-old female is brought in with a 1day history of an axillary temperature of
100.6F and mild irritability. Findings are
normal on examination except for a runny
nose and a moderately distorted,
immobile, red right eardrum. There is no
history of recent illness or otitis in the past.
The most appropriate management would
be: (check one)
A. azithromycin (Zithromax) for 5 days
B. amoxicillin for 10 days
C. amoxicillin for 5 days
D. oral decongestants
E. observation and a repeat examination in
2 weeks
B. amoxicillin for 10 days. The treatment for otitis media is evolving. Recommendations
by the American Academy of Family Physicians and the American Academy of
Pediatrics advocate a 10-day course of antibiotics for children under the age of 2
years if the diagnosis is certain. If the diagnosis is not certain and the illness is not
severe, there is an option of observation with follow-up. For children over the age of 2
years, the recommendation is still to treat if the diagnosis is certain, but there is an
option of observation and follow-up if the illness is not severe and follow-up can be
guaranteed.
Amoxicillin is the first-line therapy; the recommended dosage is 80-90 mg/kg/day in
two divided doses, which increases the concentration of amoxicillin in the middle ear
fluid to help with resistant Pneumococcus.
Azithromycin, because of a broader spectrum and potential for causing resistance, is
not considered the treatment of first choice. Treatment regimens ranging from 5 to 7
days are appropriate for selected children over the age of 5 years.
Oral decongestants and antihistamines are not recommended for children with acute
otitis media.
157.
A 5-month-old infant has had several episodes of wheezing,
not clearly related to colds. The pregnancy and delivery
were normal; the infant received phototherapy for 1 day for
hyperbilirubinemia. He had an episode of otitis media 1
month ago. There is no chronic runny nose or strong family
history of asthma. He spits up small amounts of formula
several times a day, but otherwise appears well. His growth
curve is normal. An examination is unremarkable except for
mild wheezing.
Which one of the following is the most likely diagnosis?
(check one)
B. Gastroesophageal reflux. Gastroesophageal reflux is a common
cause of wheezing in infants. At 5 months of age, most infants no
longer spit up several times a day, and this is a major clue that this
child's wheezing may be from the reflux. In addition, there is no
family history of asthma and the wheezing is not related to
infections.Cystic fibrosis is more likely to present with recurrent
infections and failure to thrive than with intermittent wheezing.
A. Benign reactive airway disease of infancy
B. Gastroesophageal reflux
C. Unresolved respiratory syncytial virus infection
D. Early asthma
E. Cystic fibrosis
158.
A 5-year-old African-American male presents with behavior
problems noted in the first 3 months of kindergarten. The
mother explains that the child does not pay attention and
often naps in class. He averages 10 hours of sleep nightly and
is heard snoring frequently. The mother has a history of
attention-deficit disorder and takes atomoxetine
(Strattera). The boy's examination is within normal limits
except for his being in the 25th percentile for weight and
having 3+ tonsillar enlargement. The most reasonable plan
at this point would include which one of the following?
(check one)
B. Polysomnography. Obstructive sleep apnea is increasingly
recognized in children. The peak incidence is in the preschool-age
range of 2-5 years when adenotonsillar tissue is greatest in relation
to airway size. It is associated with obesity in older children.
Common clinical manifestations include snoring with sleep
interruptions and respiratory pauses. Polysomnography is the gold
standard for the diagnosis. Although the child has inattention,
excessive drowsiness is not seen in attention-deficit/hyperactivity
disorder (ADHD) and medications for that condition are not
indicated. None of his symptoms suggests a seizure disorder, so an
EEG would not be helpful.
A. An electroencephalogram
B. Polysomnography
C. Atomoxetine
D. Methylphenidate (Ritalin)
159.
A 5-year-old female is seen for a kindergarten physical and is
noted to be below the 3rd percentile for height. A review of
her chart shows that her height curve has progressively
fallen further below the 3rd percentile over the past year.
She was previously at the 50th percentile for height. The
physical examination is otherwise normal, but your workup
shows that her bone age is delayed.
Of the following conditions, which one is the most likely
cause of her short stature?
(check one)
A. Constitutional growth delay
B. Growth hormone deficiency
C. Genetic short stature
D. Turner syndrome
E. Skeletal dysplasia
B. Growth hormone deficiency. This patient has delayed bone age
coupled with a reduced growth velocity, which suggests an
underlying systemic cause. Growth hormone deficiency is one
possible cause for this. Although bone age can be delayed with
constitutional growth delay, after 24 months of age growth curves
are parallel to the 3rd percentile. Bone age would be normal with
genetic short stature. Patients with Turner syndrome or skeletal
dysplasia have dysmorphic features, and bone age would be
normal.
160.
A 5-year-old male fell
while playing and
complained that his wrist
hurt. The next day he is
brought to your office
because he refuses to use
his arm.
B. A radial fracture. Even though they are the most common fracture in this age group, radial fractures
can be missed byclinicians. The bend in the cortex of the distal radius indicates the fracture. Sometimes
referred to as a buckle or torus fracture, it will heal with almost any choice of treatment. Most clinicians
opt for casting to reduce the chance of reinjury during the first few weeks of healing, but the parents'
preferences in this regard are important. Some pediatric long-bone fractures involve growth plates, and
the results can be indeterminate, requiring either more advanced imaging or comparison views of the
opposite limb.
Which one of the
following best describes
the condition seen in the
radiographs shown in
Figure 9?
(check one)
A. A normal appearance
B. A radial fracture
C. An ulnar fracture
D. A radioulnar fracture
E. Indeterminate result
161.
A 5-year-old male is
scheduled for elective
hernia repair at 11:00 a.m.
Which one of the
following would be the
most appropriate
recommendation? (check
one)
A. No solid food for 8
hours prior to surgery
and clear liquids until 2
hours prior to surgery
B. No solid food 4 hours
prior to surgery and clear
liquids until 2 hours prior
to surgery
C. No solid food after
midnight and nothing by
mouth 8 hours prior to
surgery
D. Nothing by mouth 2
hours prior to surgery
E. Nothing by mouth 8
hours prior to surgery
A. No solid food for 8 hours prior to surgery and clear liquids until 2 hours prior to surgery. Recent
American Society of Anesthesiologists guidelines recommend the following restrictions on diet prior to
surgery for pediatric patients: 8 hours for solid food, 6 hours for formula, 4 hours for breast milk, and 2
hours for clear liquids. These changes have resulted in decreased numbers of canceled cases and
pediatric patients who are less irritable preoperatively and less dehydrated at the time of anesthesia
induction.
162.
A 5-year-old white male has an
itchy lesion on his right foot. He
often plays barefoot in a city park
that is subject to frequent
flooding. The lesion is located
dorsally between the web of his
right third and fourth toes, and
extends toward the ankle. It
measures approximately 3 cm in
length, is erythematous, and has
a serpiginous track. The
remainder of his examination is
within normal limits. Which one
of the following is the most likely
cause of these findings? (check
one)
A. Dog or cat hookworm (Ancylostoma species). This patient has cutaneous larva migrans, a
common condition caused by dog and cat hookworms. Fecal matter deposited on soil or sand
may contain hookworm eggs that hatch and release larvae, which are infective if they penetrate
the skin. Walking barefoot on contaminated ground can lead to infection. Echinococcosis
(hydatid disease) is caused by the cestodes (tapeworms) Echinococcus granulosus and
Echinococcus multilocularis, found in dogs and other canids. It infects humans who ingest eggs
that are shed in the animals feces and results in slow-growing cysts in the liver or lungs, and
occasionally in the brain, bones, or heart. Toxoplasmosis is caused by the protozoa Toxoplasma
gondii, found in cat feces. Humans can contract it from litter boxes or feces-contaminated soil, or
by consuming infected undercooked meat. It can be asymptomatic, or it may cause cervical
lymphadenopathy, a mononucleosis-like illness; it can also lead to a serious congenital infection if
the mother is infected during pregnancy, especially during the first trimester. Toxocariasis due
to Toxocara canis and Toxocara cati causes visceral or ocular larva migrans in children who
ingest soil contaminated with animal feces that contains parasite eggs, often found in areas such
as playgrounds and sandboxes.
A. Dog or cat hookworm
(Ancylostoma species)
B. Dog or other canid tapeworm
(Echinococcus granulosus)
C. Cat protozoa (Toxoplasma
gondii)
D. Dog or cat roundworm
(Toxocara canis or T. mystax)
163.
A 6-month-old Hispanic female
has had itching and irritability for
4-5 weeks. There is a family
history of atopy and asthma.
Physical examination reveals an
excoriated dry rash bilaterally
over the antecubital and
popliteal fossae, as well as some
involvement of the face. In
addition to maintenance therapy
with an emollient, which one of
the following topical
medications would be
appropriate first-line treatment
for flare-ups in this patient?
(check one)
A. A calcineurin inhibitor such as
pimecrolimus (Elidel)
B. An anesthetic
C. An antihistamine
D. An antibiotic
E. A corticosteroid
E. A corticosteroid. This child has atopic dermatitis (eczema). It is manifested by a pruritic rash on
the face and/or extensor surfaces of the arms and/or legs, especially in children. There often is a
family history of atopy or allergies. In addition to the regular use of emollients, the mainstay of
maintenance therapy, topical corticosteroids have been shown to be the best first-line treatment
for flare-ups of atopic dermatitis. Topical calcineurin inhibitors should be second-line treatment
for flare-ups, but are not recommended for use in children under 2 years of age. Antibiotics
should be reserved for the treatment of acutely infected lesions. There is no evidence to support
the use of topical anesthetics or analgesics in the treatment of this disorder.
164.
A 6-month-old white male is
brought to your office because he
has "blisters" in his diaper area. On
examination, you find large bullae
filled with cloudy yellow fluid.
Some of the blisters have ruptured
and the bases are covered with a
thin crust.
Which one of the following is most
appropriate in the management of
this condition? (check one)
D. Trimethoprim/sulfamethoxazole (Bactrim, Septra). Bullous impetigo is a localized skin
infection characterized by large bullae; it is caused by phage group II Staphylococcus aureus.
Cultures of fluid from an intact blister will reveal the causative agent. The lesions are caused
by exfolatin, a local toxin produced by the S. aureus, and develop on intact skin. Complications
are rare, but cellulitis occurs in <10% of cases. Strains of Staphylococcus associated with
impetigo in the U.S. have little or no nephritogenic potential.
Systemic therapy should be used in patients with widespread lesions. With the emergence of
MRSA, trimethoprim/sulfamethoxazole and clindamycin are options for outpatient therapy.
Intravenous vancomycin can be used to treat hospitalized patients with more severe
infections.
A. Rinsing diapers with a vinegar
solution
B. A topical antifungal agent
C. Penicillin
D. Trimethoprim/sulfamethoxazole
(Bactrim, Septra)
165.
A 6-year-old male is brought in for
evaluation by his mother, who is
concerned that he may have
asthma. She reports that he coughs
about 3 days out of the week and
has a nighttime cough
approximately 1 night per week.
There is a family history of eczema
and allergic rhinitis. Which one of
the following would be the
preferred initial treatment for this
patient? (check one)
A. A leukotriene receptor
antagonist such as montelukast
(Singulair)
B. A low-dose inhaled
corticosteroid such as budesonide
(Pulmicort Turbuhaler)
C. A long-acting beta-agonist such
as salmeterol (Serevent)
D. A mast-cell stabilizer such as
cromolyn sodium (Intal)
B. A low-dose inhaled corticosteroid such as budesonide (Pulmicort Turbuhaler). The National
Asthma Education and Prevention Program (NAEPP) updated its recommendations for the
treatment of asthma in 2002. Treatment is based on asthma classification. This child meets the
criteria for mild persistent asthma: symptoms more than 2 times per week but less than once a
day, symptoms less than 2 nights per month, peak expiratory flow (PEF) or FEV1 >80% of
predicted, and a PEF variability of 20%-30%. Asthma controller medications are recommended
for all patients with persistent asthma, and the preferred long-term controller treatment in
mild persistent asthma is a low-dose inhaled corticosteroid. Cromolyn, leukotriene modifiers,
nedocromil, and sustained-release theophylline are alternatives, but are not preferred initial
agents. Quick-acting, quick-relief agents such as short-acting beta-agonists are appropriate for
prompt reversal of acute airflow obstruction.
166.
A 6-year-old white male visits your office with
chief complaints of a recent onset of fever,
bilateral knee and ankle pain, colicky
abdominal pain, and rash. On examination,
his temperature is 38.3 degrees C (101.0
degrees F), and there is a prominent palpable
reddish-brown rash on the buttocks and
thighs. There is pain on motion of his knees
and ankles, and mild diffuse abdominal
tenderness. The stool is positive for occult
blood. Laboratory Findings Hemoglobin 11.0
g/dL (N 11.5-13.5) Hematocrit 33% (N 34-40)
WBCs 14,500/mm3 (N 5500-15,000); 85% segs,
15% lymphs Platelets 345,000/mm3 (N 150,000400,000) Prothrombin time 12 sec (N 11-15)
Which one of the following is the most likely
diagnosis? (check one)
C. Henoch-Schonlein purpura. Henoch-Schonlein purpura typically follows an
upper respiratory tract infection, and presents with low-grade fever, fatigue,
arthralgia, and colicky abdominal pain. The hallmark of the disease is the rash,
which begins as pink maculopapules, progresses to petechiae or purpura, which
are clinically palpable, and changes in color from red to dusty brown before
fading. Arthritis, usually involving the knees and ankles, is present in two-thirds of
cases, and gastrointestinal tract involvement results in heme-positive stools in 50%
of cases. Laboratory findings are not specific or diagnostic, and include indications
of mild to moderate thrombocytosis, leukocytosis, and anemia, and an elevated
erythrocyte sedimentation rate. Treatment is typically symptomatic and
supportive, although corticosteroids are indicated in the rare patient with lifethreatening gastrointestinal or central nervous system manifestations. Systemic
juvenile-onset rheumatoid arthritis usually presents with an evanescent salmonpink rash. Rocky Mountain spotted fever does not present with arthritis and the
rash begins distally on the legs. Iron ingestion does not typically cause a rash,
fever, or arthritis. Disseminated anthrax does not present with a rash and joint
symptoms.
A. Systemic onset juvenile rheumatoid
arthritis
B. Rocky Mountain spotted fever
C. Henoch-Schonlein purpura
D. Disseminated anthrax
E. Acute iron ingestion
167.
A 7-year-old African-American male is brought
to your office with a 1-day history of purulent,
crusted eyelashes in the morning, and red eye.
There is no history of visual change, foreign
body, or injury. The child is otherwise in good
health and has normal developmental
milestones. No fever or respiratory distress is
noted. A clinical diagnosis of bacterial
conjunctivitis is made. The mother is anxious
to keep the child in school. Which one of the
following would be the most appropriate
time for the child to return to school? (check
one)
A. Once treatment is started
B. When there is no crusting or drainage in the
morning
C. After 1 week of treatment
D. When the absence of fever for 24 hours is
documented
E. When there is resolution of conjunctival
erythema
A. Once treatment is started. Once therapy is initiated, children with bacterial
conjunctivitis should be allowed to remain in school. Careful hand hygiene is
important, however, and behavior must be appropriate to maintain adequate
hygiene. No specific length of treatment or evidence of clinical response is required
before returning to school. Reference: Pickering LK (ed): Red Book: 2006 Report of
the Committee on Infectious Diseases, ed 27. American Academy of Pediatrics,
2006, p 149.
168.
A 7-year-old female is brought to your clinic by her
mother, who has concerns about her behavior. For
the last 2 months, the patient has resisted going to
school. Each school morning she complains of not
feeling well and asks to stay home. When forcibly
taken to school she cries and begs to go home. Once
at home she is playful and engages in normal
activities. She also resists attending her usual
swimming lessons in the evenings. She has frequent
nightmares in which one of her parents dies.
After a thorough history and physical examination
rule out an underlying medical condition, you
diagnose the patient with: (check one)
A. separation anxiety disorder
B. generalized anxiety disorder
C. acute stress disorder
D. panic disorder with agoraphobia
E. social phobia
A. separation anxiety disorder. This patient suffers from separation anxiety
disorder, which is unique to pediatric patients and is characterized by
excessive anxiety regarding separation from the home or from people the
child is attached to, such as family members or other caregivers. The anxiety
is beyond what is developmentally appropriate for the child's age. Patients
may even suffer distress from anticipation of the separation. Other
characteristics include persistent worry about harm occurring to major
attachment figures, worry about an event that may separate the patient
from caregivers, reluctance to attend school due to the separation it implies,
fear of being alone, recurring nightmares with themes of separation, and
physical complaints when faced with separation. Children diagnosed with
separation anxiety disorder must be under 18 years of age and have had
symptoms for at least 4 weeks.
Social phobia is a persistent fear of a specific object or situation. Exposure to
the object provokes an immediate anxiety response such as a panic attack.
To meet the criteria for social phobia, patients must suffer symptoms for at
least 6 months. Generalized anxiety disorder is characterized as excessive
anxiety and worry regarding a number of events or activities. Physical
symptoms include restlessness, irritability, or sleep disturbance. Symptoms
must be present for at least 6 months.
Acute stress disorder occurs after a traumatic event that the individual
considers life threatening. Patients experience dissociative symptoms,
flashbacks, and increased arousal. Symptoms are present for at least 2 days,
with a maximum of 4 weeks. Beyond 4 weeks, a diagnosis of posttraumatic
stress disorder is made. Panic disorder with agoraphobia is characterized by
recurrent panic attacks with a fear of being in situations in which the patient
cannot escape or may be embarrassed by doing so. Symptoms must be
present for 1 month for the diagnosis to be made (SOR C).
169.
A 7-year-old female with a history of asthma is
brought to your office for a routine follow-up visit.
She has a history of exercise-induced asthma, but
also has had exacerbations in the past that were
unrelated to exercise. In the past month, she has
premedicated herself with albuterol (Proventil,
Ventolin) with a spacer before recess 5 days/week as
usual. She has also needed her albuterol to treat
symptoms (wheezing and/or shortness of breath)
once or twice per week and had one exacerbation
requiring medical treatment in the past year. She
has had no nighttime symptoms. Albuterol as
needed is her only medication.
After reinforcing asthma education, which one of
the following would be most appropriate? (check
one)
A. Referral to an asthma specialist
B. Addition of a low-dose inhaled corticosteroid
C. Addition of a long-acting -agonist
D. Elimination of premedication with albuterol,
restricting use to an as-needed basis
E. No changes to her regimen
E. No changes to her regimen. This patient's asthma is well-controlled
according to the 2007 NHLBI asthma guidelines. The "rule of twos" is useful
in assessing asthma control: in children under the age of 12, asthma is NOT
well-controlled if they have had symptoms or used a -agonist for symptom
relief more than twice per week, had two or more nocturnal awakenings
due to asthma symptoms in the past month, or had two or more
exacerbations requiring systemic corticosteroids in the past year. For
individuals over 12 years of age, there must be more than two nocturnal
awakenings per month to classify their asthma as not well controlled.
Exercise-induced asthma is considered separately. A -agonist used as
premedication before exercise is not a factor when assessing asthma control.
Since this patient does not exceed the rule of twos, her asthma is categorized
as well-controlled and no changes to her therapy are indicated. Asthma
education should be reinforced at every visit.
170.
A 7-year-old male complains of left shoulder pain after a
bicycle accident. The neurovascular evaluation is normal.
A radiograph is shown in Figure 4.
...
171.
A 7-year-old male is hospitalized after sustaining
abdominal trauma in an accident. The child is conscious.
His pulse rate is 150 beats/min, his systolic blood pressure
is palpated at 60 mm Hg, and his respiratory rate is
40/min. His hemoglobin level is 4.0 g/dL because of
trauma-related blood loss. His clinical condition is
deteriorating despite an infusion of intravenous volume
expanders, but the parents are Jehovah's Witnesses and
refuse to consent to a blood transfusion because of their
religious convictions. Your prognosis is that without a
blood transfusion the patient will die. According to
medical-legal precedent, which one of the following is
correct? (check one)
A. The patient should receive the transfusion regardless of the
parents' wishes. The refusal to accept any medical intervention,
including life-saving blood transfusions, has been well established for
adults who have the ability to definitively communicate their wishes.
Also, parents have the power to give or withhold consent to medical
treatment on behalf of their children. However, Western courts have
deemed that parents cannot refuse emergency, life-saving treatment
to children based on these principles: (1) the child's interests and
those of the state outweigh parental rights to refuse medical
treatment; (2) parental rights do not give parents life and death
authority over their children; and (3) parents do not have an absolute
right to refuse medical treatment for their children, if that refusal is
regarded as unreasonable.
A. The patient should receive the transfusion regardless of
the parents' wishes
B. The patient can be transfused regardless of the parents'
wishes once he becomes asystolic
C. The parents may refuse the transfusion if they are in
agreement
D. The parents may refuse the transfusion if the patient
identifies himself as a Jehovah's Witness
E. The parents may refuse the transfusion if there is a
legally executed advance directive
172.
A 7-year-old male presents with a 3-day history of sore
throat, hoarseness, fever to 100 degrees (38 degrees C),
and cough. Examination reveals injection of his tonsils,
no exudates, and no abnormal breath sounds. Which one
of the following would be most appropriate? (check one)
A. Recommend symptomatic treatment
B. Perform a rapid antigen test for streptococcal
pharyngitis
C. Treat empirically for streptococcal pharyngitis
D. Perform a throat culture for streptococcal pharyngitis
E. Perform an office test for mononucleosis
A. Recommend symptomatic treatment. Pharyngitis is a common
complaint, and usually has a viral cause. The key factors in diagnosing
streptococcal pharyngitis are a fever over 100.4 degrees F, tonsillar
exudates, anterior cervical lymphadenopathy, and absence of cough.
Age plays a role also, with those <15 years of age more likely to have
streptococcal infection, and those 10-25 years of age more likely to
have mononucleosis. The scenario described is consistent with a viral
infection, with no risk factors to make streptococcal infection likely;
therefore, this patient should be offered symptomatic treatment for
likely viral infection. Testing for other infections is not indicated unless
the patient worsens or does not improve.
173.
A 7-year-old male presents with a
fever of 38.5C (101.3F), a sore
throat, tonsillar inflammation, and
tender anterior cervical adenopathy.
He does not have a cough or a runny
nose. His younger sister was treated
for streptococcal pharyngitis last
week and his mother would like him
to be treated for streptococcal
infection.
A. Empiric antibiotic treatment for streptococcal pharyngitis is warranted.. The patient has a
score of 5 under the Modified Centor scoring system for management of sore throat. Patients
with a score 4 are at highest risk (at least 50%) of having group A -hemolytic streptococcal
(GABHS) pharyngitis, and empiric treatment with antibiotics is warranted. Various national
and international organizations disagree about the best way to manage pharyngitis, with no
consensus as to when or how to test for GABHS and who should receive treatment. The
minimal benefit seen with tonsillectomy in reducing the incidence of recurrent GABHS
pharyngitis does not justify the risks or cost of surgery. Treatment of pets for the prevention
of GABHS infection has proven ineffective.
Which one of the following is true
concerning this situation?
(check one)
A. Empiric antibiotic treatment for
streptococcal pharyngitis is
warranted.
B. The chance of this patient having a
positive rapid antigen detection test
for Streptococcus is <50%.
C. There is a generalized consensus
among the various national
guidelines for management of
pharyngitis.
D. The patient should have a
tonsillectomy when he recovers from
this infection.
E. The family dog should be treated
for streptococcal infection.
174.
A 7-year-old male with recurrent
sinusitis has difficulty breathing
through his nose. He has had chronic
diarrhea and his weight is at the 5th
percentile. Nasal polyps are noted on
examination, in the form of grayish
pale masses in both nares. No nasal
purulence or odor is present. Which
one of the following tests should you
order? (check one)
A. A serum angiotensin-converting
enzyme level
B. A serum alpha1-antitrypsin level
C. A serum ceruloplasmin level
D. An erythrocyte sedimentation rate
E. A sweat chloride test
E. A sweat chloride test. This child has chronic diarrhea, recurrent sinusitis, and nasal polyps,
and is underweight. Nasal polyps tend to occur more often in adult males, with the
prevalence increasing in both sexes after age 50. Any child 12 years or younger who
presents with nasal polyps should be suspected of having cystic fibrosis until proven
otherwise. A sweat chloride test, along with a history and clinical examination, is necessary to
evaluate this possibility. Nasal polyps are found in 1% of the normal population, but a full
18% of those with cystic fibrosis are afflicted. There is no association of polyps with Wilson's
disease, sarcoidosis, or emphysema, so serum ceruloplasmin, angiotensin-converting
enzyme, and alpha1-antitrypsin levels would not be useful. An erythrocyte sedimentation
rate likewise would yield limited information.
175.
An 8-year-old female is brought to
your office because she has begun to
limp. She has had a fever of 38.8C
(101.8F) and says that it hurts to bear
weight on her right leg. She has no
history of trauma.
On examination, she walks with an
antalgic gait and hesitates to bear
weight on the leg. Range of motion of
the right hip is limited in all
directions and is painful. Her
sacroiliac joint is not tender, and the
psoas sign is negative. Laboratory
testing reveals an erythrocyte
sedimentation rate of 55 mm/hr (N 010), a WBC count of 15,500/mm 3 (N
4500-13,500), and a C-reactiveprotein
level of 2.5 mg/dL (N 0.5-1.0).
D. Ultrasonography. This child meets the criteria for possible septic arthritis. In this case
ultrasonography is recommended over other imaging procedures. It is highly sensitive for
detecting effusion of the hip joint. If an effusion is present, urgent ultrasound-guided
aspiration should be performed. Bone scintigraphy is excellent for evaluating a limping
child when the history, physical examination, and radiographic and sonographic findings
fail to localize the pathology. CT is indicated when cortical bone must be visualized. MRI
provides excellent visualization of joints, soft tissues, cartilage, and medullary bone. It is
especially useful for confirming osteomyelitis, delineating the extent of malignancies,
identifying stress fractures, and diagnosing early Legg-Calv-Perthes disease. Plain film
radiography is often obtained as an initial imaging modality in any child with a limp.
However, films may be normal in patients with septic arthritis, providing a false-negative
result.
Which one of the following will
provide the most useful diagnostic
information to further evaluate this
patient's problem?
(check one)
A. MRI
B. CT
C. A bone scan
D. Ultrasonography
E. Plain-film radiography
176.
An 8-year-old female is brought to
your office with a 3-day history of
bilateral knee pain. She has had no
associated upper respiratory
symptoms. On examination she is
afebrile. Her knees have full range of
motion and no effusion, but she has a
purpuric papular rash on both lower
extremities.
Which one of the following is the
most likely cause of her symptoms?
(check one)
A. Henoch-Schnlein purpura
B. Rocky Mountain spotted fever
C. Juvenile rheumatoid arthritis
D. Lyme disease
E. Rheumatic fever
A. Henoch-Schnlein purpura. The combination of arthritis with a typical palpable purpuric
rash is consistent with a diagnosis of Henoch-Schnlein purpura. This most often occurs in
children from 2 to 8 years old. Arthritis is present in about two-thirds of those affected.
Gastrointestinal and renal involvement are also common.
Rocky Mountain spotted fever presents with a rash, but arthralgias are not typical. These
patients are usually sick with a fever and headache. Juvenile rheumatoid arthritis is
associated with a salmon-pink maculopapular rash, but not purpura. The rash associated
with Lyme disease is erythema migrans, which is a bull's-eye lesion at the site of a tick bite.
The rash associated with rheumatic fever is erythema marginatum, which is a pink, raised,
macular rash with sharply demarcated borders.
177.
An 8-year-old male is brought to your office for
evaluation of recurrent headaches. His mother
explains that the headaches occur at least twice a
week and often require him to miss school. The
patient says he sometimes feels nauseated and
that being in a dark room helps. His mother
states that she had migraines as a child. The
child's only other medical issue is constipation. A
head CT ordered by another physician was
negative.
D. Propranolol (Inderal). This patient most likely is suffering from recurrent
migraine headaches; at the described frequency and intensity, he meets the
criteria for prophylactic medication. Ibuprofen or acetaminophen could still be
used as rescue medications, but a daily agent is indicated and propranolol is
the best choice for this patient (SOR B). Sumatriptan is not approved for
children under the age of 12 years. Carbamazepine has significant side effects
and requires monitoring. Amitriptyline is a commonly used agent, but it could
worsen his constipation.
Which one of the following would be best for
preventing these episodes? (check one)
A. Sumatriptan (Imitrex)
B. Ibuprofen
C. Carbamazepine (Tegretol)
D. Propranolol (Inderal)
E. Amitriptyline
178.
An 8-year-old male presents to your office 2 days
after returning from a trip to Mexico with his
family. He developed watery, nonbloody diarrhea
on the day of departure. He has mild abdominal
cramping, but no fever or vomiting. His mother
had similar symptoms, which were milder and
resolved with over-the-counter treatments.
Which one of the following would be most
appropriate to treat this patient's condition?
(check one)
C. Azithromycin (Zithromax). Traveler's diarrhea commonly occurs in travelers
to Mexico and developing countries. It is usually caused by bacterial organisms
such as Escherichia coli, Campylobacter, Shigella, and Salmonella. Viral and
parasitic organisms are less common causes, unless the diarrhea persists for 2
weeks. Appropriate medications include antidiarrheal agents such as
loperamide, bismuth subsalicylate, and antibiotics. Fluoroquinolones are
effective in adults, but should not be used in an 8-year-old. Azithromycin
isgenerally effective and safe in children. Metronidazole, mebendazole, and
metoclopramide would not be likely to successfully treat bacterial traveler's
diarrhera.
A. Metronidazole (Flagyl)
B. Ciprofloxacin (Cipro)
C. Azithromycin (Zithromax)
D. Mebendazole
E. Metoclopramide (Reglan)
179.
An 8-year-old male presents with cervical
lymphadenitis. He has a kitten at home and you
are concerned about cat-scratch disease. Which
one of the following antibiotics is most
appropriate for treatment of Bartonella henselae
infection? (check one)
A. Azithromycin (Zithromax). Azithromycin has been shown to reduce the
duration of lymphadenopathy in cat-scratch disease (SOR B). Other antibiotics
that have been used include rifampin, ciprofloxacin,
trimethoprim/sulfamethoxazole, and gentamicin. Ceftriaxone,
amoxicillin/clavulanate, doxycycline, and clindamycin are not effective in the
treatment of Bartonella infection.
A. Azithromycin (Zithromax)
B. Ceftriaxone (Rocephin)
C. Amoxicillin/clavulanate (Augmentin)
D. Doxycycline
E. Clindamycin (Cleocin)
180.
An 8-year-old white male presents with a 4-day
history of erythematous cheeks, giving him a
"slapped-cheek" appearance. Examination of the
extremities reveals a mildly pruritic, reticulated,
erythematous, maculopapular rash (see Figure 1).
He is afebrile and no other constitutional
symptoms are present.
...
181.
A 9-month-old male is brought to your office by his
mother because of concerns about his eating. She states
that he throws tantrums while sitting in his high chair,
dumps food on the floor, and refuses to eat. She has
resorted to feeding him cookies, crackers, and juice,
which are all he will eat. A complete physical
examination, including a growth chart of weight, length,
and head circumference, is normal. Which one of the
following would be the most appropriate
recommendation? (check one)
A. Use disciplinary measures to force the child to eat a
healthy breakfast, lunch, and dinner
B. Leave the child in the high chair until he has eaten all
of the healthy meal presented
C. Play feeding games to encourage consumption of
healthy meals or snacks
D. Skip the next meal if the child refuses to eat
E. Provide healthy foods for all meals and snacks, and end
the meal if the child refuses to eat
182.
A 9-month-old male is seen for a routine well-baby
examination. There have been no health problems and
developmental milestones are normal. Review of the
growth chart shows that length, weight, and head
circumference have continued to remain at the 75th
percentile. The examination is normal with the exception
of the anterior fontanelle being closed. Proper
management at this time would include: (check one)
A. A CT scan of the head
B. MRI of the head
C. A CBC, a metabolic profile, and thyroid studies
D. Referral to a neurologist
E. Serial measurement of head circumference
183.
A 9-month-old white male is brought to your office for a
well-child visit. You note that the child's weight gain has
been flat over the last several months. He has fallen from
the 75th percentile to the 15th for weight, and his
percentile for length is beginning to decline as well. The
mother states that the child began having diarrhea as
soon as she began giving him various grain cereals and
baby foods 5 months ago. The remainder of a review of
systems and a social and family history is unremarkable.
Physical examination reveals an undernourished infant
with mild abdominal distention. A check of the infant's
hemoglobin shows a microcytic anemia with a low serum
ferritin level. Which one of the following is the most
likely diagnosis? (check one)
A. Thalassemia minor
B. Celiac sprue
C. Cystic fibrosis
D. Congenital megacolon (Hirschsprung's disease)
E. Inborn error of metabolism
E. Provide healthy foods for all meals and snacks, and end the meal if
the child refuses to eat. It is estimated that 3%-10% of infants and
toddlers refuse to eat according to their caregivers. Unlike other
feeding problems such as colic, this problem tends to persist without
intervention. It is recommended that caregivers establish food rules,
such as healthy scheduled meals and snacks, and apply them
consistently. Parents should control what, when, and where children
are being fed, whereas children should control how much they eat at
any given time in accordance with physiologic signals of hunger and
fullness. No food or drinks other than water should be offered
between meals or snacks. Food should not be offered as a reward or
present. Parents can be reassured that a normal child will learn to eat
enough to prevent starvation. If malnutrition does occur, a search for a
physical or mental abnormality should be sought.
E. Serial measurement of head circumference. The anterior fontanelle
in the newborn is normally 0.6-3.6 cm, with the mean size being 2.1 cm.
It may actually enlarge the first few months, but the medial age of
closure is 13.8 months. The anterior fontanelle closes at 3 months in 1%
of cases, and by 1 year, 38% are closed. While early closure of the
anterior fontanelle may be normal, the head circumference must be
carefully monitored. The patient needs to be monitored for
craniosynostosis (premature closure of one or more sutures) and for
abnormal brain development. When craniosynostosis is suspected, a
skull radiograph is useful for initial evaluation. If craniosynostosis is
seen on the film, a CT scan should be obtained.
B. Celiac sprue. Celiac sprue is a condition of acquired malabsorption
that resolves when the patient is exposed to a gluten-free diet. Gluten
is a substance found in wheat, rye, and barley, but not in corn or rice
products. Children with this sensitivity will develop inflammation and
destruction of the microvilli in the small intestine as a result of an
immune response to gluten. Patients with celiac sprue often present as
this child has, between 4 and 24 months of age with impaired growth,
diarrhea, and abdominal distention. An iron deficiency anemia can
occur with impairment of iron absorption from the small intestine.
Lesser cases of malabsorption are common, and this condition often
goes unrecognized into adolescence or adulthood. Serologic tests, and
ultimately a biopsy of the small intestine, can confirm the diagnosis.
184.
A 10-week-old term male infant is brought to
your office with a 2-day history of difficulty
breathing. He has been healthy since birth, with
the exception of a 3-day episode of wheezing and
rhinorrhea 3 weeks ago. Your initial
examination shows an alert infant with
increased work of breathing, rhinorrhea, and
wheezing. His oxygen saturation is 93% and his
temperature is 38.4C (101.1F).
Which one of the following would be most
appropriate at this point? (check one)
A. Antigen testing or another rapid assay
B. A baseline chest radiograph
C. A trial of nebulized albuterol (AccuNeb)
D. Advising the parents that the child can safely
be returned to day care tomorrow
185.
C. A trial of nebulized albuterol (AccuNeb). The American Academy of Pediatrics
guideline on the diagnosis and management of bronchiolitis recommends
against the use of laboratory or radiographic studies to make the diagnosis,
although additional testing may be appropriate if there is no improvement.
Bronchiolitis can be caused by a number of different viruses, alone or in
combination, and the knowledge gained from virologic testing rarely influences
management decisions or outcomes for the vast majority of children.
While the guideline does not support routine use of bronchodilators in the
management of bronchiolitis, it does allow for a trial of bronchodilators as an
option in selected cases, and continuation of the treatment if the patient shows
objective improvement in respiratory status. Bronchodilators have not been
shown to affect the course of bronchiolitis with respect to outcomes.
The guideline places considerable emphasis on hygienic practices, including the
use of alcohol-based hand sanitizers before and after contact with the patient or
inanimate objects in the immediate vicinity. Education of the family about
hygienic practices is recommended as well. Returning the child to day care the
next day is potentially harmful.
An 11-year-old female has been diagnosed with
"functional abdominal pain" by a pediatric
gastroenterologist. Her mother brings her to see
you because of concerns that another diagnosis
may have been overlooked despite a very
thorough and completely normal evaluation for
organic causes.
Which one of the following would you
recommend? (check one)
E. Stress reduction and participation in usual activities as much as possible. The
diagnosis of functional abdominal pain is made when no structural, infectious,
inflammatory, or biochemical cause for the pain can be found. It is the most
common cause of recurrent abdominal pain in children 4-16 years of age. The
use of medications may be helpful in reducing (but rarely eradicating)
functional symptoms, and remaining open to the possibility of a previously
unrecognized organic disorder is appropriate. However, continuing to focus on
organic causes, invasive tests, or physician visits can actually perpetuate a child's
complaints and distress.
A. A trial of inpatient hospital admission
B. Increased testing and levels of referral until a
true diagnosis is reached
C. Removing the child from school and activities
whenever symptoms occur
D. Medications to eradicate symptoms
E. Stress reduction and participation in usual
activities as much as possible
It is estimated that approximately 30%-50% of children with functional
abdominal pain will have resolution of their symptoms within 2 weeks of
diagnosis. Recommendations for managing this problem include focusing on
participation in normal age-appropriate activities, reducing stress and
addressing emotional distress, and teaching the family to cope with the
symptoms in a way that prevents secondary gain on the part of the child.
186.
A 12-month-old white female whom you have seen
regularly for all of her scheduled well child care is found
to have a hemoglobin level of 9.0 g/dL (N for age 10.513.5). She started whole milk at 9 months of age. She
appears healthy otherwise and has no family history of
anemia. A CBC reveals a mild microcytic, hypochromic
anemia with RBC poikilocytosis, but is otherwise
normal. The RBC distribution width is also elevated.
D. prescribe oral iron. Iron deficiency is almost certainly the diagnosis in
this child. The patient's response to a therapeutic trial of iron would be
most helpful in establishing the diagnosis. Additional tests might be
necessary if there is no response.
Of the following, the most appropriate next step would
be to: (check one)
A. order tests for serum iron and total iron-binding
capacity
B. order a serum ferritin level
C. order hemoglobin electrophoresis
D. prescribe oral iron
E. perform stool guaiac testing
187.
A 12-year-old female is brought to your office with an 8day history of sore throat and fever, along with
migratory aching joint pain. She is otherwise healthy
and has no history of travel, tick exposure, or prior
systemic illness. A physical examination is notable for
exudative pharyngitis; a blanching, sharply demarcated
macular rash over her trunk; and a III/VI systolic
ejection murmur. Joint and neurologic examinations
are normal. A rapid strep test is positive and her Creactive protein level is elevated.
Of the following, the most likely diagnosis is: (check
one)
A. juvenile rheumatoid arthritis
B. infective endocarditis
C. Kawasaki syndrome
D. acute rheumatic fever
E. Lyme disease
188.
A 12-year-old Hispanic female develops fever, knee pain
with swelling, diffuse abdominal pain, and a palpable
purpuric rash. A CBC and platelet count are normal.
Her long-term prognosis depends on the severity of
involvement of the: (check one)
A. gastrointestinal tract
B. heart
C. liver
D. kidneys
E. lungs
D. acute rheumatic fever. Acute rheumatic fever is very common in
developing nations. It was previously rare in the U.S., but had a
resurgence in the mid-1980s. It is most common in children ages 5-15
years. The diagnosis is based on the Jones criteria. Two major criteria, or
one major criterion and two minor criteria, plus evidence of a preceding
streptococcal infection, indicate a high probability of the disease.
Major criteria include carditis, migratory polyarthritis, erythema
marginatum, chorea, and subcutaneous nodules. Minor criteria include
fever, arthralgia, an elevated erythrocyte sedimentation rate or Creactive protein (CRP) level, and a prolonged pulse rate interval on EKG.
The differential diagnosis is extensive and there is no single laboratory
test to confirm the diagnosis. This patient meets one major criterion
(erythema marginatum rash) and three minor criteria (fever, elevated
CRP levels, and arthralgia). Echocardiography should be performed if
the patient has cardiac symptoms or an abnormal cardiac examination,
to rule out rheumatic carditis.
D. kidneys. This patient has Henoch-Schnlein purpura. This condition
is associated with a palpable purpuric rash, without thrombocytopenia.
Other diagnostic criteria include bowel angina (diffuse abdominal pain
or bowel ischemia), age 20, renal involvement, and a biopsy showing
predominant immunoglobulin A deposition. The long-term prognosis
depends on the severity of renal involvement. Almost all children with
Henoch-Schnlein purpura have a spontaneous resolution, but 5% may
develop end-stage renal disease. Therefore, patients with renal
involvement require careful monitoring (SOR A).
189.
A 12-year-old Hispanic female is brought
to your office because of the recent
onset of a white vaginal discharge. She is
otherwise asymptomatic and has never
menstruated. She denies sexual activity
and a general examination reveals no
abnormalities. You note the presence of
breast buds and scant pubic hair.
Microscopic examination of the vaginal
discharge shows sheets of vaginal
epithelial cells. Which one of the
following is most likely in this setting?
(check one)
E. Physiologic secretions. This child is entering puberty. In the 6- to 12-month period
before menarche, girls often develop a physiologic vaginal discharge secondary to the
increase in circulating estrogens. The gray-white discharge is non-irritating. When
physiologic discharge is examined with the microscope, sheets of vaginal epithelial cells
are seen. The only treatment necessary is reassurance of both parents and child that this
is a normal process that will subside with time. The other conditions listed are pathologic
and have other associated symptoms and findings not seen in this case. Pinworms
normally cause perianal and vulvar pruritus and irritation. The findings in sexual abuse
range from an inflamed vulvovaginal area, to evidence of sexually transmitted diseases,
to evidence of local trauma. Trichomoniasis would cause vulvovaginal irritation and
microscopic examination of the discharge would show Trichomonas organisms. A vaginal
foreign body would usually present with a foul and/or bloody vaginal discharge.
A. Pinworm (Enterobius vermicularis)
infestation
B. Sexual abuse
C. Vaginal foreign body
D. Trichomoniasis
E. Physiologic secretions
190.
A 12-year-old male is brought to your
office with an animal bite. After talking
with the patient, you learn that he was
bitten on his left hand as he attempted
to pet a stray cat a little over 24 hours
ago. He says that the bite was very
painful, and that it bled for a few
minutes. His parents cared for the bite
by rinsing it and covering it with a
bandage. His chart indicates that he
received a tetanus shot last year.
On examination, the patient is afebrile
with stable vital signs. The site is warm
and tender to light palpation, with
surrounding erythema measuring
approximately 3 cm in diameter.
Which one of the following is the most
likely infectious agent in this situation?
(check one)
A. Candida albicans
B. Capnocytophaga canimorsus
C. Methicillin-resistant Staphylococcus
aureus (MRSA)
D. Pasteurella multocida
E. Streptococcus pneumoniae
D. Pasteurella multocida. Pasteurella species are isolated from up to 50% of dog bite
wounds and up to 75% of cat bite wounds, and the hand is considered a high-risk area
for infection (SOR A). Although much more rare, Capnocytophaga canimorsus, a
fastidious gram-negative rod, can cause bacteremia and fatal sepsis after animal bites,
especially in asplenic patients or those with underlying hepatic disease. Anaerobes
isolated from dog and cat bite wounds include Bacteroides, Fusobacterium,
Porphyromonas, Prevotella, Propionibacterium and Peptostreptococcus.
In addition to animal oral flora, human skin flora are also important pathogens, but are
less commonly isolated. These can include streptococci and staphylococci, including
methicillin-resistant Staphylococcus aureus (MRSA). Coverage for MRSA may be
especially important if the patient has risk factors for colonization with communityacquired MRSA. Pets can also become colonized with MRSA and transmit it via bites and
scratches.
Cat bites that become infected with Pasteurella multocida can be complicated by cellulitis,
which may form around the wound within 24 hours and is often accompanied by
redness, tenderness, and warmth. The use of prophylactic antibiotics is associated with a
statistically significant reduction in the rate of infection in hand bites (SOR A). If infection
develops and is left untreated, the most common complications are tenosynovitis and
abscess formation; however, local complications can include septic arthritis and
osteomyelitis. Fever, regional adenopathy, and lymphangitis are also seen.
191.
A 12-year-old male middle-school
wrestler comes to your office
complaining of a recurrent painful rash
on his arm. There appear to be several
dry vesicles. The most likely diagnosis is
which one of the following? (check one)
C. Herpes gladiatorum. The most common infection transmitted person-to-person in
wrestlers is herpes gladiatorum caused by the herpes simplex virus. Molluscum
contagiosum causes keratinized plugs. Human papillomavirus causes warts. Tinea
corporis is ringworm, which is manifested by round to oval raised areas with central
clearing. Mat burn is an abrasion.
A. Molluscum contagiosum
B. Human papillomavirus
C. Herpes gladiatorum
D. Tinea corporis
E. Mat burn
192.
A 12-year-old male presents with left hip
pain. He is overweight and recently
started playing tennis to lose weight. He
says the pain started gradually after his
last tennis game, but he does not recall
any injury. He is walking with a limp.
On examination he is afebrile and has
limited internal rotation of the left hip.
D. Slipped capital femoral epiphysis. Slipped capital femoral epiphysis is the most
common hip disorder in this patient's age group. It usually occurs between the ages of 8
and 15 and is more common in boys and overweight or obese children. It presents with
limping and pain, and limited internal rotation of the hip is noted on physical
examination.
Septic arthritis would typically present with a fever. Juvenile rheumatoid arthritis,
transient synovitis, and Legg-Calv-Perthes disease are more common in younger
children.
What is the most likely cause of the hip
pain? (check one)
A. Septic arthritis
B. Juvenile rheumatoid arthritis
C. Transient synovitis
D. Slipped capital femoral epiphysis
E. Legg-Calv-Perthes disease
193.
A 12-year-old male uses a short-acting
bronchodilator three times per week to
control his asthma. Lately he has been
waking up about twice a week due to
his symptoms.
Which one of the following medications
would be most appropriate?
(check one)
A. Inhaled medium-dose
corticosteroids
B. A scheduled short-acting
bronchodilator
C. A scheduled long-acting
bronchodilator
D. A leukotriene inhibitor
A. Inhaled medium-dose corticosteroids. This patient has moderate persistent asthma.
Although many parents are concerned about corticosteroid use in children with open
growth plates, inhaled corticosteroids have not been proven to prematurely close growth
plates, and are the most effective treatment with the least side effects. Scheduled use of a
shortacting bronchodilator has been shown to cause tachyphylaxis, and is not
recommended. The same is true for long-acting bronchodilators. Leukotriene use may be
beneficial, but compared to those using inhaled corticosteroids, patients using
leukotrienes are 65% more likely to have an exacerbation requiring systemic
corticosteroids.
194.
A 12-year-old male who
lives on a farm presents
with lesions on his toes
(shown in Figure 7).
Which one of the
following items from the
patient's history is
relevant to the diagnosis?
(check one)
D. Exposure to cold temperatures. This patient has pernio, or chilblains, which is a localized inflammatory
lesion of the skin, usually found
in the extremities following exposure to nonfreezing cold temperatures. It is generally a benign condition,
and is not associated with any systemic diseases. These lesions are red-purple plaques with deep
swelling,
and are accompanied by itching or burning. They are not associated with infections or connective tissue
disease.
A. Recent tooth
extraction and gingival
surgery
B. A family history of
systemic lupus
erythematosus
C. Recurrent fevers for
the past 2 weeks
D. Exposure to cold
temperatures
E. Vaccination of the
sheep he is raising for a 4H project
195.
A 12-year-old white male
asthmatic has an acute
episode of wheezing. You
diagnose an acute
asthma attack and
prescribe an inhaled 2adrenergic agonist. After
2 hours of treatment, he
continues to experience
wheezing and shortness
of breath.
Which one of the
following is the most
appropriate addition to
acute outpatient
management?
(check one)
A. Oral theophylline
(Theo-Dur)
B. Oral corticosteroids
C. An oral -adrenergic
agonist
D. Inhaled cromolyn
(Intal)
E. Inhaled
corticosteroids
B. Oral corticosteroids. The treatment of choice for occasional acute symptoms of asthma is an inhaled 2adrenergic agonist such as albuterol, terbutaline, or pirbuterol. If symptoms do not respond to agonists, they should be treated with a short course of systemic corticosteroids. Theophylline has limited
usefulness for treatment of acute symptoms in patients with intermittent asthma; it is a less potent
bronchodilator than subcutaneous or inhaled adrenergic drugs, and therapeutic serum concentrations
can cause transient adverse effects such as nausea and central nervous system stimulation in patients
who have not been taking the drug continuously. Cromolyn can decrease airway hyperreactivity, but
has no bronchodilating activity and is useful only for prophylaxis. Inhaled corticosteroids should be used
to suppress the symptoms of chronic persistent 2 asthma. Oral 2-selective agonists are less effective and
have a slower onset of action than the same drugs given by inhalation.
196.
A 12-year-old white male is brought to your office after
accidentally cutting his left hand with a pocketknife. On
examination you find a deep 2-cm laceration at the base of the
thenar eminence. To test for motor injury to the median nerve you
would have the patient: (check one)
A. Extend the thumb and fingers
B. Oppose the thumb and little finger
C. Flex the wrist
D. Abduct the thumb and index finger
197.
A 13-year-old male is found to have hypertrophic cardiomyopathy.
His father also had hypertrophic cardiomyopathy, and died
suddenly at age 38 following a game of tennis. The boy's mother
asks you for advice regarding his condition. What advice should
you give her? (check one)
A. He may participate in noncontact sports
B. He should receive lifelong treatment with beta-blockers
C. His condition usually decreases lifespan
D. His hypertrophy will regress with age
E. His siblings should undergo echocardiography
198.
A 13-year-old white female reports a 6-month history of
intermittent abdominal cramping, with each episode becoming
progressively worse. Based on her history, there is no obvious
relationship to eating, voiding, or defecating. She reports that she
has not yet begun menstruating and is not sexually active. Her
weight has been stable. She appears to be in mild emotional
distress about being "the last girl in her class to have a period." She
is in no physical discomfort and her vital signs are normal.
Secondary sexual characteristics appear to be developing
normally. She is in the 57th percentile for height and the 65th
percentile for weight. A complete physical examination confirms
your presumptive diagnosis. The therapeutic procedure of choice
would be: (check one)
A. Appendectomy
B. Colonoscopy
C. Hymenotomy
D. Cystoscopy
E. Paracentesis
B. Oppose the thumb and little finger. The ability to touch
the tip of the thumb to the tip of the little finger indicates
normal motor function of the median nerve. The radial
nerve controls extension of the thumb and fingers. The
median nerve partially controls flexion of the wrist, but the
site of innervation is proximal to the wound site at the base
of the thumb. Abduction of the thumb is a function of the
radial nerve. Finger abduction is a function of the ulnar
nerve.
E. His siblings should undergo echocardiography.
Hypertrophic cardiomyopathy is an autosomal dominant
condition and close relatives of affected individuals should
be screened. The hypertrophy usually stays the same or
worsens with age. This patient should not participate in
strenuous sports, even those considered noncontact. Betablockers have not been shown to alter the progress of the
disease. The mortality rate is believed to be about 1%, with
some series estimating 5%. Thus, in most cases lifespan is
normal.
C. Hymenotomy. The key to making a diagnosis of
imperforate hymen, aside from the obvious finding on
physical examination, lies in the systematic drawing of
inferences. One can speculate that this patient's recurrent
crescendo abdominal cramping represented six menstrual
sheddings, with no egress from the body. Her delay in
menarche, despite normal growth parameters, offers
another clue that structural amenorrhea is present.
Amounts of retained blood vary among patients; up to 3000
mL have been reported. A large volume can accumulate
without causing any permanent damage, and subsequent
fertility is usually normal. Hymenotomy will relieve the
pressure, and normal menses should ensue.
199.
A 14-year-old African-American
female presents for a routine
evaluation. On examination, you
note a rubbery, well-defined,
nontender breast mass
approximately 2 cm in diameter.
The patient denies any history of
breast tenderness, nipple
discharge, or skin changes. The
most likely diagnosis is: (check one)
A. Fibrocystic breast disease
B. Fibroadenoma
C. Benign breast cyst
D. Cystosarcoma phyllodes
E. Intraductal papilloma
200.
A 14-year-old female is brought to
your office by her mother because
of a 3-month history of irritability,
hypersomnia, decline in school
performance, and lack of interest in
her previous extracurricular
activities. The mother is also your
patient, and you know that she has
a history of depression and has
recently separated from her
husband. After an appropriate
workup, you diagnose depression
in the daughter.
For initial therapy you recommend:
(check one)
A. amitriptyline
B. methylphenidate (Ritalin)
C. divalproex sodium (Depakote)
D. cognitive-behavioral therapy
B. Fibroadenoma. Most breast masses in adolescent girls are benign. Fibroadenoma is the
most common, accounting for approximately two-thirds of all adolescent breast masses. It is
characterized by a slow growing, nontender, rubbery, well-defined mass, most commonly
located in the upper, outer quadrant. Size varies, and is most commonly in the range of 2-3
cm. Fibrocystic disease is found in older adolescents and is characterized by bilateral
nodularity and cyclic tenderness. Benign breast cysts are characterized by a spongy, tender
mass with symptoms exacerbated by menses. Cysts are frequently multiple, and spontaneous
regression occurs in 50% of patients. Cystosarcoma phyllodes is a rare tumor with malignant
potential, although most are benign. It presents as a firm, rubbery mass that may enlarge
rapidly. Skin necrosis is usually associated with the tumor. Intraductal papillomas are usually
benign but do have malignant potential. They are commonly subareolar and are associated
with nipple discharge. These tumors are rare in the adolescent population.
D. cognitive-behavioral therapy. This patient has multiple risk factors for depression: the
hormonal changes of puberty, a family history of depression, and psychosocial stressors.
Cognitive-behavioral therapy is effective in treating mild to moderate depression in children
and adolescents (SOR A). SSRIs are an adjunctive treatment reserved for treatment of severe
depression, and have limited evidence for effectiveness in children and adolescents.
Amitriptyline should not be used because of its limited effectiveness and adverse effects (SOR
A). Methylphenidate is used for treating attention deficit disorder, not depression. Divalproex
sodium is used to treat bipolar disorder.
201.
A 14-year-old female is brought to your
office by her parents because of concerns
regarding her low food intake, excessive
exercise, and weight loss. Her weight is
less than 75% of ideal for her height.
Which one of the following sets of
additional findings would indicate that
the patient suffers from severe anorexia
nervosa?
(check one)
E. Hypotension, bradycardia, and hypothermia. Characteristic vital signs in patients
with severe anorexia nervosa include hypotension, bradycardia, and hypothermia.
Criteria for hospital admission include a heart rate <40 beats/min, blood pressure
<80/50 mm Hg, and temperature <36C (97F). Increased cardiac vagal hyperactivity is
thought to cause the bradycardia.
A. Hypertension, tachycardia, and
hyperthermia
B. Hypertension, tachycardia, and
hypothermia
C. Hypotension, tachycardia, and
hypothermia
D. Hypotension, bradycardia, and
hyperthermia
E. Hypotension, bradycardia, and
hypothermia
202.
A 14-year-old male is brought to your
office by his mother to establish care. The
patient has been diagnosed with asthma,
but has not been on any medications for
the past year. When questioned, he
reports that his asthmatic symptoms
occur daily and more than one night per
week. On examination, he is found to
have a peak expiratory flow of 75%. Based
on these findings, the most accurate
classification of this patient's asthma is:
(check one)
A. Mild intermittent
B. Mild persistent
C. Moderate persistent
D. Severe persistent
C. Moderate persistent. The National Asthma Education and Prevention Program
(NAEPP) classifies asthma into four categories. Mild intermittent asthma is characterized
by daytime symptoms occurring no more than 2 days per week and nighttime
symptoms no more than 2 nights per month. The peak expiratory flow (PEF) or forced
expiratory volume in 1 second (FEV1) is 80% or more of predicted. Mild persistent
asthma is characterized by daytime symptoms more than 2 days per week, but less
than once a day, and nighttime symptoms more than 2 nights per month. PEF or FEV1 is
80% or more of predicted. Moderate persistent asthma is characterized by daytime
symptoms daily and nighttime symptoms more than 1 night per week. PEF or FEV1 is
60%-80% of predicted. Severe persistent asthma is characterized by continuous
daytime symptoms and frequent nighttime symptoms. PEF or FEV1 is 60% or less of
predicted.
203.
A 15 and a half year-old female is brought to your office
by her mother. They are concerned because she has not
started her periods. She has been healthy, and has
grown several inches in the last year. Her height is now
152 cm (60 in) and she weighs 44 kg (98 lb). She started
to develop breast buds about a year ago and has scant
pubic hair. She denies sexual activity. The mother's
menarche occurred at age 15. Which one of the
following is true in this case? (check one)
A. The patient has delayed puberty and should have her
hormone levels evaluated
B. The patient will likely start her periods within a year
C. Oral contraceptives will be needed to trigger
menarche
D. A pregnancy test should be performed
E. The daughter's age of menarche is unrelated to her
mother's age of menarche
204.
A 15-month-old male is brought to the emergency
department following a generalized tonic-clonic seizure
at home. The parents report that the seizure lasted 5
minutes, with confusion for the next 15 minutes. This is
the child's first seizure. There is no family history of
seizures. His medical history is normal except for a 1day history of a URI. While initially lethargic in the
emergency department, the child is now awake and
playful, with a temperature of 39.5 degrees C (103.2
degrees F) and a normal examination. Appropriate
diagnostic tests are performed, including a blood
glucose level, which is 96 mg/dL. Which one of the
following would be most appropriate to administer at
this point? (check one)
A. Phenytoin (Dilantin) intravenously
B. Ceftriaxone (Rocephin) intravenously
C. Acetaminophen orally
D. Carbamazepine (Tegretol) orally
E. Phenobarbital orally
B. The patient will likely start her periods within a year. The changes
associated with puberty occur in an orderly sequence over a definite
time frame. Any deviation from this sequence or time frame should be
regarded as abnormal. In girls, pubertal development typically requires
4.5 years. Although generally the first sign of puberty is accelerated
growth, breast budding is usually the first recognized pubertal change,
followed by the presence of pubic hair, peak growth velocity, and
menarche. Girls must have adequate nutrition and reach a critical body
weight and body fat percentage before menarche occurs. There is a
concordance in the age of menarche in mother-daughter pairs and
between sisters. Delayed or interrupted puberty is defined as failure to
develop any secondary sex characteristics by age 13, to have menarche
by age 16, or to have menarche 5 or more years after the onset of
pubertal development.
C. Acetaminophen orally. This child has had a simple febrile seizure, the
most common seizure disorder of childhood. Treatment includes finding
a source for the fever; this should include a lumbar puncture if
meningitis is suspected. The most common infections associated with
febrile seizures include viral upper respiratory infections, otitis media,
and roseola. Antipyretics are the first-line treatment. Antibiotics are
indicated only for appropriate treatment of underlying infections.
Phenytoin and carbamazepine are ineffective for febrile seizures.
Phenobarbital is sometimes used for prevention of recurrent febrile
seizures, but is not indicated as an initial therapy. Only 30%-50% of
children with an initial febrile seizure will have recurrent seizures.
205.
A 15-month-old male is brought to
your office 3 hours after the onset of an
increased respiratory rate and
wheezing. He has an occasional cough
and no rhinorrhea. His immunizations
are up to date and he attends day care
regularly. His temperature is 38.2C
(100.8F), respiratory rate 42/min, and
pulse rate 118 beats/min.
E. Oral high-dose amoxicillin (90 mg/kg/day), with close outpatient follow-up. The diagnosis
of community-acquired pneumonia is mostly based on the history and physical
examination. Pneumonia should be suspected in any child with fever, cyanosis, and any
abnormal respiratory finding in the history or physical examination. Children under 2
years of age who are in day care are at higher risk for developing community-acquired
pneumonia. Laboratory tests are rarely helpful in differentiating viral versus bacterial
etiologies and should not be routinely performed. Outpatient antibiotics are appropriate if
the child does not have a toxic appearance, hypoxemia, signs of respiratory distress, or
dehydration. Streptococcus pneumoniae is one of the most common etiologies in this age
group, and high-dose amoxicillin is the drug of choice.
The child is sitting quietly on his
mother's lap. His oxygen saturation is
94% on room air. On examination you
note inspiratory crackles in the left
lower lung field. The child appears to
be well hydrated and the remainder of
the examination, including an HEENT
examination, is normal. Nebulized
albuterol (AccuNeb) is administered
and no improvement is noted.
Which one of the following would be
most appropriate in the management
of this patient? (check one)
A. Laboratory evaluation
B. Inpatient monitoring, with no
antibiotics at this time
C. Hospitalization and intravenous
ceftriaxone (Rocephin)
D. Close outpatient follow-up, with no
antibiotics at this time
E. Oral high-dose amoxicillin (90
mg/kg/day), with close outpatient
follow-up
206.
A 15-year-old African-American male
presents to the emergency department
with a chief complaint of fever,
abdominal pain, nausea, and anorexia.
In addition to the usual laboratory
evaluation, which one of the following
imaging modalities would be most
helpful for confirming a diagnosis of
appendicitis? (check one)
A. Plain flat plate and upright
radiographs of the abdomen
B. An air contrast barium enema
C. Abdominal ultrasonography
D. A spiral CT scan of the abdomen
E. MRI of the abdomen
D. A spiral CT scan of the abdomen. A retrospective review of 650 patients with suspected
appendicitis showed a sensitivity of 97% and a specificity of 98% for spiral CT. In patients in
whom the clinical diagnosis was uncertain, sensitivity was 92% and specificity was 85%. Two
prospective studies comparing ultrasonography with spiral CT have favored spiral CT.
Ultrasonography is used in women who are pregnant and women in whom there is a high
degree of suspicion of gynecologic disease. Abdominal radiography has low specificity and
sensitivity for the diagnosis of acute appendicitis. Air contrast barium enema also has low
accuracy. Limitations of MRI include increased cost, decreased availability, and increased
examination time compared to CT.
207.
A 15-year-old male presents for a routine evaluation. He
has no complaints. He has a BMI of 30 kg/m2, which
places him in the 97th percentile for his age. The
remainder of his examination is normal; however, a
random blood glucose level is 162 mg/dL. Which one of
the following would be the most appropriate next step
for this patient? (check one)
A. Obtain a fasting blood glucose level
B. Start metformin (Glucophage), 500 mg daily, and
follow up in 4 weeks
C. Order a hemoglobin A1c level
D. Advise the patient to start a weight-loss program and
follow up in 4 weeks
208.
A 15-year-old white male is being evaluated after a fall
down one flight of stairs. He was transported by the local
rescue squad with his cervical spine immobilized. He
walked briefly at the scene and did not lose
consciousness. His only complaint is a mild, generalized
headache. One episode of vomiting occurred shortly after
the accident. No weakness or numbness has been noted.
Vital signs, mental status, and neurologic findings are
normal. Radiologic evaluation of the cervical spine is
remarkable only for an air-fluid level in the sphenoid
sinus. Which one of the following abnormalities is most
likely to be associated with this radiologic finding?
(check one)
A. Obtain a fasting blood glucose level. This patient should have
further testing for diabetes mellitus. Current recommendations for
diagnosing diabetes mellitus are based on either a fasting glucose level
or a 2-hour 75-g oral glucose tolerance test. A casual blood glucose level
>200 mg/dL is also diagnostic of diabetes mellitus in patients with
symptoms of hyperglycemia. If unequivocal hyperglycemia is not
present, the diagnosis must be confirmed by testing on another day.
Metformin can be used to treat diabetes mellitus in adolescents, but it
is not recommended for prevention in this age group. A diagnosis of
diabetes mellitus should be established prior to starting metformin.
Current recommendations for treating adolescents with type 2
diabetes mellitus include weight loss through dietary modification and
exercise.
A. A basilar skull fracture. A post-traumatic air-fluid level in the
sphenoid sinus is associated with basilar skull fractures. This finding is
frequently noted on cervical spine films. Orbital floor fractures may be
associated with double vision, fluid in the maxillary sinus, an air-fluid
level in the maxillary sinus, and diplopia. Epidural hematomas are
more frequently associated with skull fractures in the area of the
meningeal artery. Zygomatic arch fractures are more visible on
Towne's view. Characteristic swelling and lateral orbital bruising are
typically present. Mandible fractures may be associated with dental
misalignment or bleeding. Panoramic views are often diagnostic.
A. A basilar skull fracture
B. An orbital floor fracture
C. An epidural hematoma
D. A zygomatic arch fracture
E. A mandible fracture
209.
A 16-year-old female cross-country runner has pain
around both ankles. On examination, pain is elicited on
foot inversion and there is decreased motion of the hind
foot and peroneal tightness. A rigid flat foot also is
observed.
Which one of the following is the most likely diagnosis?
(check one)
A. Non-ossification of the os trigonum
B. Sever's apophysitis
C. Plantar fasciitis
D. Navicular stress fracture
E. Tarsal coalition
E. Tarsal coalition. Tarsal coalition is the fusion of two or more tarsal
bones. It occurs in mid-to late adolescence and is bilateral in 50% of
those affected. Pain occurs around the ankle, and there is decreased
range of motion of the hindfoot and pain on foot inversion on
examination. Os trigonum results from non-ossification of cartilage. It
usually is unilateral and causes palpable tenderness of the heel.
Sever's apophysitis is inflammation of the calcaneal apophysis, and
causes pain in the heel. Plantar fasciitis causes tenderness over the
anteromedial heel. Navicular stress fractures are tender over the
dorsomedial navicular.
210.
A 16-year-old highschool football player
plants his left foot to
make a cut and feels his
left leg give way. He
feels a pop in the knee,
followed by acute pain.
He is evaluated on the
field, and examination
with the knee flexed 20
reveals that the tibia
can be displaced farther
anteriorly than with the
uninvolved knee.
Which one of the
following conditions is
most likely?
(check one)
A. Patellar tendon
rupture
B. Posterior cruciate
ligament tear
C. Anterior cruciate
ligament tear
D. Tibial plateau
fracture
E. Patellar dislocation
211.
A 16-year-old male
accompanied by his
mother presents to
your outpatient clinic
with concerns about his
short stature and
"boyish" looks. He is a
sophomore in high
school but is frequently
mistaken for someone
much younger.
Radiographs reveal a
bone age of 14.7 years.
Which one of the
following would
suggest the need for
further evaluation?
(check one)
A. A family history of
delayed growth
B. Height below the
fifth percentile for age
C. Weight below the
fifth percentile for age
D. Prepubescent
testicular size
C. Anterior cruciate ligament tear. Anterior cruciate ligament (ACL) tears are the most common ligament
injury requiring surgery. Females have a significantly higher rate of ACL tears, with the majority of tears in
both men and women occurring without physical contact. In addition to the immediate problems, there is a
significant increase in premature osteoarthritis of the knee. Approximately 50% of patients with this injury
develop osteoarthritis in 10-20 years. Findings that help make the diagnosis of ACL tear include a
noncontact mechanism of injury, an audible popping sound, early swelling of the joint, and the inability to
participate in the game after the injury. Many patients can walk normally and can perform such straightplane activities as climbing stairs, biking, or jogging.
Physical examination using the Lachman test or pivot shift test can be used to further assess whether the
ligament is torn. MRI can be used to confirm the diagnosis, although it is not needed if the diagnosis is clear
from the history and examination. The other conditions listed are also sports-related knee injuries, but
have different mechanisms of injury or physical findings. Patients with patellar tendon rupture are unable
to fully extend their knee and examination shows a palpable defect in the patellar ligament and a highriding patella. While the mechanism of injury in patients with posterior cruciate ligament tears may be
similar to that of ACL injury, the examination would show posterior rather than anterior displacement of
the tibia when the knee is flexed at 90 (the posterior drawer sign). The mechanism of injury of tibial
plateau fractures in a healthy young male generally involves a highenergy collision causing a valgus force
with axial loading. Patients with patellar dislocations have symptoms similar to those of an ACL injury,
including an audible crack or pop and the feeling of the knee giving way after a twisting motion.
Immediately following the injury, however, examination would show an obvious deformity, but the patella
may spontaneously relocate prior to the on-field exam. There would be no instability on the Lachman
maneuver.
D. Prepubescent testicular size. Most cases of short stature are due to constitutional growth delay, a term
which implies that the child is normal but delayed in his development. A hallmark of this condition is being
below the fifth percentile for height for most of childhood. Usually these children are thin and have a
family history of delayed development. Bone age would be expected to be at least 2.5 standard deviations
below the mean for agematched peers of the same chronologic age. However, most experts agree that if no
signs of puberty are seen by 14 years of age (no breast development in girls, no testicular enlargement in
boys), then further workup for a more serious condition should be sought. Other indications for
evaluation would be no menarche in a girl by 16 years of age and underdeveloped genitalia in a boy 5
years after his first pubertal changes.
212.
A 16-year-old male is brought to
your office by his mother for
"stomachaches." On review of
systems, he also complains of
headaches, occasional bedwetting,
and trouble sleeping. His
examination is within normal
limits. His mother says that he is
often in the nurse's office at school,
and doesn't seem to have any
friends. After some questions from
you, he admits to being called
names and teased at school. Which
one of the following would be most
appropriate? (check one)
C. Explore whether his school counselor has a process to address this problem. Childhood
bullying has potentially serious implications for bullies and their targets. The target children
are typically quiet and sensitive, and may be perceived to be weak and different. Children
who say they are being bullied must be believed and reassured that they have done the right
thing in acknowledging the problem. Parents should be advised to discuss the situation with
school personnel. Bullying is extremely difficult to resolve. Confronting bullies and expecting
victims to conform are not successful approaches. The presenting symptoms are not
temporary, and in fact can progress to serious issues such as suicide, substance abuse, and
victim-to-bully transformation. These are not signs or symptoms of thyroid disease. The
Olweus Bullying Program developed in Norway is a well documented, effective program for
reducing bullying among elementary and junior-high-school students by altering social norms
and by changing school responses to bullying incidents, including efforts to protect and
support victims. Students who have been bullied regularly are most likely to carry weapons to
school, be in frequent fights, and eventually be injured.
A. Explain that he must try to
conform to be more popular
B. Explain that these symptoms are
a stress reaction and will lessen
with time
C. Explore whether his school
counselor has a process to address
this problem
D. Order a TSH level
213.
A 16-year-old male is brought to
your office by his mother for
"stomachaches." On the review of
systems he also complains of
headaches, occasional bedwetting,
and trouble sleeping. His
examination is within normal
limits. His mother says that he is
often in the nurse's office at school,
and doesn't seem to have any
friends. When you discuss these
problems with him, he admits to
being teased and called names at
school.
Which one of the following would
be most appropriate? (check one)
A. Explain that he must try to
conform to be more popular
B. Explain that these symptoms are
a stress reaction and will lessen
with time
C. Explore whether his school
counselor has a process to address
this problem
D. Order a TSH level
C. Explore whether his school counselor has a process to address this problem. Childhood
bullying has potentially serious implications for bullies and their targets. The target children
are typically quiet and sensitive, and may be perceived to be weak and different. Children
who say they are being bullied must be believed and reassured that they have done the right
thing in acknowledging the problem. Parents should be advised to discuss the situation with
school personnel.
Bullying is extremely difficult to resolve. Confronting bullies and expecting victims to conform
are not successful approaches. The presenting symptoms are not temporary, and in fact can
progress to more serious problems such as suicide, substance abuse, and victim-to-bully
transformation. These are not signs or symptoms of thyroid disease.
The Olweus Bullying Prevention Program developed in Norway is a well documented, effective
program for reducing bullying among elementary and middle-school students by altering
social norms and by changing school responses to bullying incidents, including efforts to
protect and support victims. Students who have been bullied regularly are more likely to
carry weapons to school, be in frequent fights, and eventually be injured.
214.
A 16-year-old sexually active nulliparous
white female complains of pelvic pain and
vaginal discharge. On examination she is
found to have a temperature of 39.8
degrees C (102.0 degrees F), pain with
movement of the cervix, and tenderness
and a mass in the right adnexa. According
to CDC guidelines, which one of the
following treatments would be
appropriate? (check one)
D. Hospitalization for treatment with cefoxitin intravenously and doxycycline orally or
intravenously, then doxycycline orally twice a day to complete 14 days of treatment.
Patients with PID and tubo-ovarian abscess and high fever should be hospitalized and
treated for at least 24 hours with intravenous antibiotics. Amoxicillin and penicillin G
procaine are no longer recommended because of the increasing prevalence of
penicillinase-producing and chromosomally-mediated resistant Neisseria
gonorrhoeae. If cefoxitin is used intramuscularly for outpatient treatment, it should be
combined with probenecid. If ceftriaxone is used for outpatient treatment,
probenecid is not required. Reexamination should be done within 3 days of initiation
of therapy.
A. Outpatient treatment with penicillin G
procaine (Wycillin) intramuscularly;
probenecid (Benemid) orally; plus
doxycycline (Vibramycin) orally for 14 days
and reexamination in 3 days
B. Outpatient treatment with ceftriaxone
(Rocephin) intramuscularly; probenecid
orally; plus doxycycline twice a day for 14
days and reexamination in 1 week
C. Outpatient treatment with cefoxitin
(Mefoxin) intramuscularly; plus
doxycycline twice a day for 14 days and
reexamination in 10 days
D. Hospitalization for treatment with
cefoxitin intravenously and doxycycline
orally or intravenously, then doxycycline
orally twice a day to complete 14 days of
treatment
215.
A 16-year-old white male is seen for a
preparticipation sports examination. His
height is 183 cm (72 in), his weight is 64 kg
(141 lb), and he appears to have long arms.
A physical examination reveals a high
arched palate, kyphosis, myopia, and
pectus excavatum. Which one of the
following valvular abnormalities is most
likely in this patient? (check one)
A. Mitral stenosis
B. Pulmonic stenosis
C. Aortic stenosis
D. Aortic insufficiency
E. Bicuspid aortic valve
D. Aortic insufficiency. This adolescent has findings of Marfan syndrome. It is
associated with arachnodactly, arm span greater than height, a high arched palate,
kyphosis, lenticular dislocation, mitral valve prolapse, myopia, and pectus excavatum.
Cardiac examination may reveal an aortic insufficiency murmur, or a murmur
associated with mitral valve prolapse. Cardiovascular defects are progressive, and
aortic root dilation occurs in 80%-100% of affected individuals. Aortic regurgitation
becomes more common with increasing age.
216.
A 17-year-old female sees you for a
preparticipation evaluation. She has
run 5 miles a day for the last 6
months, and has lost 6 lb over the
past 2 months. Her last menstrual
period was 3 months ago. Other than
the fact that she appears to be slightly
underweight, her examination is
normal.
To fit the criteria for the female
athlete triad, she must have which
one of the following? (check one)
A. A formal diagnosis of an eating
disorder
B. Amenorrhea for 1 year
C. A Z-score on bone-density testing
of -2.5 or less
D. Withdrawal bleeding after
progesterone administration
E. A history of a stress fracture
resulting from minimal trauma
217.
A 17-year-old soccer player presents
for a preparticipation examination.
His family history is significant for the
sudden death of his 12-year-old sister
while playing basketball, and for his
mother and maternal grandmother
having recurrent syncopal episodes.
His medical history and examination
are completely normal. Prior to
approving his participation in sports,
which one of the following is
recommended? (check one)
A. A resting EKG
B. A stress EKG
C. An echocardiogram
D. Pulmonary function testing
E. No further evaluation
E. A history of a stress fracture resulting from minimal trauma. The initial definition of the
female athlete triad was amenorrhea, osteoporosis, and disordered eating. The American
College of Sports Medicine modified this in 2007, emphasizing that the triad components
occur on a continuum rather than as individual pathologic conditions. The definitions have
therefore expanded. Disordered eating is no longer defined as the formal diagnosis of an
eating disorder. Energy availability,defined as dietary energy intake minus exercise energy
expenditures, is now considered a risk factor for the triad, as dietary restrictions and
substantial energy expenditures disrupt pituitary and ovarian function.
Primary amenorrhea is defined as lack of menstruation by age 15 in females with secondary
sex characteristics. Secondary amenorrhea is the absence of three or more menstrual cycles
in a young woman previously experiencing menses. For those with secondary amenorrhea,
a pregnancy test should be performed. If this is not conclusive, a progesterone challenge
test may be performed. If there is withdrawal bleeding, the cause would be anovulation.
Those who do not experience withdrawal bleeding have hypothalamic amenorrhea, and fit
one criterion for the triad.
Athletes who have amenorrhea for 6 months, disordered eating, and/or a history of a stress
fracture resulting from minimal trauma should have a bone density test. Low bone mineral
density for age is the term used to describe at-risk female athletes with a Z-score of -1 to -2.
Osteoporosis is defined as having clinical risk factors for experiencing a fracture, along with
a Z-score <-2.
A. A resting EKG. A family history of sudden death and recurrent syncope is highly
suspicious for genetic long-QT syndrome. It is best diagnosed with a resting EKG that shows
a QTc >460 msec in females and >440 msec in males. This syndrome especially places young
people at risk for sudden death. Management may include -blockers, an implantable
cardioverter-defibrillator, and no participation in competitive sports.
218.
A 17-year-old white female at 20 weeks gestation presents
with a 2-day history of painful vesicular lesions on her labia.
This is the first time she has ever had this problem. Her last
sexual contact was 10 days ago. She has also had a low-grade
fever, malaise, headache, and mild, diffuse abdominal pain.
On examination she has vesicles and erythematous papules
on the labia bilaterally. A few firm, tender inguinal nodes are
also noted.
Which one of the following tests is most sensitive for
confirming the diagnosis? (check one)
A. A Papanicolaou smear of the lesions
B. Amniocentesis
C. Serologic studies
D. Viral polymerase chain reaction (PCR) testing
E. A Tzanck test
219.
A 17-year-old white female has a history of anorexia nervosa,
and weight loss has recently been a problem. The patient is an
academically successful high-school student who lives with
her parents and a younger sibling. Her BMI is 17.4 kg/m2 . Her
serum electrolyte levels and an EKG are normal.
Which one of the following interventions is most likely to be
successful? (check one)
A. Family-based treatment
B. Adolescent-focused individual therapy
C. Fluoxetine (Prozac)
D. Phenelzine (Nardil)
E. Desipramine (Norpramin)
D. Viral polymerase chain reaction (PCR) testing. Diagnosis and
appropriate treatment of genital herpes during pregnancy is
particularly important because of the high mortality in neonates
who contract herpes during delivery and then develop
disseminated infection. In those who survive, there is a very high
risk of serious neurologic sequelae.
HSV is acquired by deposition of the virus on a break in the skin
or mucous membranes during close physical contact with an
infected person. Neonatal infection most commonly results from
transmission via the birth canal, although transplacental
transmission can occur. The risk of HSV infection in the neonate is
higher during an episode of primary genital herpes than during a
recurrent episode.
DNA polymerase chain reaction testing is 95% sensitive as long as
an ulcer is present, and has a specificity of 90%. The diagnosis is
established by culturing the virus from an infected lesion. A
Tzanck prep and Papanicolaou smear can detect cellular changes,
but both have low sensitivity. Serologic diagnosis is mainly an
epidemiologic tool and has limited clinical usefulness. Cultures of
the virus by amniocentesis have shown both false-positive and
false-negative results.
A. Family-based treatment. Family-based treatment for the
adolescent with anorexia nervosa has been found to provide
superior results when compared with individual adolescentfocused therapy (SOR B). Antidepressants have not been
successful. They may be indicated for coexisting conditions, but
this is more common with bulimia.
220.
A 17-year-old white female visits you for a
physical examination prior to entering
college. During the review of systems her
only complaint is cyclic lower abdominal
cramps around the onset of menstruation.
She reports that pain has been present to
some degree with most of her periods since
about 6 months after menarche. The pain is
often severe enough for her to miss school.
Each episode lasts 24-48 hours and is
somewhat relieved by rest and
acetaminophen. Her menstrual history is
otherwise normal. She denies ever being
sexually active and tells you that she has
received little empathy from her mother,
who had similar symptoms as an adolescent
that improved after her first pregnancy.
Pelvic and rectal examinations are within
normal limits. Which one of the following
management choices would be appropriate
at this time? (check one)
A. Referral for hysterosalpingography
B. Referral for psychological counseling
C. Danazol (Danocrine)
D. Acetaminophen/hydrocodone (Vicodin
HP)
E. Naproxen sodium (Anaprox)
E. Naproxen sodium (Anaprox). The patient's history is typical of primary
dysmenorrhea, defined as severe cramping pain in the lower abdomen that occurs
during menses; it may also occur prior to the onset of menses in the absence of
associated pelvic pathology. Although many women complain of pain beginning with
the first cycle, symptoms usually begin at the onset of ovulation around 6-12 months
after menarche. Symptoms typically last 48 hours or less, but sometimes may last up
to 72 hours. It is common to find daughters with dysmenorrhea whose mothers had
the same symptoms. Additionally, the symptoms of primary dysmenorrhea often
resolve after the first pregnancy. In this patient, who has no history suggesting an
emotional disorder, there is no need for psychological counseling at this time. Further
evaluation could include ultrasonography to rule out causes of dysmenorrhea such
as uterine leiomyomata, adnexal masses, and endometrial polyps. However, a trial of
symptomatic therapy is most reasonable before other invasive studies, such as a
laparoscopic examination or a hysterosalpingogram, are ordered. It is not reasonable
to begin danazol without a diagnosis of endometriosis, which is by definition
secondary dysmenorrhea. Since neither inhibits prostaglandin synthetase,
acetaminophen (which she had already tried without complete relief) combined with
a narcotic is not an appropriate management strategy. Multiple placebo-controlled
studies have shown that NSAIDs such as naproxen, at the onset of symptoms,
provide significant relief of primary dysmenorrhea compared to placebo.
221.
An 18-month-old white male has been brought into your office multiple times
over the past year with a reported fever of over 101 degrees F (38 degrees C). The
child's reported temperatures at home have usually been higher than those
measured at the time of the office visit. The remainder of the history is usually
unremarkable. The child has a sibling who is in good health, but another sibling
died several years ago for unknown reasons. On two occasions you diagnosed
acute otitis media and acute bronchitis. However, at most visits the child has not
had any abnormal physical findings. Repeated laboratory studies have been
within normal limits, including complete blood counts, erythrocyte
sedimentation rates, blood cultures, chest radiographs, and urinalyses. Almost
always, the mother has reported little reduction in fever with age-appropriate
doses of acetaminophen or ibuprofen. At the last office visit the child's
temperature was measured at 40.6 degrees C (105.1 degrees F). Although the
examination was once again unrevealing, it was decided to hospitalize the child
for close observation and evaluation by an infectious disease consultant. Closedcircuit television observation in the hospital showed the mother putting the
thermometer into hot water before a nurse came to record the patient's
temperature. During the hospitalization you make a diagnosis. Which one of the
following is a strong indicator of the suspected final diagnosis? (check one)
A. The child has seen no other health-care provider but you
B. Both parents have been involved with each office visit
C. The child is afebrile while staying at the day-care center
D. The parents have resisted having painful or risky diagnostic tests performed on
the child
222.
An 18-year-old female basketball player comes to your office the day after
sustaining an inversion injury to her ankle. She says she treated the injury
overnight with rest, ice, compression, and elevation. You examine her and
diagnose a moderate to severe lateral ankle sprain. In addition to rehabilitative
exercises, you advise (check one)
A. a short-term cast
B. a posterior splint that allows no flexion or extension
C. a semi-rigid stirrup brace (Air-Stirrup, "Aircast")
D. an elastic bandage
E. no external brace or support
223.
An 18-year-old male comes to your office because of the recent onset of
recurrent, unpredictable episodes of palpitations, sweating, dyspnea,
gastrointestinal distress, dizziness, and paresthesias. His physical examination is
unremarkable except for moderate obesity. Laboratory findings, including a
CBC, blood chemistry profile, and thyroid-stimulating hormone (TSH) level,
reveal no abnormalities.
The most likely diagnosis is: (check one)
A. mitral valve prolapse
B. paroxysmal supraventricular tachycardia
C. pheochromocytoma
D. generalized anxiety disorder
E. panic disorder
C. The child is afebrile while staying at the
day-care center. This is a characteristic
presentation of factitious disorder by proxy,
or what is commonly known as Munchausen
syndrome by proxy. Warning signs for this
disorder include the episodes of illness
beginning only when the child is, or has
recently been, with the parent; the parent
taking the child to numerous caregivers,
resulting in multiple diagnostic evaluations
but neither cure nor definitive diagnosis; the
other parent (usually the father) being
notably uninvolved despite the ostensible
health crises; the parent not being assured
by normal test results and continually
advocating for painful or risky diagnostic
tests for the child; the child persistently failing
to tolerate or respond to usual medical
therapies; and another child in the family
having an unexplained illness or childhood
death.
C. a semi-rigid stirrup brace (Air-Stirrup,
"Aircast"). In acute ankle sprains, functional
treatment with a semi-rigid brace that allows
flexion and axtension, or a soft lace-up brace
is recommended over immobilization. Casting
or posterior splinting is no longer
recommended. Elastic bandaging does not
offer the same lateral and medial support.
External ankle support has been shown to
improve proprioception.
E. panic disorder. Panic disorder typically
presents with the symptoms described, in
late adolescence or early adulthood. The
attacks are sporadic and last 10-60 minutes.
Generalized anxiety disorder is more
common, and common symptoms include
restlessness, fatigue, muscle tension,
irritability, difficulty concentrating, and sleep
disturbance. Patients with mitral valve
prolapse usually have an abnormal cardiac
examination. Pheochromocytoma is
associated with headache and hypertension,
and usually occurs in thin patients.
Paroxysmal supraventricular tachycardia is
usually not associated with gastrointestinal
distress or paresthesias.
224.
An 18-year-old male
presents with a sore throat,
adenopathy, and fatigue. He
has no evidence of airway
compromise. A heterophil
antibody test is positive for
infectious mononucleosis.
Appropriate management
includes which one of the
following? (check one)
225.
E. Avoidance of contact sports. Infectious mononucleosis presents most commonly with a sore throat,
fatigue, myalgias, and lymphadenopathy, and is most prevalent between 10 and 30 years of age. Both
an atypical lymphocytosis and a positive heterophil antibody test support the diagnosis, although
false-negative heterophil testing is common early in the disease course. The cornerstone of treatment
for mononucleosis is supportive, including hydration, NSAIDs, and throat sprays or lozenges.
In general, corticosteroids do not have a significant effect on the clinical course of infectious
mononucleosis, and they should not be used routinely unless the patient has evidence of acute
airway obstruction. Antihistamines are also not recommended as routine treatment for
mononucleosis. The use of acyclovir has shown no consistent or significant benefit, and antiviral drugs
are not recommended.
A. A corticosteroid
B. An antihistamine
C. An antiviral agent
D. Strict bed rest
E. Avoidance of contact
sports
There is also no evidence to support bed rest as an effective management strategy for mononucleosis.
Given the evidence from other disease states, bed rest may actually be harmful.
An 18-year-old male seen in
your office is found to be
overweight and to have
acanthosis nigricans. Both
of his parents have a history
of diabetes mellitus. His
fasting plasma glucose level
is 111 mg/dL (N <100). Which
one of the following is the
correct diagnosis? (check
one)
A. Prediabetes. This patient has prediabetes, which is defined as having a fasting plasma glucose level
of 101-125 mg/dL. These patients are at high risk for developing diabetes mellitus later in life.
Prediabetes is associated with metabolic syndrome, and weight loss, exercise, and certain
pharmacologic agents have been shown to prevent or delay the subsequent development of diabetes
mellitus. Diabetes mellitus is diagnosed in three ways: symptoms of diabetes (polyuria, polydipsia,
unexplained weight loss) plus a random plasma glucose level 200 mg/dL; a fasting plasma glucose
level 126 mg/dL; or a glucose level 200 mg/dL on a 2-hour 75-g oral glucose tolerance test. It is
important to note that in the absence of unequivocal hyperglycemia the diagnosis must be confirmed
by repeat testing on a subsequent day. Once the diagnosis of diabetes is confirmed, further testing is
needed to differentiate between type 1, type 2, and maturity-onset diabetes of youth.
A. Prediabetes
B. Type 1 diabetes mellitus
C. Type 2 diabetes mellitus
D. Maturity-onset diabetes
of the young
Although most patients will not have a palpably enlarged spleen on examination, it is likely that all, or
nearly all, patients with mononucleosis have splenomegaly. This was demonstrated in a small study in
which 100% of patients hospitalized for mononucleosis had an enlarged spleen by ultrasound
examination, whereas only 17% of patients with splenomegaly have a palpable spleen. Patients should
be advised to avoid contact- or collision-type activities for 3-4 weeks because of the increased risk of
rupture.
226.
An 18-year-old primigravida at 38 weeks' gestation complains of a headache. Her
blood pressure is 130/92 mm Hg. The fetal heart rate is 140 beats/min. A urine
dipstick shows 2+ protein. Laboratory Findings Hemoglobin 10.8 g/dL (N 12.016.0) Hematocrit 32.4% (N 36.0-46.0) Platelets 110,000/mm3 (N 150,000-400,000)
WBCs 14,900/mm3 (N 4000-10,000) Creatinine 0.5 mg/dL (N 0.8-1.3) AST (SGOT) 31
U/L (N 0-37) ALT (SGPT) 60 U/L (N 0-65) LDH 240 U/L (N 100-190) Bilirubin 1.9 mg/dL
(N 0.0-1.0) A non stress test is reactive and the amniotic fluid index is 9.4 (N 8.020.0). The patient is admitted for further testing. After 24 hours repeat testing
shows the following: Hemoglobin 9.8 g/dL Hematocrit 30.2% Platelets
92,000/mm3 WBCs 15,200/mm3 Creatinine 0.6 mg/dL AST (SGOT) 72 U/L ALT (SGPT)
98 U/L LDH 620 U/L Bilirubin 2.4 mg/dL 24-hour urine protein 2400 mg Which one
of the following would be the most appropriate course of action at this point?
(check one)
D. Induction of labor with oxytocin (Pitocin) if
the cervix is favorable. This patient has
hemolysis, elevated liver enzymes, and low
platelets (HELLP) syndrome and needs to be
delivered. There is no reason to delay
delivery in a term pregnancy. HELLP
syndrome is a form of severe preeclampsia.
If the patient has a favorable cervical
examination, labor induction with oxytocin is
appropriate. If the cervix is unfavorable,
cesarean delivery should be considered to
expedite delivery.
A. Continued monitoring, repeating the 24-hour urine collection, and repeating
the laboratory studies tomorrow
B. Immediate delivery by cesarean section
C. Discharge to home on bed rest, with close follow-up
D. Induction of labor with oxytocin (Pitocin) if the cervix is favorable
227.
An 18-year-old single white female at 30 weeks' gestation presents to the hospital
with uterine contractions 10 minutes apart. Her previous pregnancy 18 months
ago resulted in a preterm birth at 29 weeks' gestation. The most accurate test to
determine whether this patient will need hospitalization and tocolysis would be:
(check one)
A. Serum corticotropin-releasing hormone
B. Maternal serum alpha-fetoprotein
C. Serum human chorionic gonadotropin (hCG)
D. Salivary estriol concentration
E. Vaginal fetal fibronectin
228.
An 18-year-old white female presents with small, localized warts on the vulva and
lower vaginal mucosa. She wants to avoid injections and surgical treatment if
possible. Which one of the following is an acceptable topical agent for treating
these vaginal lesions? (check one)
A. Trichloroacetic acid
B. Podofilox gel (Condylox)
C. Imiquimod cream (Aldara)
D. Interferon
E. Podophyllin 25% solution in alcohol (Podocon-25, Podofin)
E. Vaginal fetal fibronectin. Of the
biochemical markers listed, the most clinically
useful test to differentiate women who are at
high risk for impending preterm delivery
from those who are not is the fetal
fibronectin in cervical or vaginal secretions.
In symptomatic women, this is most accurate
in predicting spontaneous preterm delivery
within 7-10 days. It is less accurate in those
who are asymptomatic. If the fetal
fibronectin is negative, it may be possible to
avoid interventions such as hospitalization,
tocolysis, and corticosteroid administration.
A. Trichloroacetic acid. Trichloroacetic acid is
acceptable for use on vaginal mucosa. It is
also acceptable for use when pregnancy is a
possibility. Professional application is
necessary. Podofilox and podophyllin in
alcohol are not safe for use on mucosa.
Imiquimod cream is also not approved for
mucosal use. Interferon requires injection.
229.
A 19-year-old college freshman consults you at the
request of her cross-country coach because she has
not had a period in 2 of the last 3 months. She notes
that her current training regimen is much more
intense than in high school last year. She has an
appropriate body image and denies caloric
restriction. A pregnancy test at the student health
center was negative. On examination she is lean and
highly trained. Her examination is otherwise normal.
Which one of the following would be the most
appropriate recommendation for this patient?
(check one)
A. Estrogen supplementation
B. Cyclic oral contraceptive pills
C. Increased caloric intake
D. Bisphosphonate therapy
E. Discontinuation of elite-level athletics
230.
A 19-year-old college student comes to your office
with her mother. The mother reports that her
daughter has frequently been observed engaging in
binge eating followed by induced vomiting. She has
also admitted to using laxatives to prevent weight
gain.
Which one of the following laboratory abnormalities
is most likely to be found in this patient?
(check one)
A. Hypokalemia
B. Hypoglycemia
C. Hyponatremia
D. Hypercalcemia
E. Hypermagnesemia
C. Increased caloric intake. This patient has exercise-related
oligomenorrhea, but does not have the eating disorder that characterizes
the female athlete triad. Menstrual problems in athletes do correlate with
bone density loss and impaired recovery from exercise. Additionally,
menstrual irregularity of varying severity is extremely common in female
distance runners, perhaps affecting as many as 60%. Hormonal
manipulation has not been shown to affect bone density, though it may
produce withdrawal bleeding. Bisphosphonate therapy has been shown
to be ineffective, and is not recommended in women of child-bearing age.
The main issue in well-nourished female athletes seems to be that energy
intake is not increased to match energy expenditures at high levels of
training. Unlike those with the female athlete triad, there is little evidence
that athletes without eating disorders suffer substantial harm from
exercise-induced menstrual problems. Ending an athletic career for this
reason alone is not justified.
A. Hypokalemia. The patient described is likely suffering from bulimia.
These patients use vomiting, laxatives, or diuretics to prevent weight gain
after binge eating. This often causes a loss of potassium, leading to
weakness, cardiac arrhythmias, and respiratory difficulty. The levels of
other electrolytes are not as dramatically affected.
231.
A 19-year-old college student comes to your office with
significant pain in his right great toe that is making it difficult
for him to walk. He has never had this problem before.
When you examine him you find increased swelling with
marked erythema and seropurulent drainage and ulceration
of the medial nail fold. The toe is very tender to touch,
particularly when pressure is applied to the tip of the toe. The
most appropriate initial management would be: (check one)
A. oral antibiotics that cover common skin flora, for 5-7 days
B. soaking the toe in warm, soapy water for 10-20 minutes
twice daily, followed by application of a topical antibiotic,
with a return visit in 3-5 days
C. elevation of the nail with a wisp of cotton
D. partial avulsion of the medial nail plate and phenolization
of the matrix at this visit
E. partial avulsion of both the medial and lateral nail plates at
this visit
232.
A 19-year-old female high-school student is brought to your
office by a friend who is concerned about the patient having
cut her wrists. The patient denies that she was trying to kill
herself, and states that she did this because she "just got so
angry" at her boyfriend when she caught him sending a text
message to another woman. She denies having a depressed
mood or anhedonia, and blames her fluctuating mood on
everyone who "keeps abandoning her," making her feel like
she's "nothing." She admits that she has difficulty controlling
her anger. Her sleep quality and pattern appear normal, as
does her appetite. She denies hallucinations or delusions. The
wounds on her wrists appear superficial and there is evidence
of previous cutting behavior on her forearms. Her vital signs
are stable.
Which one of the following would be most beneficial for this
patient? (check one)
A. Clonazepam (Klonopin)
B. Fluoxetine (Prozac)
C. Quetiapine (Seroquel)
D. Inpatient psychiatric admission
E. Psychotherapy
D. partial avulsion of the medial nail plate and phenolization of the
matrix at this visit. This ingrown nail meets the criteria for
moderate severity: increased swelling, seropurulent drainage,
infection, and ulceration of the nail fold. In these cases, antibiotics
before or after phenolization of the matrix do not decrease
healing time, postoperative morbidity, or recurrence rates (SOR
B). A conservative approach, elevating the nail edge with a wisp of
cotton or a gutter splint, is reasonable in patients with a mild to
moderate ingrown toenail who do not have significant pain,
substantial erythema, or purulent drainage.
Either immediate partial nail avulsion followed by phenolization,
or direct surgical excision of the nail matrix is effective for the
treatment of ingrown nails (SOR B). Pretreatment with soaking
and antibiotics has not been demonstrated to add therapeutic
benefit or to speed resolution. Several studies demonstrate that
once the ingrown portion of the nail is removed and matricectomy
is performed, the localized infection will resolve without the need
for antibiotic therapy. Bilateral partial matricectomy maintains the
functional role of the nail plate (although it narrows the nail plate)
and should be considered in patients with a severe ingrown
toenail or to manage recurrences.
E. Psychotherapy. This patient displays most of the criteria for
borderline personality disorder. This is a maladaptive personality
type that is present from a young age, with a strong genetic
predisposition. It is estimated to be present in 1% of the general
population and involves equal numbers of men and women;
women seek care more often, however, leading to a
disproportionate number of women being identified by medical
providers.
Borderline personality disorder is defined by high emotional
lability, intense anger, unstable relationships, frantic efforts to
avoid a feeling of abandonment, and an internal sense of
emptiness. Nearly every patient with this disorder engages in selfinjurious behavior (cutting, suicidal gestures and attempts), and
about 1 in 10 patients eventually succeeds in committing suicide.
However, 90% of patients improve despite having made
numerous suicide threats. Suicidal gestures and attempts peak
when patients are in their early 20s, but completed suicide is most
common after age 30 and usually occurs in patients who fail to
recover after many attempts at treatment. In contrast, suicidal
actions such as impulsive overdoses or superficial cutting, most
often seen in younger patients, do not usually carry a high shortterm risk, and serve to communicate distress.
Inpatient hospitalization may be an appropriate treatment option
if the person is experiencing extreme difficulties in living and daily
functioning, and pharmacotherapy may offer a mild degree of
symptom relief. While these modalities have a role in certain
patients, psychotherapy is considered the mainstay of therapy,
especially in a relatively stable patient such as the one described.
233.
A 19-year-old female runner has a 1-week history of
constant groin pain. There is limited hip motion on
flexion and internal rotation of the right hip.
Radiographs of the hip and pelvis are normal. Which
one of the following is the most likely diagnosis?
(check one)
A. Iliotibial band syndrome
B. Stress fracture of the right femoral neck
C. Osteitis pubis
D. Pelvic inflammatory disease
234.
A 19-year-old white female presents for an initial
family planning evaluation. Specifically, she is
interested in oral contraception. She is not
presently sexually active, but has a steady boyfriend.
She has no contraindications to oral contraceptive
use. She has mild acne vulgaris. You discuss possible
side effects and benefits of combined oral
contraceptives, including improvement of her acne.
Which one of the following is also associated with
oral contraceptive use? (check one)
B. Stress fracture of the right femoral neck. Stress fractures of the femoral
neck are most commonly seen in military recruits and runners. They
present with persistent groin pain, and limited hip flexion and internal
rotation. Radiographs may be normal early. Iliotibial band syndrome also
occurs in runners and presents with stinging pain over the lateral femoral
epicondyle. Osteitis pubis occurs in distance runners and presents with
pain in the anterior pelvic area and tenderness over the symphysis pubis.
Pelvic inflammatory disease is associated with abdominal pain and fever.
B. Decreased risk of ovarian cysts. Women who take combination oral
contraceptives have a reduced risk of both ovarian and endometrial cancer.
This benefit is detectable within a year of use and appears to persist for
years after discontinuation. Other benefits include a reduction in
dysfunctional uterine bleeding and dysmenorrhea; a lower incidence of
ovarian cysts, ectopic pregnancy and benign breast disease; and an
increase in hemoglobin concentration. Many women also benefit from the
convenience of menstrual regularity. All combination oral contraceptives
raise sex hormone-binding globulin and decrease free testosterone
concentrations, which can lead to improvement in acne.
A. Increased risk of ovarian cancer
B. Decreased risk of ovarian cysts
C. Increased risk for ectopic pregnancy
D. Increased incidence of dysmenorrhea
235.
A 19-year-old white male with a history of fever,
fatigue, weight loss, and mild diarrhea of 2 months'
duration is found to have a palpable mass in the
right lower quadrant of the abdomen. The most
likely diagnosis is: (check one)
A. Crohn's disease (regional enteritis)
B. Ulcerative colitis
C. Amebic colitis
D. Diverticulitis
E. Lymphoma
A. Crohn's disease (regional enteritis). When Crohn's disease affects
primarily the distal small intestine (regional enteritis), a most characteristic
clinical pattern emerges. A young person, usually in the second or third
decade, will present with a period of episodic abdominal pain, largely
postprandial and often periumbilical, occasionally with low-grade fever and
mild diarrhea. Anorexia, nausea, and vomiting may also be present. Weight
loss is frequent. Some patients may be aware of tenderness in the right
lower quadrant and even of a palpable mass in that region.
236.
A 20-month-old male presents with a history of a
fever up to 38.5C (101.3F), pulling at both ears,
drainage from his right ear, and a poor appetite
following several days of nasal congestion. This is
his first episode of acute illness, and he has no
history of drug allergies.
C. Amoxicillin. This patient has acute bilateral otitis media, with presumed
tympanic membrane perforation, and qualifies by any criterion for treatment
with antibiotics. Amoxicillin, 80-90 mg/kg/day, should be the first-line antibiotic
for most children with acute otitis media (SOR B). The other medications listed
are either ineffective because of resistance (e.g., penicillin), are second-line
treatments (e.g., amoxicillin/clavulanate), or should be used in patients with a
penicillin allergy or in other special situations.
The fever is confirmed on examination and the
child is found to be fussy but can be distracted.
He is eating adequately and shows no signs of
dehydration. Positive findings include mild nasal
congestion, a purulent discharge from the right
auditory canal, and a red, bulging, immobile
tympanic membrane in the left auditory canal.
Which one of the following would be first-line
treatment for this patient? (check one)
A. Ceftriaxone (Rocephin)
B. Amoxicillin/clavulanate (Augmentin)
C. Amoxicillin
D. Azithromycin (Zithromax)
E. Penicillin VK
237.
A 20-year-old college wrestler is seen for an
examination prior to the wrestling season. He
tells you that some friends have told him he
should start taking dehydroepiandrosterone
(DHEA), and he asks for your advice.
Which one of the following is true about the
effects of this drug?
(check one)
D. It does not enhance either performance or muscle strength.
Dehydroepiandrosterone (DHEA) is illegal under the Anabolic Steroid Control
Act of 2004, and is prohibited by the NCAA and the International Olympic
Committee. Like androstenedione, DHEA is a precursor to testosterone, but
neither of these substances has been shown to enhance either performance
or strength. In fact, they increase serum estrogen and luteinizing hormone
levels.
A. It enhances performance but not muscle
strength
B. It enhances muscle strength but not
performance
C. It enhances both performance and muscle
strength
D. It does not enhance either performance or
muscle strength
238.
A 20-year-old female is seen for follow-up 6 weeks
after delivery. Her pregnancy was complicated by
preeclampsia. Her examination is unremarkable.
This patient will be at increased risk for which
one of the following in midlife? (check one)
A. Breast cancer
B. Diabetes mellitus
C. Hypothyroidism
D. Kidney disease
E. Hypertension
E. Hypertension. Preeclampsia affects as many as 5% of first pregnancies and is
manifested as hypertension, proteinuria, edema, and rapid weight gain after
20 weeks gestation. Very young mothers and those over age 35 have a higher
risk. Patients who have had preeclampsia have a fourfold increased risk of
hypertension and a twofold increased risk of ischemic heart disease, stroke,
and venous thromboembolism. There does not appear to be an association
between preeclampsia and cancer, breast cancer in particular.
239.
A 20-year-old female long-distance runner presents with
a 3-month history of amenorrhea. A pregnancy test is
negative, and other blood work is normal. She has no
other medical problems and takes no medications. With
respect to her amenorrhea, you advise her (check one)
A. to increase her caloric intake
B. that this is a normal response to training
C. to begin an estrogen-containing oral contraceptive
D. to stop running
240.
A 20-year-old female presents with a sudden onset of
fever, chills, and headache of 2 days duration, and now
has a pink blanching rash. The rash covers most of her
body, including the palms of her hands and the soles of
her feet, but not including her face. She recently
returned from a camping trip, but has had no recent
contact with anyone who has been ill. Which one of the
following would be the most appropriate treatment for
this patient's symptoms? (check one)
A. to increase her caloric intake. Amenorrhea is an indicator of
inadequate calorie intake, which may be related to either reduced food
consumption or increased energy use. This is not a normal response to
training, and may be the first indication of a potential developing
problem. Young athletes may develop a combination of conditions,
including eating disorders, amenorrhea, and osteoporosis (the female
athlete triad). Amenorrhea usually responds to increased calorie intake
or a decrease in exercise intensity. It is not necessary for patients such
as this one to stop running entirely, however.
A. Doxycycline, 100 mg twice daily for 10 days. This is a classic description
of rickettsial illness (in the United States this would most likely be Rocky
Mountain Spotted Fever): a history of outdoor activity, the sudden
onset of fever, chills, and rash on the palms of the hands and the soles
of the feet. Penicillin, cephalexin, and azithromycin do not cover
rickettsia. Reassurance would be inappropriate because this condition
can be life threatening and should always be treated.
A. Doxycycline, 100 mg twice daily for 10 days
B. Azithromycin (Zithromax), 500 mg daily for 3 days
C. Cephalexin (Keflex), 500 mg twice daily for 10 days
D. Penicillin VK, 500 mg twice daily for 10 days
E. Reassurance
241.
A 20-year-old male presents with a complaint of pain in
his right testis. The onset of pain has been gradual and
has been associated with dysuria and urinary frequency.
The patient has no medical problems and is sexually
active. On examination he has some swelling and mild
tenderness of the testis. The area posterior to the testis
is swollen and very tender. He has a normal cremasteric
reflex, and the pain improves with elevation of the
testicle.
Which one of the following would be the most
appropriate management of this patient?
(check one)
A. Surgical evaluation
B. Doppler ultrasonography
C. Ceftriaxone (Rocephin) and doxycycline
D. Levofloxacin (Levaquin)
E. Ciprofloxacin (Cipro)
C. Ceftriaxone (Rocephin) and doxycycline. This patient has
epididymitis. In males 14-35 years of age, the most common causes are
Neisseria gonorrhoeae and Chlamydia trachomatis. The recommended
treatment in this age group is ceftriaxone, 250 mg intramuscularly, and
doxycycline, 100 mg twice daily for 10 days (SOR C). A single 1-g dose of
azithromycin may be substituted for doxycycline. In those under age 14
or over age 35, the infection is usually caused by one of the common
urinary tract pathogens, and levofloxacin, 500 mg once daily for 10
days, would be the appropriate treatment (SOR C).
If there is concern about testicular torsion, urgent surgical evaluation
and ultrasonography are appropriate. Testicular torsion is most
common between 12 and 18 years of age but can occur at any age. It
usually presents with an acute onset of severe pain and typically does
not have associated urinary symptoms. On examination there may be a
high-riding transversely oriented testis with an abnormal cremasteric
reflex and pain with testicular evaluation. Color Doppler
ultrasonography will show a normal-appearing testis with decreased
blood flow.
242.
A 20-year-old patient comes to the
emergency department complaining of
shortness of breath. On examination his
heart rate is 180 beats/min, and his blood
pressure is 122/68 mm Hg. An EKG reveals a
narrow complex tachycardia with a regular
rhythm.
Which one of the following would be the
most appropriate initial treatment? (check
one)
C. Adenosine (Adenocard). After vagal maneuvers are attempted in a stable patient
with supraventricular tachycardia, the patient should be given a 6-mg dose of
adenosine by rapid intravenous push. If conversion does not occur, a 12-mg dose
should be given. This dose may be repeated once. If the patient is unstable,
immediate synchronized cardioversion should be administered.
A. Amiodarone (Cordarone)
B. Diltiazem (Cardizem)
C. Adenosine (Adenocard)
D. Magnesium
E. Synchronized cardioversion
243.
A 20-year-old single white female who is a
patient of yours was raped in her apartment
at 7:00 a.m. today. She is brought to your
office at 9:00 a.m. for assessment and
treatment. Despite having occasional
intercourse with her boyfriend, she has
never used any type of contraceptive. They
last had intercourse approximately 1 week
ago, and the boyfriend has been out of town
on business since then. The patient has a
history of irregular periods, and her last
normal period was approximately 2 and a
half weeks ago. You note live sperm on a wet
mount. In addition to many other issues
that must be addressed at this visit, the
patient asks about emergency
contraception. Which one of the following
would be accurate advice to the patient
regarding this topic? (check one)
A. Emergency contraception does not
interfere with an established, postimplantation pregnancy
B. The estrogen/progestin combination
regimen appears to be more effective than
the levonorgestrel-only regimen
C. To be most effective, each dose of the 2dose regimen should be administered at
least 72 hours apart
D. Fetal malformations have been reported
as a result of the unsuccessful use of the
high-dose emergency contraceptive regimen
A. Emergency contraception does not interfere with an established, postimplantation pregnancy. An FDA Advisory Committee has recommended over-thecounter marketing of Plan B, an emergency contraceptive package that contains two
0.75-mg tablets of levonorgestrel to be taken 12 hours apart. Plan B is one of the two
FDA-approved products for this indication. The Preven emergency contraceptive kit
includes four tablets, each containing 0.25 mg of levonorgestrel and 50 g of ethinyl
estradiol; these are taken two at a time 12 hours apart. In a randomized, controlled
trial comparing the single versus combined estrogen/progestin, the single-drug
regimen was shown to be more effective. Pregnancy occurred in 11 of 976 women
(1.1%) given levonorgestrel alone, and in 31 of 979 (3.2%) given ethinyl estradiol plus
levonorgestrel. The proportion of pregnancies prevented, compared to the
expected number without treatment, was 85% with levonorgestrel and 57% with the
combination. In both regimens, the interval between individual doses is 12 hours. In
this case, emergency contraception may be appropriate in the face of a possible
pregnancy from previous consensual intercourse. Emergency contraception has not
been found to interfere with an established post-implantation pregnancy.
Furthermore, no fetal malformations have been reported as a result of the
unsuccessful use of high-dose oral contraceptives for emergency contraception.
244.
A 20-year-old white female presents with painful and frequent
urination that has had a gradual onset over the past week. She
has never had a urinary tract infection. There is no associated
hematuria, flank pain, suprapubic pain, or fever. She says she
has not noted any itching or vaginal discharge. A midstream
urine specimen taken earlier in the week showed significant
pyuria but a culture was reported as no growth. She has taken
an antibiotic for 2 days without relief. Her only other
medication is an oral contraceptive agent. Which one of the
following is the most likely infectious agent? (check one)
A. Escherichia coli
B. Chlamydia trachomatis
C. Candida albicans
D. Staphylococcus saprophyticus
B. Chlamydia trachomatis. Women who present with symptoms of
acute dysuria, frequency, and pyuria do not always have
bacterial cystitis. In fact, up to 30% will show either no growth or
insignificant bacterial growth on a midstream urine culture. Most
commonly these patients represent cases of sexually transmitted
urethritis caused by Chlamydia trachomatis, Neisseria
gonorrhoeae, or herpes simplex virus.
In this case, the gradual onset, absence of hematuria, and weeklong duration of symptoms suggest a sexually transmitted
disease. A history of a new sexual partner or a finding of
mucopurulent cervicitis would confirm the diagnosis. Empiric
treatment with a tetracycline and a search for other sexually
transmitted diseases would then be indicated.
Another possible diagnosis is urinary tract infection with
Escherichia coli or Staphylococcus species; however, the onset of
these infections is usually abrupt and accompanied by other
signs, such as suprapubic pain or hematuria. Candida is unlikely
because there is no accompanying discharge or itching, and the
patient's symptoms predate the use of antibiotics.
245.
A 20-year-old white male presents to your office after a fall on
an outstretched hand while skateboarding. He has pain at the
anatomic snuffbox with no abrasion. Radiographs are
negative. Which one of the following would be the most
appropriate management? (check one)
A. A long arm cast for 8 weeks
B. A thumb spica splint and follow-up radiographs in 2 weeks
C. A sugar tong splint and follow-up radiographs in 2 weeks
D. An Ace bandage and follow-up radiographs in 2 weeks
E. An Ace bandage and follow-up in 2 weeks if the patient is
still experiencing pain
246.
A 21-year-old female complains of bulging veins in her right
shoulder region, along with swelling and a "tingling"
sensation in her right arm that has developed over the past 2
days. There were no unusual events other than her regular
workouts with her swim team. Ultrasonography confirms an
upper extremity deep-vein thrombosis of her right axillary
vein.
Which one of the following would be the most appropriate
treatment? (check one)
A. Intravenous heparin for 72 hours, followed by oral warfarin
(Coumadin) for 3 months
B. Low molecular weight heparin (LMWH) subcutaneously for
5 days only
C. LMWH subcutaneously for at least 5 days, followed by oral
warfarin for 3 months
D. LMWH subcutaneously for at least 5 days, followed by oral
warfarin indefinitely
E. Oral warfarin for 3 months
B. A thumb spica splint and follow-up radiographs in 2 weeks.
This is a classic presentation of a possible scaphoid fracture. This
fracture is important to diagnose and treat appropriately
because of a high rate of non-union. If radiographs are negative,
the patient should be placed in a thumb spica splint and have
repeat radiographs in 2 weeks, because initial studies may be
negative. An Ace bandage or a sugar tong splint would be
inappropriate because they do not immobilize the thumb. A long
arm cast for 8 weeks would immobilize the thumb, but could lead
to loss of function, and may overtreat the injury if it is not truly a
scaphoid fracture.
C. LMWH subcutaneously for at least 5 days, followed by oral
warfarin for 3 months. Upper extremity deep-vein thrombosis
(UE-DVT) accounts for 4% of all cases of DVT. Catheter-related
thromboses make up the majority of these cases. Occult cancer,
use of oral contraceptives, and inheritable thrombophilia are
other common explanations. Another proposed risk factor is the
repetitive compression of the axillary-subclavian vein in athletes
or laborers, which is the most likely cause of this patient's UE-DVT.
Taken as a whole, UE-DVT is generally associated with fewer
venous complications, including less chance for
thromboembolism, postphlebitic syndrome, and recurrence
compared to lower-extremity deep-vein thrombosis (LE-DVT).
However, the rates of these complications are still high enough
that most experts recommend treatment identical to that of LEDVT. Specifically, heparin should be given for 5 days, and an oral
vitamin-K antagonist for at least 3 months.
247.
A 21-year-old married
Hispanic female who is
using no method of
contraception presents to
your office for evaluation
of vaginal spotting 6
weeks after her last
menstrual period. Her
periods have previously
been regular. She has had
one previous episode of
pelvic inflammatory
disease. A home
pregnancy test is positive.
Which one of the
following is true in this
situation? (check one)
A. Serum hCG levels
should double every 2-3
days if the pregnancy is
viable
B. Painless bleeding
excludes the diagnosis of
ectopic pregnancy
C. Laparoscopy should be
performed to exclude
ectopic pregnancy
D. A serum progesterone
level >25 ng/mL indicates
that ectopic pregnancy is
likely
A. Serum hCG levels should double every 2-3 days if the pregnancy is viable. Early diagnosis of ectopic
pregnancy requires a high index of suspicion. Risk factors include previous ectopic pregnancy, tubal
sterilization, pelvic inflammatory disease, IUD use, and in utero exposure to diethylstilbestrol. The classic
triad of missed menses, pain, and bleeding may not always be present. In early pregnancies of less than
5 weeks' gestation, serial hCG levels are helpful. Serum hCG levels double every 1.4-2 days. In a healthy
pregnancy the level is expected to increase by at least 66% in 48 hours. Combining serial hCG levels with
transvaginal ultrasonography is the best combination for evaluation of first-trimester problems. Serum
hCG levels correlate well with sonographic landmarks. At 5 weeks' gestation in a normal pregnancy,
serum hCG is >1000 mIU/mL and a gestational sac can be visualized in the uterus. Serum hCG is >2500
mIU/mL at 6 weeks and a yolk sac can be seen within the gestational sac. An hCG level of 5000 mIU/mL is
compatible with visualization of a fetal pole. When the level is 17,000 mIU/mL, cardiac activity can be
detected. Progesterone levels are also predictive of fetal outcome. A single level of 25 ng/mL or higher
indicates a healthy pregnancy and excludes ectopic pregnancy with a sensitivity of 98%. If the level is <5
ng/mL, the pregnancy is nonviable. Assessment of fetal well-being is difficult if levels are in the
intermediate range of 5-25 ng/mL.
248.
A 21-year-old primigravida at 28 weeks gestation
complains of the recent onset of itching. On
examination she has no obvious rash. The pruritus
started on her palms and soles and spread to the
rest of her body. Laboratory evaluation reveals
elevated serum bile acids and mildly elevated
bilirubin and liver enzymes.
The most effective treatment for this condition is:
(check one)
E. ursodiol (Actigall). This patient's symptoms and laboratory values are most
consistent with intrahepatic cholestasis of pregnancy. Ursodiol has been
shown to be highly effective in controlling the pruritus and decreased liver
function (SOR A) and is safe for mother and fetus. Topical antipruritics and
oral antihistamines are not very effective. Cholestyramine may be effective in
mild or moderate intrahepatic cholestasis, but is less effective and safe than
ursodiol.
A. triamcinolone (Kenalog) cream
B. cholestyramine (Questran)
C. diphenhydramine (Benadryl)
D. doxylamine succinate
E. ursodiol (Actigall)
249.
A 21-year-old sexually active female presents with
acute pelvic pain of several days' duration. A pelvic
examination reveals right-sided tenderness and a
general fullness in that area. In addition to
laboratory testing, you decide to order an imaging
study.
Which one of the following is the best choice at
this time? (check one)
A. Transabdominal ultrasonography
B. Transvaginal ultrasonography
C. Contrast CT of the abdomen and pelvis
D. Hysteroscopy
E. Hysterosalpingography
B. Transvaginal ultrasonography. The best initial imaging study for acute
pelvic pain in women is transvaginal ultrasonography (SOR C). This provides
the greatest level of detail regarding the uterus and adnexae, superior to
transcutaneous ultrasonography. CT of the abdomen/pelvis and
hysterosalpingography may be indicated eventually in some patients with
pelvic pain, but they are not the initial studies of choice. Hysteroscopy is not
routinely used in the evaluation of pelvic pain.
250.
A 22-year-old competitive cross-country
skier presents with a complaint of not being
able to perform as well as she expects. She
has been training hard, but says she seems
to get short of breath more quickly than she
should. She also coughs frequently while
exercising. A review of systems is otherwise
negative. Her family history is negative for
cardiac or pulmonary diseases. Her physical
examination is completely normal, and
pulmonary function tests obtained before
and after bronchodilator use are normal.
After you discuss your findings with the
patient, she acknowledges that her
expectations may be too high, but can think
of no other cause for her problem. Which
one of the following would be the next
reasonable step? (check one)
D. A trial of inhaled albuterol (Proventil) for exercise-induced bronchospasm.
Exercise-induced bronchoconstriction (EIB) is a very common and underdiagnosed
condition in athletes. It is defined as a 10% lowering of FEV1 when challenged with
exercise. The exercise required to cause bronchoconstriction is 5-8 minutes at 80% of
maximal oxygen consumption. EIB is much more common in high-ventilation sports,
such as track and cross-country skiing. It is also more common in winter sports,
because of the inspiration of cold, dry air. In some studies the incidence among crosscountry skiers is as high as 50%, and 40% of those who have positive tests for
bronchospasm are unaware of the problem. A physical examination, as well as
pulmonary function tests at rest and before and after bronchodilators, will be
normal unless there is underlying asthma. Among athletes with EIB, 10% will not
have asthma. Bronchoprovocative testing can be ordered, but if it is not available a
trial with an albuterol inhaler is reasonable. Cardiomyopathy or valvular dysfunction
not found during the physical examination is possible, but much less likely.
Psychological stresses are also a possible etiology, but should not receive undue
attention, especially when simple questioning is not productive and more likely
diagnoses have not been ruled out. Poor training methods are also possible, but in a
competitive athlete this is not the most likely cause.
A. An echocardiogram to look for
cardiomyopathy or valvular dysfunction
B. Counseling regarding competition stress
and athlete burnout syndrome
C. A sports medicine consultation to
evaluate her training regimen
D. A trial of inhaled albuterol (Proventil) for
exercise-induced bronchospasm
251.
A 22-year-old female presents with lower
right leg pain. She reports that it hurts when
she presses her shin. She has been training
for a marathon over the past 4 months and
has increased her running frequency and
distance. She now runs almost every day
and is averaging approximately 40 miles per
week. She has little pain while at rest, but
the pain intensifies with weight bearing and
ambulation. She initially thought the pain
was from shin splints, but it has intensified
this week and she has had to shorten her
usual running distances due to worsening
pain.
On examination you note tenderness to
palpation over the anterior aspect of her
mid-tibia. She also has trace edema
localized to the area of tenderness.
Which one of the following imaging studies
should be performed first? (check one)
A. Plain radiographs
B. CT
C. MRI
D. Ultrasonography
E. Bone scintigraphy
A. Plain radiographs. The findings in this patient are consistent with a stress fracture.
Plain radiographs should be the initial imaging modality because of availability and
low cost (SOR C). These are usually negative initially, but are more likely to be positive
over time. If the initial films are negative and the diagnosis is not urgently needed, a
second plain radiograph can be performed in 2-3 weeks.
Although CT is useful for evaluation of bone pathology, it is not commonly used as
even second-line imaging for stress fractures, due to lower sensitivity and higher
radiation exposure than other modalities. Triple-phase bone scintigraphy has a high
sensitivity and was previously used as a second-line modality; however, MRI has
equal or better sensitivity than scintigraphy and higher specificity. MRI is now
recommended as the second-line imaging modality when plain radiographs are
negative and clinical suspicion of stress fracture persists (SOR C). Musculoskeletal
ultrasonography has the advantage of low cost with no radiation exposure, but
additional studies are needed before it can be recommended as a standard imaging
modality.
252.
A 22-year-old male has acute low back
pain without paresthesias or other
neurologic signs. There is no lower
extremity weakness.
Which treatment has been shown to
be of most benefit initially? (check
one)
D. Resumption of physical activity as tolerated. For patients who have acute back pain
without sciatic involvement, a return to normal activities as tolerated has been shown to be
more beneficial than either bed rest or a basic exercise program. Bed rest for more than 2
or 3 days in patients with acute low back pain is ineffective and may be harmful. Patients
should be instructed to remain active. Injections should be considered only if conservative
therapy fails.
A. Complete bed rest for 2 weeks
B. Bed rest plus local injection of
corticosteroids
C. A low-back strengthening program
D. Resumption of physical activity as
tolerated
253.
A 22-year-old male presents to your
office with a 2-hour history of a
painful right scrotal mass. The
physical examination raises concerns
that the patient may have testicular
torsion. The imaging study of choice
would be (check one)
A. a plain film
B. color duplex Doppler
ultrasonography
C. CT
D. MRI
E. a nuclear scan
B. color duplex Doppler ultrasonography. The history and physical examination are critical
for making a diagnosis in patients with scrotal pain. Transillumination may also be
performed as part of the clinical assessment. If the diagnosis is uncertain, ultrasonography
with color Doppler imaging has become the accepted standard for evaluation of the acutely
swollen scrotum (SOR B). Ultrasonography alone can confirm the diagnosis in a number of
conditions, such as hydrocele, spermatocele, and varicocele. For other conditions such as
orchitis, carcinoma, or torsion, color Doppler ultrasonography is essential because it will
show increased flow in orchitis, normal or increased flow in carcinoma, and decreased blood
flow in testicular torsion.
For testicular torsion, color Doppler ultrasonography has a sensitivity of 86%-88% and a
specificity of 90%-100%. When testicular torsion is strongly suspected, emergent surgical
consultation should be obtained before ultrasonography is performed, because surgical
exploration as soon as possible is critical to salvaging the testis and should not be delayed
for imaging unless the diagnosis is in doubt.
While radionuclide imaging would be accurate for diagnosing testicular torsion, it is not used
for this purpose because of time limits and lack of easy availability. CT or MRI may be
appropriate if ultrasonography indicates a possibility of carcinoma. Plain films are not useful
in assessing scrotal swelling or masses.
254.
A 22-year-old male with no previous
history of shoulder problems is
injured in a fall. He has immediate
pain and is unable to abduct his arm.
You examine him and order an MRI,
which reveals an acute tear of the
rotator cuff. Which one of the
following is the best initial treatment
for this injury? (check one)
A. Observation without treatment for
1 month
B. Immobilization for 1 month
C. Physical therapy for 1 month
D. Corticosteroid injection
E. Surgical repair
E. Surgical repair. An acute rupture of any major tendon should be repaired as soon as
possible. Acute tears of the rotator cuff should be repaired within 6 weeks of the injury if
possible (SOR C). Nonsurgical management is not recommended for active persons.
Observing for an extended period will likely lead to retraction of the detached tendon,
possible resorption of tissue, and muscle atrophy.
255.
A 22-year-old white female comes to your office
complaining of dizziness. She was in her usual good health
until about 2 weeks before this visit, when she developed
a case of gastroenteritis that other members of her family
have also had. Since that time she has been lightheaded
when standing, feels her heart race, and gets headaches or
blurred vision if she does not sit or lie down. She has not
passed out but has been unable to work due to these
symptoms. She is otherwise healthy and takes no regular
medications.
C. Postural orthostatic tachycardia syndrome (POTS). Postural
orthostatic tachycardia syndrome (POTS) is manifested by a rise in
heart rate >30 beats/min or by a heart rate >120 beats/min within 10
minutes of being in the upright position. Symptoms usually include
position-dependent headaches, abdominal pain, lightheadedness,
palpitations, sweating, and nausea. Most patients will not actually pass
out, but some will if they are unable to lie down quickly enough. This
condition is most prevalent in white females between the ages of 15
and 50 years old. Often these patients are hardworking, athletic, and
otherwise in good health.
A physical examination is normal except for her heart
rate, which rises from 72 beats/min when she is lying or
sitting to 112 beats/min when she stands. Her blood
pressure remains unchanged with changes of position.
Routine laboratory tests and an EKG are normal.
There is a high clinical correlation between POTS and chronic fatigue
syndrome. Although no single etiology for POTS has been found, the
condition is thought to have a genetic predisposition, is often incited
after a prolonged viral illness, and has a component of deconditioning.
The recommended initial management is encouraging adequate fluid
and salt intake, followed by the initiation of regular aerobic exercise
combined with lower-extremity strength training, and then the use of
-blockers.
What is the most likely cause of this patient's condition?
(check one)
A. Myocarditis
B. A seizure disorder
C. Postural orthostatic tachycardia syndrome (POTS)
D. Systemic lupus erythematosus
E. Somatization disorder
256.
A 23-month-old child is brought to your office with a 2day history of a fever to 102F (39C), cough, wheezing,
and mildly labored breathing. He has no prior history of
similar episodes and there is no improvement with
administration of an aerosolized bronchodilator.
Which one of the following is now indicated? (check one)
A. Bronchodilator aerosol treatment every 6 hours
B. Corticosteroids
C. An antibiotic
D. A decongestant
E. Supportive care only
E. Supportive care only. This child has typical findings of bronchiolitis.
The initial infection usually occurs by the age of 2 years. It is caused by
respiratory syncytial virus (RSV). Bronchodilator treatment may be
tried once and discontinued if there is no improvement. Treatment
usually consists of supportive care only, including oxygen and
intravenous fluids if indicated (SOR B). Corticosteroids, antibiotics, and
decongestants are of no benefit. RSV infection may recur, since an
infection does not provide immunity. Up to 10% of infected children
will have wheezing past age 5, and bronchiolitis may predispose them
to asthma.
257.
A 23-year-old female comes to your office 6 days after
giving birth to her first child by cesarean section. Her
pregnancy was complicated by preeclampsia. During
the history she reports brief crying spells, irritability,
poor sleep, and nervousness. Her husband notes that
"even the littlest thing can set her off." She has a
history of major depression 2 years ago that resolved
with psychotherapy and SSRI treatment. She and her
husband are concerned that she may be suffering
from postpartum depression.
D. A previous history of depression. "Baby blues" are differentiated from
postpartum depression by the severity and duration of symptoms. Baby
blues occur in 80% of postpartum women and are associated with mild
dysfunction. They begin during the first 2-3 days after delivery and resolve
within 10 days. Symptoms include brief crying spells, irritability, poor sleep,
nervousness, and emotional reactivity. An estimated 5%-7% of women
develop a postpartum major depression associated with moderate to
severe dysfunction during the first 3 months post partum. While women
with baby blues are at risk for progression to major depression, no more
than 8%-10% will progress to a major postpartum depression.
Which one of the following is the greatest risk factor
for postpartum depression in this patient? (check
one)
A previous history of major depressive disorder significantly increases the
risk of developing postpartum depression (RR = 4.5), and a prior episode of
postpartum depression is the strongest risk factor for postpartum
depression in subsequent pregnancies. Prenatal and obstetric
complications and socioeconomic status have not consistently been shown
to be risk factors. First pregnancy is also not a significant risk factor.
A. Operative delivery
B. First pregnancy and delivery
C. Preeclampsia
D. A previous history of depression
258.
A 23-year-old female sees you with a complaint of
intermittent irregular heartbeats that occur once
every week or two, but do not cause her to feel
lightheaded or fatigued. They last only a few seconds
and resolve spontaneously. She has never passed out,
had chest pain, or had difficulty with exertion. She is
otherwise healthy, and a physical examination is
normal. Which one of the following cardiac studies
should be ordered initially? (check one)
D. An EKG. The symptom of an increased or abnormal sensation of one's
heartbeat is referred to as palpitations. This condition is common to
primary care, but is often benign. Commonly, these sensations have their
basis in anxiety or panic. However, about 50% of those who complain of
palpitations will be found to have a diagnosable cardiac condition. It is
recommended to start the evaluation for cardiac causes with an EKG, which
will assess the baseline rhythm and screen for signs of chamber
enlargement, previous myocardial infarction, conduction disturbances,
and a prolonged QT interval.
A. 24-hour ambulatory EKG monitoring (Holter
monitor)
B. 30-day continuous closed-loop event recording
C. Echocardiography
D. An EKG
E. Electrophysiologic studies
259.
A 23-year-old female with a history of systemic lupus
erythematosus presents with a 48-hour history of
vague left precordial pain. Serum markers for acute
cardiac injury are normal. An EKG performed in the
emergency department is shown in Figure 7.
...
260.
A 23-year-old gravida 1 para 0 at 36
weeks gestation presents to the
office complaining of ankle
swelling and headache for the
past 2 days. She denies any
abdominal pain or visual
disturbances. On examination
you note a fundal height of 35 cm,
a fetal heart rate of 140 beats/min,
2+ lower extremity edema, and a
blood pressure of 144/92 mm Hg.
A urine dipstick shows 1+
proteinuria. A cervical
examination reveals 2 cm
dilation, 90% effacement, -1
station, and vertex presentation.
Which one of the following is the
most appropriate next step in the
management of this patient?
(check one)
A. Laboratory evaluation, fetal testing, and 24-hour urine for total protein. This patient most
likely has preeclampsia, which is defined as an elevated blood pressure and proteinuria after 20
weeks gestation. The patient needs further evaluation, including a 24-hour urine for
quantitative measurement of protein, blood pressure monitoring, and laboratory evaluation
that includes hemoglobin, hematocrit, a platelet count, and serum levels of transaminase,
creatinine, albumin, LDH, and uric acid. A peripheral smear and coagulation profiles also may
be obtained. Antepartum fetal testing, such as a nonstress test to assess fetal well-being, would
also be appropriate.
Ultrasonography should be done to assess for fetal intrauterine growth restriction, but only
after an initial laboratory and fetal evaluation. Delivery is the definitive treatment for
preeclampsia. The timing of delivery is determined by the gestational age of the fetus and the
severity of preeclampsia in the mother. Vaginal delivery is preferred over cesarean delivery, if
possible, in patients with preeclampsia. It is not necessary to start this patient on
antihypertensive therapy at this point. An obstetric consultation should be considered for
patients with preeclampsia.
A. Laboratory evaluation, fetal
testing, and 24-hour urine for
total protein
B. Ultrasonography to check for
fetal intrauterine growth
restriction
C. Initiation of antihypertensive
treatment
D. Immediate induction of labor
E. Immediate cesarean delivery
261.
A 23-year-old gravida 3 para 1 at 28
weeks' gestation whose blood
type is O-negative is antibody
positive (D antibody) on a routine
28-week screen. Which one of the
following best describes the
clinical significance of this
finding? (check one)
A. The fetus HAS hemolytic
disease and requires appropriate
monitoring and treatment
B. The fetus is AT RISK for
hemolytic disease only if the
biological father is Rh-negative
C. The fetus is AT RISK for
hemolytic disease only if the
biological father is Rh-positive
D. The current fetus is NOT at risk
for hemolytic disease, but
subsequent pregnancies may be
at risk
C. The fetus is AT RISK for hemolytic disease only if the biological father is Rh-positive. When a
person is Rh negative, this indicates that they do not have type D antigen on their red blood
cells. If a woman is exposed to Rh D antigen-positive red blood cells, she can have an immune
response of variable strength. This may occur in the setting of pregnancy (transplacental
fetomaternal transfusion), or exposure outside of pregnancy (e.g., transfusion with mismatched
blood). If a maternal antibody screen for D antigen is positive, this indicates that the current
fetus MAY be at risk for hemolytic disease. The level of risk is determined by the antibody titer.
For example, an antibody titer of 1:4 poses much less risk to the fetus than a titer of 1:64.
Determination of the blood type of the father is helpful if paternity is certain. If the father is
homozygous Rh negative, there is no risk of alloimmunization to the fetus and the fetus is NOT at
risk for hemolytic disease. In this scenario, maternal sensitization occurred either from a prior
pregnancy with a different partner or from another source (e.g., transfusion). If the father is
heterozygous or homozygous Rh positive, then the fetus IS at risk. If paternity is uncertain, a
polymerase chain reaction can be performed on 2 mL of amniotic fluid or 5 mL of chorionic villi to
accurately determine the fetal Rh status.
262.
A 23-year-old Hispanic female at 18 weeks' gestation
presents with a 4-week history of a new facial rash.
She has noticed worsening with sun exposure. Her
past medical history and review of systems is
normal. On examination, you note symmetric,
hyperpigmented patches on her cheeks and upper
lip. The remainder of her examination is normal.
The most likely diagnosis is: (check one)
A. Lupus erythematosus
B. Pemphigoid gestationis (herpes gestationis)
C. Melasma (chloasma)
D. Prurigo gestationis
263.
A 23-year-old male returns from a Florida beach
vacation, where he sustained a cut to his foot while
wading. The cut wasn't treated when it happened,
and it is healing, but he says that it feels like
something in the wound is "poking" him. Of the
following, which one would most likely be easily
visible on plain film radiography? (check one)
C. Melasma (chloasma). Melasma or chloasma is common in pregnancy, with
approximately 70% of pregnant women affected. It is an acquired
hypermelanosis of the face, with symmetric distribution usually on the
cheeks, nose, eyebrows, chin, and/or upper lip. The pathogenesis is not
known. UV sunscreen is important, as sun exposure worsens the condition.
Melasma often resolves or improves post partum. Persistent melasma can be
treated with hydroquinone cream, retinoic acid, and/or chemical peels
performed post partum by a dermatologist. The facial rash of lupus is usually
more erythematous, and lupus is relatively rare. Pemphigoid gestationis is a
rare autoimmune disease with extremely pruritic, bullous skin lesions that
usually spare the face. Prurigo gestationis involves pruritic papules on the
extensor surfaces and is usually associated with significant excoriation by the
uncomfortable patient.
B. A glass splinter. Almost all glass is visible on radiographs if it is 2 mm or
larger, and contrary to popular belief, it doesn't have to contain lead to be
visible on plain films. Many common or highly reactive materials, such as
wood, thorns, cactus spines, some fish bones, other organic matter, and most
plastics, are not visible on plain films. Alternative techniques such as
ultrasonography or CT scanning may be effective and necessary in those
cases. Sea urchin spines, like many animal parts, have not been found to be
easily detected by plain radiography.
A. A wood splinter
B. A glass splinter
C. A plastic splinter
D. A sea urchin spine
264.
A 23-year-old white male is brought to the
emergency department with slurred speech,
confusion, and ataxia. He works as an auto
mechanic and has been known to consume alcohol
heavily in the past, but denies recent alcohol
intake. He appears intoxicated, but no odor of
alcohol is noted on his breath. Abnormalities on
the metabolic profile include a carbon dioxide
content of 10 mmol/L (N 20-30). His blood alcohol
level is <10 mg/dL (0.01%). A urinalysis shows
calcium oxalate crystals and an RBC count of 1020/hpf. Woods lamp examination of the urine
shows fluorescence. His arterial pH is 7.25.
Which one of the following would be most
appropriate at this point? (check one)
A. Immediate hemodialysis
B. Gastric lavage
C. Administration of activated charcoal
D. Fomepizole (Antizol)
D. Fomepizole (Antizol). Ethylene glycol poisoning should be suspected in
patients with metabolic acidosis of unknown cause and subsequent renal
failure, as rapid diagnosis and treatment will limit the toxicity and decrease
both morbidity and mortality. This diagnosis should be considered in a
patient who appears intoxicated but does not have an odor of alcohol, and
has anion gap acidosis, hypocalcemia, urinary crystals, and nontoxic blood
alcohol levels. Ethylene glycol is found in products such as engine coolant, deicing solution, and carpet and fabric cleaners. Ingestion of 100 mL of ethylene
glycol by an adult can result in toxicity.
The American Academy of Clinical Toxicology criteria for treatment of
ethylene glycol poisoning with an antidote include a plasma ethylene glycol
concentration >20 mg/dL, a history of ingesting toxic amounts of ethylene
glycol in the past few hours with an osmolal gap >10 mOsm/kg H O2 (N 5-10),
and strong clinical suspicion of ethylene glycol poisoning, plus at least two of
the following: arterial pH <7.3, serum bicarbonate <20 mmol/L, or urinary
oxalate crystals.
Until recently, ethylene glycol poisoning was treated with sodium
bicarbonate, ethanol, and hemodialysis. Treatment with fomepizole (Antizol)
has this specific indication, however, and should be initiated immediately
when ethylene glycol poisoning is suspected. If ethylene glycol poisoning is
treated early, hemodialysis may be avoided, but once severe acidosis and
renal failure have occurred hemodialysis is necessary. Ethylene glycol is
rapidly absorbed, and use of ipecac or gastric lavage is therefore not
effective. Large amounts of activated charcoal will only bind to relatively small
amounts of ethylene glycol, and the therapeutic window for accomplishing
this is less than 1 hour.
265.
A 24-year-old female at 36 weeks' gestation plans to
breastfeed her infant. She has a history of bipolar
disorder, but currently is doing well without
medication, and also has a history of frequent urinary
tract infections. She asks you about medications that
she may need to take after delivery, and how they may
affect her newborn. Which one of the following would
be contraindicated if she breastfeeds her infant? (check
one)
D. Lithium. Of the drugs listed, the only maternal medication that affects
the infant is lithium. Breastfed infants of women taking lithium can have
blood lithium concentrations that are 30%-50% of therapeutic levels.
A. Amoxicillin
B. Macrodantin (Macrobid)
C. Valproic acid (Depakote)
D. Lithium
266.
A 24-year-old female had been healthy with no
significant medical illnesses until about 3 months ago,
when she was diagnosed with schizophrenia and
treatment was initiated. She is now concerned because
she has gained 10 lb since beginning treatment. A
comprehensive metabolic panel is normal, with the
exception of a fasting blood glucose level of 156 mg/dL.
Which one of the following medications would be most
likely to cause these findings?
(check one)
E. Olanzapine (Zyprexa). Second-generation, or "atypical,"
antipsychotics are associated with weight gain, elevated triglycerides,
and type 2 diabetes mellitus. Olanzapine and clozapine are associated
with the highest risk. Clonazepam, a benzodiazepine, does not share
these risks. Thioridazine and chlorpromazine are first-generation
antipsychotics, and carry less risk of these side effects. Aripiprazole,
although it is a second-generation antipsychotic, has been found to
cause weight gain and metabolic changes similar to those seen with
placebo.
A. Clonazepam (Klonopin)
B. Thioridazine
C. Chlorpromazine
D. Aripiprazole (Abilify)
E. Olanzapine (Zyprexa)
267.
A 24-year-old female has a history of mood swings over
the past several months, which have created marital and
financial problems, in addition to jeopardizing her
career as a television news reporter. You have made a
diagnosis of bipolar disorder, and she has finally
accepted the need for treatment. However, she insists
that you choose a drug that "won't make me fat."
Which one of the following would be best for addressing
her concerns? (check one)
A. Aripiprazole (Abilify)
B. Olanzapine (Zyprexa)
C. Quetiapine (Seroquel)
D. Risperidone (Risperdal)
A. Aripiprazole (Abilify). All of the atypical antipsychotics are associated
with some degree of weight gain. Of the choices listed, aripiprazole is
associated with the least amount of weight gain, generally less than 1
kilogram. The other agents listed are likely to cause considerably more
weight gain.
268.
A 24-year-old female has noted excessive
hair loss over the past 2 months, with a
marked increase in hairs removed when
she brushes her hair. She delivered a
healthy baby 5 months ago. She is on no
medications, and is otherwise healthy.
Examination of her scalp reveals diffuse
hair thinning without scarring. An
evaluation for thyroid dysfunction and
iron deficiency is negative.
Which one of the following is the most
likely cause of her hair loss?
(check one)
A. Telogen effluvium
B. Anagen effluvium
C. Alopecia areata
D. Female-pattern hair loss
E. Discoid lupus erythematosus
269.
A 24-year-old female presents for her
annual examination. She is single and has
had several male sexual partners during
the past year. You include screening for
chlamydial infection in your evaluation,
and the test is reported as positive. She is
asymptomatic. Which one of the
following is true concerning this
situation? (check one)
A. Telogen effluvium. The recycling of scalp hair is an ongoing process, with the hair
follicles rotating through three phases. The actively growing anagen-phase hairs give
way to the catagen phase, during which the follicle shuts down, followed by the resting
telogen phase, during which the hair is shed. The normal ratio of anagen to telogen
hairs is 90:10.
This patient most likely has a telogen effluvium, a nonscarring, shedding hair loss that
occurs when a stressful event, such as a severe illness, surgery, or pregnancy, triggers
the shift of large numbers of anagen-phase hairs to the telogen phase. Telogen-phase
hairs are easily shed. Telogen effluvium occurs about 3 months after a triggering event.
The hair loss with telogen effluvium lasts 6 months after the removal of the stressful
trigger.
Anagen effluvium is the diffuse hair loss that occurs when chemotherapeutic
medications cause rapid destruction of anagen-phase hair. Alopecia areata, which
causes round patches of hair loss, is felt to have an autoimmune etiology. Femalepattern hair loss affects the central portion of the scalp, and is not associated with an
inciting trigger or shedding. Discoid lupus erythematosus causes a scarring alopecia.
A. Failure to treat this patient would place her at higher risk of later infertility. It is
recommended that sexually active women under the age of 25 years be screened
routinely for Chlamydia trachomatis. Treatment of asymptomatic infections in women
reduces their risk of developing pelvic inflammatory disease, tubal infertility, ectopic
pregnancy, and chronic pelvic pain. A 1-gram dose of oral azithromycin is an
appropriate treatment, including during pregnancy. Sexual contacts during the
preceding 60 days should be either treated empirically or tested for infection and
treated if positive. The patient should avoid sexual intercourse for 7 days after initiation
of treatment. Consistent use of barrier methods for contraception reduces the risk of C.
trachomatis genital infection.
A. Failure to treat this patient would place
her at higher risk of later infertility
B. Only sexual partners with whom she
has been active during the last 2 weeks
need to be treated
C. She should avoid sexual intercourse for
1 month after treatment
D. Use of barrier methods of
contraception increases her risk for
repeat infection
270.
A 24-year-old female presents to your
clinic with a 5-day history of fever to
103F. She has no localizing symptoms or
overt physical findings. Initial testing
shows an elevated WBC count with a
disproportionate number of reactive
lymphocytes.
Which one of the following conditions is
the most likely cause of these findings?
(check one)
A. Bacterial infection
B. Connective tissue disease
C. Lymphoma
D. Viral infection
D. Viral infection. The conditions that result in an absolute increase in lymphocytes are
divided into primary causes (usually neoplastic hyperproliferation) and secondary or
reactive causes. The presence of reactive lymphocytes will often be reported on a
manual differential, since they have a distinctive appearance. The most common
conditions that produce a reactive lymphocytosis are viral infections. Most notable are
Epstein-Barr virus, infectious mononucleosis, and cytomegalovirus. Other viral
infections known to cause this finding include herpes simplex, herpes zoster, HIV,
hepatitis, and adenovirus.
Connective tissue disease can infrequently cause a reactive lymphocytosis, but other
signs or symptoms are usually present. Bacterial infections more commonly result in an
increase in neutrophils. One exception to this is Bordetella pertussis, which has been
known to cause absolute lymphocyte counts of up to 70,000/L. This infection is
associated with classic symptoms that this patient does not have.
271.
A 24-year-old female presents with
pelvic pain. She says that the pain is
present on most days, but is worse
during her menses. Ibuprofen has
helped in the past but is no longer
effective. Her menses are normal and
she has only one sexual partner. A
physical examination is normal.
A. Transvaginal ultrasonography. The initial evaluation for chronic pelvic pain should
include a urinalysis and culture, cervical swabs for gonorrhea and Chlamydia, a CBC, an
erythrocyte sedimentation rate, a -hCG level, and pelvic ultrasonography. CT and MRI are
not part of the recommended initial diagnostic workup, but may be helpful in further
assessing any abnormalities found on pelvic ultrasonography. Referral for diagnostic
laparoscopy is appropriate if the initial workup does not reveal a source of the pain, or if
endometriosis or adhesions are suspected. Colonoscopy would be indicated if the history
or examination suggests a gastrointestinal source for the pain after the initial evaluation.
Which one of the following should be
the next step in the workup of this
patient? (check one)
A. Transvaginal ultrasonography
B. CT of the abdomen and pelvis
C. MRI of the pelvis
D. A CA-125 level
E. Colonoscopy
272.
A 24-year-old female who works at a
day-care facility presents to your
office to discuss ways to avoid getting
"all the infections the kids get." She
plans to enroll her child in the facility.
She is specifically concerned about
diarrheal illnesses, and a friend has
suggested the use of probiotics.
(check one)
A. can lessen the severity and duration
of infectious diarrhea
B. are recommended only for patients
who are immunocompromised
C. have no known side effects
D. often interact with common
prescription medications
E. are not appropriate for use in
children
A. can lessen the severity and duration of infectious diarrhea. Probiotics are
microorganisms with likely health benefits, based on recent randomized, controlled trials.
Good evidence suggests that probiotics reduce the incidence, duration, and severity of
antibiotic-associated and infectious diarrhea. Common side effects include flatulence and
abdominal pain. Contraindications include short-gut syndromes and
immunocompromised states. There are no known drug interactions, and these agents
appear safe for all ages (SOR A).
273.
A 24-year-old female with a 2-year history of dyspnea on
exertion has been diagnosed with exercise-induced asthma
by another physician. Which one of the following findings
on pulmonary function testing would raise concerns that
she actually has vocal cord dysfunction? (check one)
A. A good response to an inhaled -agonist
B. Flattening of the inspiratory portion of the flow-volume
loop, but a normal expiratory phase
C. Flattening of the expiratory portion of the flow-volume
loop, but a normal inspiratory phase
D. Flattening of both the inspiratory and expiratory
portion of the flow-volume loop
E. A decreased FEV1 and a normal FVC
274.
A 24-year-old female with a past history of asthma presents
to the emergency department with an asthma
exacerbation. Treatment with an inhaled bronchodilator
and ipratropium (Atrovent) does not lead to significant
improvement, and she is admitted to the hospital for
ongoing management. On examination she is afebrile, her
respiratory rate is 24/min, her pulse rate is 92 beats/min,
and oxygen saturation is 92% on room air. She has diffuse
bilateral inspiratory and expiratory wheezes with mild
intercostal retractions. Which one of the following should
be considered in the acute management of this patient?
(check one)
A. Chest physical therapy
B. Inhaled fluticasone/salmeterol (Advair)
C. Oral azithromycin (Zithromax)
D. Oral prednisone
E. Oral theophylline
B. Flattening of the inspiratory portion of the flow-volume loop, but a
normal expiratory phase. The diagnosis of vocal cord dysfunction
should be considered in patients diagnosed with exercise-induced
asthma who do not have a good response to -agonists before
exercise. Pulmonary function testing with a flow-volume loop typically
shows a normal expiratory portion but a flattened inspiratory phase
(SOR C). A decreased FEV1 and normal FVC would be consistent with
asthma.
D. Oral prednisone. Hospital management of acute exacerbations of
asthma should include inhaled short-acting bronchodilators in all
patients. Systemic corticosteroids are recommended for all patients
admitted to the hospital. The efficacy of oral prednisone has been
shown to be equivalent to that of intravenous methylprednisolone
(SOR A). Oxygen should also be considered in most patients.
Antibiotics are not recommended in the treatment of asthma
exacerbations unless there is a comorbid infection. Inhaled
ipratropium bromide is recommended for treatment in the
emergency department, but not in the hospital (SOR A). Chest
physical therapy and methylxanthines are not recommended in the
treatment of acute asthma exacerbations.
275.
A 24-year-old gravida 1 para 1 who is 2
weeks post partum complains of
double vision, shortness of breath, and
almost dropping her baby while trying
to hold her. She says her symptoms
worsen as the day progresses. She has
no family history of neurologic or
muscular illness. A physical
examination is normal except for
unilateral ptosis and 4/5 proximal
weakness of both arms. Breath sounds
are generally decreased. Routine blood
tests, including TSH and creatine kinase
levels, are normal. A chest radiograph
and an MRI of the brain and cervical
spine are also normal. Of the following,
this presentation is most consistent
with (check one)
D. myasthenia gravis. Common neurologic disorders in young women include multiple
sclerosis, Guillain-Barr syndrome, and myasthenia gravis. Myasthenia gravis is part of
the differential diagnosis for sudden neurologic weakness, and Guillain-Barr syndrome
must also be considered in this patient. Multiple sclerosis would not result in respiratory
compromise. Myasthenia gravis is an autoimmune neuromuscular disease characterized
by varying degrees of skeletal muscle weakness. Symptoms, which vary in type and
severity, may include ptosis of one or both eyelids; blurred vision; diplopia; unstable gait;
weakness in the arms, hands, fingers, legs, and neck; difficulty swallowing; shortness of
breath; and impaired speech (dysarthria). In most cases, the first noticeable symptom is
weakness of the eye muscles. Muscles that control respiration and neck and limb
movements may also be affected. Symptoms typically worsen through the day or as the
muscles are repetitively used, and improve with rest. Fibromyalgia does not produce
objective neurologic findings, and Sheehan's syndrome would not cause a localized
neurologic deficit. In addition, the TSH level would be low or zero, and the MRI of the
brain would be abnormal. An MRI of the brain would also be abnormal if stroke
symptoms had been present for 2 weeks. The patient is unlikely to have unilateral
symptoms with polymyositis, and creatine kinase would be elevated.
A. fibromyalgia syndrome
B. Sheehan's syndrome (postpartum
hypopituitarism)
C. polymyositis
D. myasthenia gravis
E. stroke
276.
A 24-year-old male, new to your
practice, presents for a mental health
evaluation. The patient has a past
history of schizophrenia, diagnosed
several years ago. Which one of the
following, if present, would lead to a
reconsideration of this diagnosis?
(check one)
A. Auditory hallucinations
B. Loose associations
C. Elated mood
D. Social dysfunction
E. Incoherent speech
C. Elated mood. Schizophrenia can be very difficult to definitively diagnose, and there are
many subtypes. There are many sets of diagnostic criteria, but most, including DSM-IV,
include the presence of thought disorders such as hallucinations, delusions, and loose
associations; disorganized speech; catatonic behavior; and apathy or flat affect. (Two of
these must be present to meet DSM-IV criteria.) Additionally, there must be social or
occupational impairment and a minimum duration of symptoms (6 months for DSM-IV).
Mood disorders, including depression, mania, and schizoaffective disorder, must be
excluded in order to diagnose schizophrenia. Obviously, treatment of these disorders is
very different from that of schizophrenia.
277.
A 24-year-old male presents with a fever of 38.9C
(102.0F), generalized body aches, a sore throat, and
a cough. His symptoms started 24 hours ago. He is
otherwise healthy. You suspect novel influenza A
H1N1 infection, as there have been numerous cases
in your community recently. A rapid influenza
diagnostic test is positive, and you recommend
over-the-counter symptomatic treatment. You see
him 2 days later after he is admitted to the hospital
through the emergency department with
dehydration and mild respiratory distress. A
specimen is sent to the state laboratory for PCR
testing.
Which one of the following would be most
appropriate at this point? (check one)
A. Oseltamivir (Tamiflu). The currently circulating novel influenza A H1N1
virus is almost always susceptible to neuraminidase inhibitors (oseltamivir
and zanamivir) and resistant to the adamantanes (amantadine and
rimantadine). Zanamivir should not be used in patients with COPD, asthma,
or respiratory distress. Antiviral treatment of influenza is recommended for
all persons with clinical deterioration requiring hospitalization, even if the
illness started more than 48 hours before admission. Antiviral treatment
should be started as soon as possible. Waiting for laboratory confirmation is
not recommended.
A. Oseltamivir (Tamiflu)
B. Zanamivir (Relenza)
C. Amantadine (Symmetrel)
D. Rimantadine (Flumadine)
E. No antiviral treatment
278.
A 24-year-old male sustains a boxer's fracture of the
fifth metacarpal. A radiograph shows no rotational
deformity and 25 of volar angulation. After an
attempt at closed reduction the angulation remains
unchanged.
Which one of the following would be most
appropriate at this time?
(check one)
D. An ulnar gutter splint. Up to 40 of volar angulation is acceptable for fifth
metacarpal fractures. For second and third metacarpal fractures, less
angulation is acceptable. Appropriate treatment is a gutter splint.
A. Open reduction
B. Placement of a pin to prevent further
displacement
C. A short arm-thumb spica cast
D. An ulnar gutter splint
279.
A 24-year-old primigravida has nausea and vomiting
associated with pregnancy. Which one of the
following is recommended by the American
Congress of Obstetricians and Gynecologists (ACOG)
as first-line therapy? (check one)
A. Droperidol (Inapsine)
B. Ondansetron (Zofran)
C. Prochlorperazine
D. Metoclopramide (Reglan)
E. Doxylamine (Unisom) and vitamin B6
E. Doxylamine (Unisom) and vitamin B6. Approximately 10% of women with
nausea and vomiting during pregnancy require medication. Pharmacologic
therapies that have been used include vitamin B6 , antihistamines, and
prokinetic agents, as well as other medications. Randomized, placebocontrolled trials have shown that vitamin B6 is effective for this problem. The
combination of vitamin B 6 and doxylamine was studied in more than 6000
patients and was associated with a 70% reduction in nausea and vomiting,
with no evidence of teratogenicity. It is recommended by the American
Congress of Obstetricians and Gynecologists as first-line therapy for nausea
and vomiting in pregnancy. A combination pill was removed from the U.S.
market in 1983 because of unjustified concerns about teratogenicity, but the
medications can be bought separately over the counter.
In rare cases, metoclopramide has been associated with tardive dyskinesia,
and the FDA has issued a black-box warning concerning the use of this drug
in general. The 5-HT3 -receptor antagonists, such as ondansetron, are being
used for hyperemesis in pregnancy, but information is limited. Droperidol
has been used for this problem in the past, but it is now used infrequently
because of its risks, particularly heart arrhythmias.
280.
A 24-year-old white female presents to the
office with a 6-month history of abdominal
pain. A physical examination, including pelvic
and rectal examinations, is normal. Which
one of the following would indicate a need for
further evaluation? (check one)
E. Worsening of symptoms at night. Irritable bowel syndrome (IBS) is a benign,
chronic symptom complex of altered bowel habits and abdominal pain. It is the
most common functional disorder of the gastrointestinal tract. The presence of
nocturnal symptoms is a red flag which should alert the physician to an alternate
diagnosis and may require further evaluation. The other symptoms listed are
Rome I and II criteria for diagnosing irritable bowel syndrome.
A. Relief of symptoms with defecation
B. Changes in stool consistency from loose
and watery to constipation
C. Passage of mucus with bowel movements
D. Abdominal bloating
E. Worsening of symptoms at night
281.
A 25-year-old female at 31 weeks gestation
presents to the labor wing with painful
uterine contractions every 3 minutes. On
examination her cervix is 3 cm dilated and
50% effaced. Her membranes are intact and
fetal heart monitoring is reassuring. She is
treated with tocolysis,betamethasone,
antibiotics, and intravenous hydration, and
cultured for group B Streptococcus. The
neonatal intensive care unit is notified, but
the contractions ease and eventually stop.
After 2 days of observation, her cervix is
unchanged and she is discharged home.
One week later, the patient presents with
contractions for the last 8 hours. Her cervical
findings are unchanged. Her group B
Streptococcus culture was negative.
Which one of the following would be the
most appropriate next step in the
management of this patient?
(check one)
A. Repeat tocolysis, betamethasone,
antibiotics, and intravenous hydration
B. Betamethasone, antibiotics, and
intravenous hydration only
C. Antibiotics and intravenous hydration only
D. Tocolysis only
E. Expectant management
E. Expectant management. The purpose of obstetric management of preterm labor
before 34 weeks gestation is to allow time to administer corticosteroids. Treatment
does not substantially delay delivery beyond 1 week. Repeated administration of
corticosteroids does not confer more benefit than a single course. Antibiotics are
administered for prophylaxis of group B Streptococcus and are useful for delaying
delivery if membranes are ruptured. They do not add any benefit otherwise, even
though subclinical amnionitis may be a causative factor in many cases of preterm
labor. Prolonged and repeated tocolysis is believed to be harmful. Tocolysis would
not be indicated in this patient because she has had no cervical change and is
therefore having preterm contractions, not preterm labor. Careful monitoring for
fetal compromise, consultation with obstetric colleagues, and neonatal intensivecare unit involvement should be part of expectant management of preterm labor
cases.
282.
A 25-year-old female at 36 weeks gestation
presents for a routine prenatal visit. Her blood
pressure is 118/78 mm Hg and her urine has no
signs of protein or glucose. Her fundal height
shows appropriate fetal size and she says that she
feels well. On palpation of her legs, you note 2+
pitting edema bilaterally. Which one of the
following is true regarding this patient's
condition? (check one)
C. Her leg swelling requires no further evaluation. Lower-extremity edema is
common in the last trimester of normal pregnancies and can be treated
symptomatically with compression stockings. Edema has been associated with
preeclampsia, but the majority of women who have lower-extremity edema
with no signs of elevated blood pressure will not develop preeclampsia or
eclampsia. For this reason, edema has recently been removed from the
diagnostic criteria for preeclampsia. Disproportionate swelling in one leg
versus another, especially associated with leg pain, should prompt a workup
for deep venous thrombosis but is unlikely given this patient's presentation,
as are cardiac or renal conditions.
A. You should order a 24-hr urine for protein
B. A workup for possible cardiac abnormalities is
necessary
C. Her leg swelling requires no further evaluation
D. She most likely has preeclampsia
E. She most likely has deep venous thrombosis
283.
A 25-year-old female comes to your office
requesting a referral to an otolaryngologist for
surgery on her nose. She states that her nose is too
large and that "something must be done." She has
already seen multiple family physicians, as well as
several otolaryngologists. She is 168 cm (66 in) tall
and weighs 64 kg (141 lb). A physical examination
is normal, and even though she initially resists a
nasal examination, it also is normal. The size of her
nose is normal.
Which one of the following is the most likely cause
of this patient's concern about her nose? (check
one)
D. Body dysmorphic disorder. Body dysmorphic disorder is an increasingly
recognized somatoform disorder that is clinically distinct from obsessivecompulsive disorder, eating disorders, and depression. Patients have a
preoccupation with imagined defects in appearance, which causes emotional
stress. Body dysmorphic disorder may coexist with anorexia nervosa, atypical
depression, obsessive-compulsive disorder, and social anxiety. Cosmetic
surgery is often sought. SSRIs and behavior modification may help, but
cosmetic procedures are rarely helpful.
A. Obsessive-compulsive disorder
B. Anorexia nervosa
C. Depression
D. Body dysmorphic disorder
284.
A 25-year-old female has an annular rash on the
dorsal surface of both hands. The rash does not
respond to initial treatment with an antifungal
medication, and a biopsy reveals granuloma
annulare.
Which one of the following would be the most
appropriate advice for this patient?
(check one)
A. Allow the rash to resolve without further
treatment
B. Cover the rash because it is contagious
C. Treat the rash with systemic corticosteroids
D. Treat the rash with a stronger antifungal
medication
A. Allow the rash to resolve without further treatment. Granuloma annulare is
a self-limited condition. It is not contagious, and therefore would not need to
be covered to prevent transmission. Treatments may include injected or
topical corticosteroids, but oral corticosteroids have not been specifically
recommended. It may be necessary to refer the patient to a dermatologist
because many of the potential treatments can have serious side effects.
285.
A 25-year-old female has been trying to
conceive for over 1 year without success.
Her menstrual periods occur
approximately six times per year.
Laboratory evaluation of her hormone
status has been negative, and her husband
has a normal semen analysis. Her only
other medical problem is hirsutism,
which has not responded to topical
treatment. Pelvic ultrasonography of her
uterus and ovaries is unremarkable.
A. Metformin (Glucophage). This patient fits the criteria for polycystic ovary syndrome
(oligomenorrhea, acne, hirsutism, hyperandrogenism, infertility). Symptoms also
include insulin resistance. Evidence of polycystic ovaries is not required for the
diagnosis.
Metformin has the most evidence supporting its use in this situation, and is the only
treatment listed that is likely to decrease hirsutism and improve insulin resistance and
menstrual irregularities. Metformin and clomiphene alone or in combination are firstline agents for ovulation induction. Clomiphene does not improve hirsutism, however.
Progesterone is not indicated for any of this patient's problems. Spironolactone will
improve hirsutism and menstrual irregularities, but is not indicated for ovulation
induction.
Of the following, which one would be the
most appropriate treatment for her
infertility? (check one)
A. Metformin (Glucophage)
B. Danazol
C. Medroxyprogesterone (Provera)
D. Spironolactone (Aldactone)
286.
A 25-year-old female is in active labor at
term and is dilated to 7 cm. An electronic
fetal monitoring tracing is shown in
Figure 5.
...
287.
A 25-year-old female presents with
abdominal pain localized to the right
lower quadrant. Which one of the
following would be most helpful in
diagnosing acute appendicitis? (check
one)
E. Abdominal/pelvic CT. Seventy to ninety percent of patients with acute appendicitis
have leukocytosis, but this is also a characteristic of other conditions, and thus has poor
specificity for acute appendicitis. The urinalysis may exhibit microscopic pyuria or
hematuria in a patient with acute appendicitis, but these findings may also be present
with urinary tract disease. Plain radiographs of the abdomen are of limited value in
diagnosing acute appendicitis. Ultrasonography can be useful, especially in ruling out
gynecologic problems, but is technician-dependent and is not as specific nor sensitive
as CT scanning, which has a sensitivity, specificity, and overall accuracy in excess of 90%.
In cases where the CT scan is indeterminate, patients should be admitted to the
hospital for close observation with repeated physical examinations to monitor clinical
status.
A. A CBC
B. Urinalysis
C. Plain abdominal films
D. Abdominal/pelvic ultrasonography
E. Abdominal/pelvic CT
288.
A 25-year-old female presents with a
maculopapular rash that has progressed
to multiple areas and exhibits target
lesions. A cold sore appeared on her upper
lip 2 days before the rash appeared. She is
not systemically ill and is on no
medications.
Which one of the following is true
concerning this problem?
(check one)
A. Herpes simplex virus is a likely cause
B. A skin biopsy will confirm the diagnosis
C. The lesions usually disappear within 24
hours
D. The palms of the hands and soles of the
feet are not involved
E. Scarring from the lesions is often seen
after resolution
A. Herpes simplex virus is a likely cause. Herpes simplex virus is the most common
etiologic agent of erythema multiforme. Other infections, particularly Mycoplasma
pneumoniae infections and fungal infections, may also be associated with this
hypersensitivity reaction. Other causes include medications and vaccines. Skin biopsy
findings are not specific for erythema multiforme. As opposed to the lesions of urticaria,
the lesions of erythema multiforme usually are present and fixed for at least 1 week
and may evolve into target lesions. The palms of the hands and soles of the feet may be
involved. The lesions of erythema multiforme usually resolve spontaneously over 3-5
weeks without sequelae.
289.
A 25-year-old female sees you in the office for follow-up after
a visit to the emergency department for respiratory distress.
She complains of several episodes of an acute onset of
shortness of breath, wheezing, coughing, and a choking
sensation, without any obvious precipitant. She has been on
inhaled corticosteroids for 2 months without any
improvement in her symptoms. Albuterol (Proventil,
Ventolin) does not consistently relieve her symptoms. She is
asymptomatic today. Spirometry shows a normal FEV1 , a
normal FVC and FEV1 /FVC ratio, and a flattened inspiratory
loop.
The most likely diagnosis is: (check one)
A. globus hystericus
B. vocal cord dysfunction
C. asthma
D. anaphylaxis
E. COPD
B. vocal cord dysfunction. Vocal cord dysfunction is an idiopathic
disorder commonly seen in patients in their twenties and thirties in
which the vocal cords partially collapse or close on inspiration. It
mimics, and is commonly mistaken for, asthma. Symptoms include
episodic tightness of the throat, a choking sensation, shortness of
breath, and coughing. A careful history and examination reveal
that the symptoms are worse with inspiration than with exhalation,
and inspiratory stridor during the episode may be mistaken for
the wheezing of asthma. The sensation of throat tightening or
choking also helps to differentiate it from asthma.
Pulmonary function tests (PFTs) are normal, with the exception of
flattening of the inspiratory loop, which is diagnostic of extrathoracic airway compression. Fiberoptic laryngoscopy shows
paradoxical inspiratory and/or expiratory partial closure of the
vocal cords. Vocal cord dysfunction is treated with speech therapy,
breathing techniques, reassurance, and breathing a heliumoxygen mixture (heliox).
PFTs in patients with asthma are normal between exacerbations,
but when symptoms are present the FEV1 /FVC ratio is reduced, as
with COPD. With anaphylaxis, there will typically be itching or
urticaria and signs of angioedema, such as lip or tongue swelling, in
response to a trigger such as food or medication; PFTs are normal
when anaphylaxis symptoms are absent. Globus hystericus is a
type of conversion disorder in which emotional stress causes a
subjective sensation of pain or tightness in the throat, and/or
dysphagia; diagnostic tests such as spirometry and laryngoscopy
are normal.
290.
A 25-year-old Hispanic